*NURSING > QUESTIONS & ANSWERS > Physical Examination and Health Assessment test bank (summer 2020) A+ guide; All Chapters. (All)

Physical Examination and Health Assessment test bank (summer 2020) A+ guide; All Chapters.

Document Content and Description Below

Physical Examination and Health Assessment 8th Edition Chapter 1 - Evidence-Based Assessment 3 Chapter 2 - Cultural Assessment 13 Chapter 3 - The Interview 26 Chapter 4 - The Complete Health Histo... ry 43 Chapter 5 - Mental Status Assessment 55 Chapter 6 - Substance Use Assessment 70 Chapter 7 - Domestic and Family Violence Assessment 76 Chapter 8 - Assessment Techniques and Safety in the Clinical Setting 82 Chapter 9 - General Survey and Measurement 97 Chapter 10 - Vital Signs 102 Chapter 11 - Pain Assessment 116 Chapter 12 - Nutrition Assessment 123 Chapter 13 - Skin Hair and Nails 134 Chapter 14 - Head Face Neck and Regional Lymphatics 152 Chapter 15 - Eyes 166 Chapter 16 - Ears 180 Chapter 17 - Nose Mouth and Throat 195 Chapter 18 - Breasts Axillae and Regional Lymphatics 210 Chapter 19 - Thorax and Lungs 226 Chapter 20 - Heart and Neck Vessels 241 Chapter 21 - Peripheral Vascular System and Lymphatic System 255 Chapter 22 - Abdomen 269 Chapter 23 - Musculoskeletal System 282 Chapter 24 - Neurologic System 299 Chapter 25 - Male Genitourinary System 320 Chapter 26 - Anus Rectum and Prostate 334 Chapter 27 - Female Genitourinary System 344 Chapter 28 - The Complete Health Assessment Adult 362 Chapter 29 - The Complete Physical Assessment Infant Young Child and Adolescent 367 Physical Examination and Health Assessment 8th Edition 0323510809Chapter 30 - Bedside Assessment and Electronic Documentation 369 Chapter 31 - The Pregnant Woman 374 Chapter 32 - Functional Assessment of the Older Adult 385 Chapter 01: Evidence-Based Assessment Jarvis: Physical Examination and Health Assessment, 8th Edition MULTIPLE CHOICE 1. After completing an initial assessment of a patient, the nurse has charted that his respirations are eupneic and his pulse is 58 beats per minute. What type of assessment data is this? a. Objective b. Reflective c. Subjective d. Introspective 2. A patient tells the nurse that he is very nervous, nauseous, and “feels hot.” What type of assessment data is this? a. Objective b. Reflective c. Subjective d. Introspective 3. What do the patient’s record, laboratory studies, objective data, and subjective data combine to form? a. Database b. Admitting data c. Financial statement d. Discharge summary 4. When listening to a patient’s breath sounds, the nurse is unsure of a sound that is heard. Which action should the nurse take next? a. Notify the patient’s physician. b. Document the sound exactly as it was heard. c. Validate the data by asking another nurse to listen to the breath sounds. d. Assess again in 20 minutes to note whether the sound is still present. 5. The nurse is conducting a class for new graduate nurses. While teaching the class, what should the nurse keep in mind regarding what novice nurses, without a background of skills and experience from which to draw upon, are more likely to base their decisions on? a. Intuition b. A set of rules c. Articles in journals d. Advice from supervisors 6. The nurse is reviewing information about evidence-based practice (EBP). Which statement best reflects EBP? a. EBP relies on tradition for support of best practices. b. EBP is simply the use of best practice techniques for the treatment of patients. c. EBP emphasizes the use of best evidence with the clinician’s experience. d. EBP does not consider the patient’s own preferences as important. 7. The nurse is conducting a class on priority setting for a group of new graduate nurses. Which is an example of a first-level priority problem? a. Patient with postoperative pain b. Newly diagnosed patient with diabetes who needs diabetic teaching c. Individual with a small laceration on the sole of the foot d. Individual with shortness of breath and respiratory distress 8. When considering priority setting of problems, the nurse keeps in mind that second-level priority problems include which of these aspects? a. Low self-esteem b. Lack of knowledge c. Abnormal laboratory values d. Severely abnormal vital signs 9. Which critical-thinking skill helps the nurse see relationships among the data? a. Validation b. Clustering related cues c. Identifying gaps in data d. Distinguishing relevant from irrelevant 10. The nursing process is a sequential method of problem solving that nurses use and includes which steps? a. Assessment, treatment, planning, evaluation, discharge, and follow-up b. Admission, assessment, diagnosis, treatment, and discharge planning c. Admission, diagnosis, treatment, evaluation, and discharge planning d. Assessment, diagnosis, outcome identification, planning, implementation, and evaluation 11. A newly admitted patient is in acute pain, has not been sleeping well lately, and is having difficulty breathing. How should the nurse prioritize these problems? a. Breathing, pain, and sleep b. Breathing, sleep, and pain c. Sleep, breathing, and pain d. Sleep, pain, and breathing 12. Which is a barrier to incorporating EBP? a. Nurses’ lack of research skills in evaluating the quality of research studies b. Lack of significant research studies c. Insufficient clinical skills of nurses d. Inadequate physical assessment skills 13. During a staff meeting, nurses discuss the problems with accessing research studies to incorporate evidence-based clinical decision making into their practice. Which suggestion by the nurse manager would best help these problems? a. Form a committee to conduct research studies. b. Post published research studies on the unit’s bulletin boards. c. Encourage the nurses to visit the library to review studies. d. Teach the nurses how to conduct electronic searches for research studies. 14. When reviewing the concepts of health, the nurse recalls that the components of holistic health include which of these? a. Disease originates from the external environment. b. The individual human is a closed system. c. Nurses are responsible for a patient’s health state. d. Holistic health views the mind, body, and spirit as interdependent. 15. The nurse recognizes that which concept of prevention is essential in describing health? a. Disease can be prevented by treating the external environment. b. The majority of deaths among Americans under age 65 years are not preventable. c. Prevention places the emphasis on the link between health and personal behavior. d. The means to prevention is through treatment provided by primary health care practitioners. 16. The nurse is performing a physical assessment on a newly admitted patient. Which is an example of objective information obtained during the physical assessment? a. Patient’s history of allergies b. Patient’s use of medications at home c. Last menstrual period 1 month ago d. 2  5 cm scar on the right lower forearm 17. A visiting nurse is making an initial home visit for a patient who has several chronic medical problems. Which type of database is most appropriate to collect in this setting? a. A follow-up database b. A focused database c. A complete database d. An emergency database 18. In which situation is it most appropriate for the nurse to perform a focused or problem-centered history? a. Patient is admitted to a long-term care facility. b. Patient has a sudden and severe shortness of breath. c. Patient is admitted to the hospital for a scheduled surgery. d. Patient in an outpatient clinic has cold and influenza-like symptoms. 19. The clinic nurse is caring for a patient who has been coming to the clinic weekly for blood pressure checks since she changed medications 2 months ago. Which is the most appropriate action for the nurse to take? a. Collect a follow-up database and then check the patient’s blood pressure. b. Ask the patient to read her health record and indicate any changes since her last visit. c. Check the patient’s blood pressure. d. Obtain a complete health history on the patient before checking her blood pressure. 20. A patient is brought by ambulance to the emergency department with multiple injuries received in an automobile accident. He is alert and cooperative, but his injuries are quite severe. How would the nurse proceed with data collection? a. Collect history information first, then perform the physical examination and institute lifesaving measures. b. Simultaneously ask history questions while performing the examination and initiating lifesaving measures. c. Collect all information on the history form, including social support patterns, strengths, and coping patterns. d. Perform lifesaving measures and delay asking any history questions until the patient is transferred to the intensive care unit. 21. A 42-year-old patient of Asian descent is being seen at the clinic for an initial examination. Why is it important for the nurse to consider the basics of the patient’s culture during the patient’s health assessment? a. Identify the cause of his illness. b. Make accurate disease diagnoses. c. The U.S. is becoming increasingly diverse. d. Provide culturally relevant health care. 22. What is a focus of the health professional in the health promotion model? a. Changing people’s perceptions of disease b. Identifying biomedical model interventions c. Identifying negative health acts of people d. Teaching and helping people choose a healthier lifestyle 23. Which statement best describes a proficient nurse? a. Has little experience with a specified population and uses rules to guide performance. b. Has an intuitive grasp of a clinical situation and quickly identifies the accurate solution. c. Sees actions in the context of daily plans for patients. d. Understands a patient situation as a whole rather than a list of tasks and recognizes the long-term goals for the patient. MULTIPLE RESPONSE 1. The nurse is reviewing data collected after an assessment. Of the data listed below, which would be considered related cues that would be clustered together during data analysis? (Select all that apply.) a. Inspiratory wheezes noted in left lower lobes b. Hypoactive bowel sounds c. Nonproductive cough d. Edema, +2, noted on left hand e. Patient reports dyspnea upon exertion f. Rate of respirations 16 breaths per minute OTHER 1. Put the following patient situations in order of the level of priority (from highest priority to lowest priority). 1. First-level priority problem 2. Second-level priority problem 3. Third-level priority problem a. A teenager who was stung by a bee during a soccer match is having trouble breathing. b. A patient newly diagnosed with type 2 diabetes mellitus does not know how to check his own blood glucose levels with a glucometer. c. An older adult with a urinary tract infection is also showing signs of confusion and agitation. Chapter 02: Cultural Assessment Jarvis: Physical Examination and Health Assessment, 8th Edition MULTIPLE CHOICE 1. The nurse is reviewing the characteristics of culture. Which statement is correct regarding the development of one’s culture? a. Learned through language acquisition and socialization b. Genetically determined on the basis of racial background c. A nonspecific phenomenon and is adaptive but unnecessary d. Biologically determined on the basis of physical characteristics 2. During a class on the aspects of culture, the nurse shares that culture has four basic characteristics. Which statement correctly reflects one of the characteristics of culture? a. Static and unchanging b. Members share similar physical characteristics. c. Members share a common geographic origin and religion. d. Adapted to specific conditions r/t environmental and technical factors 3. During a seminar on cultural aspects of nursing, the nurse recognizes that the definition stating “the specific and distinct knowledge, beliefs, customs, and skills acquired by members of a society” reflects which term? a. Norms b. Culture c. Ethnicity d. Assimilation 4. The nurse is discussing the term subculture with a student nurse. Which statement by the nurse would best describe subculture? a. “Fitting as many people as possible into the majority culture.” b. “Identifying small groups of people who do not want to be identified with the larger culture.” c. “Singling out groups of people who suffer differential and unequal treatment as a result of cultural variations.” d. “Recognizing fairly large groups of people with shared characteristics that are not common to all members of a culture.” 5. When reviewing the demographics of ethnic groups in the United States, the nurse recalls that which is the largest and fastest growing population? a. Asian b. Hispanic c. American Indian d. African American/black 6. During an assessment, the nurse notices that a patient is handling a small charm that is tied to a leather strip around his neck. Which action by the nurse is appropriate? a. Ask the patient about the item and its significance. b. Ask the patient to lock the item with other valuables in the hospital’s safe. c. Tell the patient that a family member should take valuables home. d. No action is necessary. 7. The nurse manager is explaining culturally competent care during a staff meeting. Which statement accurately describes the concept of a culturally competent caregiver? a. Able to speak the patient’s native language b. Possesses some basic knowledge of the patient’s cultural background c. Applies the underlying background knowledge of a patient’s culture to provide the best possible health care d. Understands and attends to the total context of the patient’s situation 8. After a class on culture and ethnicity, the new graduate nurse reflects a correct understanding of the concept of ethnicity with which statement? a. “Ethnicity is dynamic and ever changing.” b. “Ethnicity is the belief in a higher power.” c. “Ethnicity pertains to a social group that may possess shared traits such as religion and language.” d. “Ethnicity is learned from birth through the processes of language acquisition and socialization.” 9. The nurse is comparing the concepts of religion and spirituality. Which statement describes an appropriate component of one’s spirituality? a. Belief in and the worship of God or gods b. Being closely tied to one’s ethnic background c. Attendance at a specific church or place of worship d. A connection with something larger than oneself and belief in transcendence 10. A woman who has lived in the United States for a year after moving from Europe has learned to speak English and is almost finished with her college studies. She now dresses like her peers and says that her family in Europe would hardly recognize her. This nurse recognizes that this situation illustrates which concept? a. Integration b. Assimilation c. Biculturalism d. Heritage consistency 11. The nurse is conducting a heritage assessment. Which question is most appropriate for this assessment? a. “Do you smoke?” b. “What is your religion?” c. “Do you have a history of heart disease?” d. “How many years have you lived in the United States?” 12. The nurse is reviewing theories of illness. The germ theory, which states that microscopic organisms such as bacteria and viruses are responsible for specific disease conditions, is a basic belief of which theory of illness? a. Holistic b. Biomedical c. Naturalistic d. Magicoreligious 13. An Asian-American woman is experiencing diarrhea, which is believed to be “cold” or “yin.” What should the nurse recognize that the woman may likely to try to treat it? a. Foods that are “hot” or “yang” b. Readings and Eastern medicine meditations c. High doses of medicines believed to be “cold” d. No treatment because diarrhea is an expected part of life 14. Many Asians believe in the yin/yang theory, which is rooted in the ancient Chinese philosophy of Tao. Which statement most accurately reflects this philosophy’s view of “health”? a. A person is able to work and produce. b. A person is happy, stable, and feels good. c. All aspects of the person are in perfect balance. d. A person is able to care for others and function socially. 15. Illness is caused by an imbalance or disharmony in the forces of nature. This statement most accurately reflects the views about illness from which theory? a. Germ theory b. Naturalistic theory c. Magicoreligious theory d. Biomedical or scientific theory 16. What does an individual who believes the magicoreligious theory of illness and disease believe is the cause of his or her illness? a. Germs and viruses b. Supernatural forces c. Eating imbalanced (hot/cold) foods d. Imbalance within his or her spiritual nature 17. What should the nurse, who is caring for an American Indian woman seeking help to regulate her diabetes, anticipate or expect of the patient? a. Will comply with the treatment prescribed b. Has given up her belief in naturalistic causes of disease c. May also be seeking the assistance of a shaman or medicine man d. Will need extra help in dealing with her illness and may be experiencing a crisis of faith 18. An older Mexican-American woman with traditional beliefs has been admitted to an inpatient care unit. When admitting this patient, what would a culturally sensitive nurse do? a. Contact the hospital administrator about the best course of action. b. Arrange for a shaman for her, because requesting one herself is not culturally appropriate. c. Further assess the patient’s cultural beliefs and offer the patient assistance in contacting a curandero or priest if she desires. d. Ask the family what they would like to do because Mexican-Americans traditionally give control of decision making to their families. 19. A 63-year-old Chinese-American man who recently found out his wife has cancer enters the hospital with complaints of chest pain, shortness of breath, and palpitations. Which statement most accurately reflects the nurse’s best course of action? a. The nurse should focus on psychosomatic complaints. b. The nurse should focus on performing a full cardiac assessment. c. The nurse should send him home with instructions to contact his physician. d. The nurse should perform both physical and psychosocial assessments. 20. Symptoms, such as pain, are often influenced by a person’s cultural heritage. Which of the following is a true statement regarding pain? a. Nurses’ attitudes toward their patients’ pain are unrelated to their own experiences with pain. b. Nurses need to recognize that many cultures practice silent suffering as a response to pain. c. A nurse’s area of clinical practice will most likely determine his or her assessment of a patient’s pain. d. A nurse’s years of clinical experience and current position are strong indicators of his or her response to patient pain. 21. The nurse is reviewing concepts of cultural aspects of pain. Which statement is true regarding pain? a. All patients will behave the same way when in pain. b. Just as patients vary in their perceptions of pain, so will they vary in their expressions of pain. c. Cultural norms have very little to do with pain tolerance, because pain tolerance is always biologically determined. d. A patient’s expression of pain is largely dependent on the amount of tissue injury associated with the pain. 22. During a class on religion and spirituality, the nurse is asked to define spirituality. Which statement by the nurse best describes spirituality? a. “Is a personal search to discover a supreme being.” b. “Is an organized system of beliefs concerning the cause, nature, and purpose of the universe.” c. “Is a belief that each person exists forever in some form, such as a belief in reincarnation or the afterlife.” d. “Focuses on a connection with something bigger than oneself and a belief in transcendence.” 23. When caring for children with a different cultural perspective, what should the nurse recognize may pose a challenge? a. Children have spiritual needs that are influenced by their stages of development. b. Children have spiritual needs that are direct reflections of what is occurring in their homes. c. Religious beliefs rarely affect the parents’ perceptions of the illness. d. Parents are often the decision makers, and they have no knowledge of their children’s spiritual needs. 24. A 30-year-old woman has recently moved to the United States with her husband. They are living with the woman’s sister until they can get a home of their own. When company arrives to visit with the woman’s sister, the woman feels suddenly shy and retreats to the back bedroom to hide until the company leaves. She explains that her reaction to guests is simply because she does not know how to speak “perfect English.” What is this woman likely experiencing? a. Culture shock b. Cultural taboos c. Cultural unfamiliarity d. Culture disorientation 25. After a symptom is recognized, the first effort at treatment is often self-treatment. Which of the following statements is true about self-treatment? a. “Not recognized as valuable by most health care providers.” b. “Usually ineffective and may delay more effective treatment.” c. “Always less expensive than biomedical alternatives.” d. “Influenced by the accessibility of over-the-counter medicines.” 26. The nurse is reviewing the hot/cold theory of health and illness. Which statement best describes the basic tenets of this theory? a. The causation of illness is based on supernatural forces that influence the humors of the body. b. Herbs and medicines are classified on their physical characteristics of hot and cold and the humors of the body. c. The four humors of the body consist of blood, yellow bile, spiritual connectedness, and social aspects of the individual. d. The treatment of disease consists of adding or subtracting cold, heat, dryness, or wetness to restore the balance of the humors of the body. 27. When providing culturally competent care, nurses must incorporate cultural assessments into their health assessments. Which statement is most appropriate to use when initiating an assessment of cultural beliefs with an older American Indian patient? a. “Are you of the Christian faith?” b. “Do you want to see a medicine man?” c. “How often do you seek help from medical providers?” d. “What cultural or spiritual beliefs are important to you?” 28. During a class on cultural practices, the nurse hears the term cultural taboo. Which statement illustrates the concept of a cultural taboo? a. Trying prayer before seeking medical help. b. Believing that illness is a punishment of sin. c. Refusing to accept blood products as part of treatment. d. Stating that a child’s birth defect is the result of the parents’ sins. 29. The nurse should recognize that categories such as ethnicity, gender, and religion illustrate which concept? a. Family b. Cultures c. Spirituality d. Subcultures 30. The nurse is reviewing concepts related to one’s heritage and beliefs. Which concept refers to belief in a divine or superhuman power(s) to be obeyed and worshipped as the creator(s) and ruler(s) of the universe? a. Culture b. Religion c. Ethnicity d. Spirituality 31. When planning a cultural assessment, the nurse should include which component? a. Family history b. Chief complaint c. Medical history d. Health-related beliefs MULTIPLE RESPONSE 1. The nurse is asking questions about a patient’s health beliefs. Which questions are appropriate? (Select all that apply.) a. “What is your definition of health?” b. “Does your family have a history of cancer?” c. “How do you describe illness?” d. “What did your mother do to keep you from getting sick?” e. “Have you ever had any surgeries?” f. “How do you keep yourself healthy?” Chapter 03: The Interview Jarvis: Physical Examination and Health Assessment, 8th Edition MULTIPLE CHOICE 1. The nurse is conducting an interview with a woman who has recently learned that she is pregnant and who has come to the clinic today to begin prenatal care. The woman states that she and her husband are excited about the pregnancy but have a few questions. She looks nervously at her hands during the interview and sighs loudly. Considering the concept of communication, which statement does the nurse know to be most accurate when describing this woman? a. Excited about her pregnancy but nervous about the labor b. Exhibiting verbal and nonverbal behaviors that do not match c. Excited about her pregnancy, but her husband is not and this is upsetting to her d. Not excited about her pregnancy but believes the nurse will negatively respond to her if she states this 2. Receiving is a part of the communication process. Which receiver is most likely to misinterpret a message sent by a health care professional? a. Well-adjusted adolescent who came in for a sports physical b. Recovering alcoholic who came in for a basic physical examination c. Man who came in with his wife who was just diagnosed with lung cancer d. Man with a hearing impairment who has an interpreter with him who came in for a follow-up blood pressure check 3. Which adjustment in the physical environment should the nurse make to promote the success of an interview? a. Arrange seating across a desk or table. b. Reduce noise by turning off televisions and radios. c. Reduce the distance between the interviewer and the patient to 2 feet or less. d. Provide dim lighting to make the room cozy and help the patient relax. 4. In an interview, the nurse may find it necessary to take notes to aid his or her memory later. Which statement is true regarding note-taking? a. Note-taking may impede the nurse’s observation of the patient’s nonverbal behaviors. b. Note-taking allows the patient to continue at his or her own pace as the nurse records what is said. c. Note-taking allows the nurse to shift attention away from the patient, resulting in an increased comfort level. d. Note-taking allows the nurse to break eye contact with the patient, which may increase his or her level of comfort. 5. The nurse asks, “I would like to ask you some questions about your health and your usual daily activities so that we can better plan your stay here.” Based on this question, the nurse is at which phase of the interview process? a. Summary b. Closing c. Working d. Opening or introduction 6. A woman has just entered the emergency department after being battered by her husband. The nurse needs to get some information from her to begin treatment. What is the best choice for an opening phase of the interview with this patient? a. “Hello, Nancy, my name is Nurse C.” b. “Mrs. H., my name is Nurse C. How are you?” c. “Hello, Mrs. H., my name is Nurse C. It sure is cold today!” d. “Mrs. H., my name is Nurse C. I’ll need to ask you a few questions about what happened.” 7. During an interview, the nurse states, “You mentioned having shortness of breath. Tell me more about that.” Which verbal skill is used with this statement? a. Reflection b. Facilitation c. Direct question d. Open-ended question 8. A patient has finished giving the nurse information about the reason he is seeking care. When reviewing the data, the nurse finds that some information about past hospitalizations is missing. At this point, which statement by the nurse would be most appropriate to gather these data? a. “Mr. Y., at your age, surely you have been hospitalized before!” b. “Mr. Y., I just need permission to get your medical records from County Medical.” c. “Mr. Y., you mentioned that you have been hospitalized on several occasions. Would you tell me more about that?” d. “Mr. Y., I just need to get some additional information about your past hospitalizations. When was the last time you were admitted for chest pain?” 9. In using verbal responses to assist the patient’s narrative, some responses focus on the patient’s frame of reference and some focus on the health care provider’s perspective. Which is an example of a verbal response that focuses on the health care provider’s perspective? a. Empathy b. Reflection c. Facilitation d. Confrontation 10. When taking a history from a newly admitted patient, the nurse notices that he often pauses and expectantly looks at the nurse. What would be the nurse’s best response to this behavior? a. Lean forward slightly and making eye contact ask “Is there anything else?” b. Smile at him and say, “Don’t worry about all of this. I’m sure we can get to the bottom of your symptoms.” c. Lean back in the chair and ask, “You are looking at me kind of funny; there isn’t anything wrong, is there?” d. Stand up and say, “I can see that this interview is uncomfortable for you. We can continue it another time.” 11. 12. A woman is discussing the problems she is having with her 2-year-old son. She says, “He won’t go to sleep at night, and during the day he has several fits. I get so upset when that happens.” Which is the best response by the nurse to gain a better understanding of the problem? a. “Go on, I’m listening.” b. “Fits? Tell me what you mean by this.” c. “Yes, it can be upsetting when a child has a fit.” d. “Don’t be upset when he has a fit; every 2 year old has fits.” 13. A 17-year-old single mother is describing how difficult it is to raise a 3-year-old child by herself. During the course of the interview she states, “I can’t believe my boyfriend left me to do this by myself! What a terrible thing to do to me!” Which of these responses by the nurse uses empathy? a. “You feel alone.” b. “You can’t believe he left you alone?” c. “It must be so hard to care for a child all alone.” d. “I would be angry, too; raising a child alone is no picnic.” 14. The nurse has used interpretation regarding a patient’s statement or actions. What should the nurse do after using this technique? a. Apologize, because using interpretation can be demeaning for the patient. b. Allow time for the patient to confirm or correct the inference. c. Continue with the interview as though nothing has happened. d. Immediately restate the nurse’s conclusion on the basis of the patient’s nonverbal response. 15. During an interview, a woman says, “I have decided that I can no longer allow my children to live with their father’s violence, but I just can’t seem to leave him.” Using interpretation, which would be the best response by the nurse? a. “You are going to leave him?” b. “If you are afraid for your children, then why can’t you leave?” c. “It sounds as if you might be afraid of how your husband will respond.” d. “It sounds as though you have made your decision. I think it is a good one.” 16. A pregnant woman states, “I just know labor will be so painful that I won’t be able to stand it. I know it sounds awful, but I really dread going into labor.” The nurse responds by stating, “Oh, don’t worry about labor so much. I have been through it, and although it is painful, many good medications are available to decrease the pain.” Which statement is true regarding the nurse’s response? a. Therapeutic response. By sharing something personal, the nurse gives hope to this woman. b. Nontherapeutic response. By providing false reassurance, the nurse actually cut off further discussion of the woman’s fears. c. Therapeutic response. By providing information about the medications available, the nurse is giving information to the woman. d. Nontherapeutic response. The nurse is essentially giving the message to the woman that labor cannot be tolerated without medication. 17. During a visit to the clinic, a patient states, “The doctor just told me he thought I ought to stop smoking. He doesn’t understand how hard I’ve tried. I just don’t know the best way to do it. What should I do?” What is the most appropriate response by the nurse? a. “I’d quit if I were you. The doctor really knows what he is talking about.” b. “Would you like some information about the different ways a person can quit smoking?” c. “Stopping your dependence on cigarettes can be very difficult. I understand how you feel.” d. “Why are you confused? Didn’t the doctor give you the information about the smoking cessation program we offer?” 18. As the nurse enters a patient’s room, the nurse finds the patient crying. The patient states that she has just found out that the lump in her breast is cancer and says, “I’m so afraid of, um, you know.” Which would be the most therapeutic response by the nurse when said in a gentle manner? a. “You’re afraid you might lose your breast?” b. “No, I’m not sure what you are talking about.” c. “I’ll wait here until you get yourself under control, and then we can talk.” d. “I can see that you are very upset. Perhaps we should discuss this later.” 19. A nurse is taking complete health histories on all of the patients attending a wellness workshop. On the history form, one of the written questions asks, “You don’t smoke, drink, or take drugs, do you?” This is an example of what type of question? a. Talking too much b. Using confrontation c. Using biased or leading questions d. Using blunt language to deal with distasteful topics 20. When observing a patient’s verbal and nonverbal communication, the nurse notices a discrepancy. What action should the nurse take in this situation? a. Ask someone who knows the patient well to help interpret this discrepancy. b. Focus on the patient’s verbal message, and try to ignore the nonverbal behaviors. c. Try to integrate the verbal and nonverbal messages and then interpret them as an average. d. Focus on the patient’s nonverbal behaviors, because these are often more reflective of a patient’s true feelings. 21. During an interview, a parent of a hospitalized child is sitting in a recliner with his legs extended and his arms at his sides. As the interviewer begins to discuss his son’s treatment, however, he suddenly changes positions and crosses his arms against his chest and crosses his legs. What does this change in posture suggest? a. Simply changing positions b. More comfortable in this position c. Tired and needs a break from the interview d. Uncomfortable talking about his son’s treatment 22. A mother brings her 28-month-old daughter into the clinic for a well-child visit. At the beginning of the visit, the nurse focuses attention away from the toddler, but as the interview progresses, the toddler begins to “warm up” and is smiling shyly at the nurse. The nurse will be most successful in interacting with the toddler if which is done next? a. Tickle the toddler, and get her to laugh. b. Stoop down to her level, and ask her about the toy she is holding. c. Continue to ignore her until it is time for the physical examination. d. Ask the mother to leave during the examination of the toddler, because toddlers often fuss less if their parent is not in view. 23. During an examination of a preschool child, the nurse will need to take her blood pressure. What might the nurse do to try to gain the child’s full cooperation? a. Tell the child that the blood pressure cuff is going to give her arm a big hug. b. Tell the child that the blood pressure cuff is asleep and cannot wake up. c. Give the blood pressure cuff a name and refer to it by this name during the assessment. d. Tell the child that by using the blood pressure cuff, we can see how strong her muscles are. 24. A 16-year-old boy has just been admitted to the hospital for overnight observation after being in an automobile accident. What is the best approach for the nurse to use to communicate with him? a. Use periods of silence to communicate respect for him. b. Be totally honest with him, even if the information is unpleasant. c. Tell him that everything that is discussed will be kept totally confidential. d. Use slang language when possible to help him open up. 25. A 75-year-old woman is at the office for a preoperative interview. The nurse is aware that the interview may take longer than interviews with younger people. What is the reason for this? a. An aged person has a longer story to tell. b. An aged person is usually lonely and likes to have someone with whom to talk. c. Aged people lose much of their mental abilities and require longer time to complete an interview. d. As a person ages, he or she is unable to hear; thus the interviewer usually needs to repeat much of what is said. 26. The nurse is interviewing a male patient who has a hearing impairment and came in because of a cold. What techniques would be most beneficial in communicating with this patient? a. Determine the communication method he prefers. b. Avoid using facial and hand gestures because most hearing-impaired people find this degrading. c. Request a sign language interpreter before meeting with him to help facilitate the communication. d. Speak loudly and with exaggerated facial movement when talking with him because doing so will help him lip read. 27. During a prenatal check, a patient begins to cry as the nurse asks her about previous pregnancies. She states that she is remembering her last pregnancy, which ended in miscarriage. Which is the best response by the nurse? a. “I’m so sorry for making you cry!” b. “I can see that you are sad remembering this. It is all right to cry.” c. “Why don’t I step out for a few minutes until you’re feeling better?” d. “I can see that you feel sad about this; why don’t we talk about something else?” 28. The nurse is performing a health interview on a patient who has a language barrier, and no interpreter is available. Which is the best example of an appropriate question for the nurse to ask in this situation? a. “Do you take medicine?” b. “Are you in any discomfort?” c. “Do you have nausea and vomiting?” d. “You have been following your doctor’s orders, haven’t you?” 29. A man arrives at the clinic for his annual wellness physical. He is experiencing no acute health problems. Which question or statement by the nurse is most appropriate when beginning the interview? a. “How is your family?” b. “How is your job?” c. “Tell me about your hypertension.” d. “How has your health been since your last visit?” 30. The nurse says to a patient, “I know it may be hard, but you should do what the doctor ordered because she is the expert in this field.” Which statement is correct about the nurse’s comment? a. It is inappropriate because it shows the nurse’s bias. b. It is appropriate because members of the health care team are experts in their area of patient care. c. This type of comment promotes dependency and inferiority on the part of the patient and is best avoided in an interview situation. d. Using authority statements when dealing with patients, especially when they are undecided about an issue, is necessary at times. 31. A female patient does not speak English well, and the nurse needs to choose an interpreter. Which of the following would be the most appropriate choice? a. Trained interpreter b. Male family member c. Female family member d. Volunteer college student from the foreign language studies department of Care 32. During a follow-up visit, the nurse discovers that a patient has not been taking his insulin on a regular basis. The nurse asks, “Why haven’t you taken your insulin?” Which statement is an appropriate evaluation of this situation? a. This question may place the patient on the defensive. b. This question is an effective way to search for information. c. Discussing his behavior with his wife would have been better. d. A direct question is the best way to discover the reasons for his behavior. 33. The nurse is nearing the end of an interview. Which statement is appropriate at this time? a. “Did we forget something?” b. “Is there anything else you would like to mention?” c. “I need to go on to the next patient. I’ll be back.” d. “While I’m here, let’s talk about your upcoming surgery.” 34. What data should the nurse collect during the interview portion of a health assessment. a. Physical b. Historical c. Objective d. Subjective 35. During an interview, the nurse would expect that most of the interview will take place at what distance? a. Intimate zone b. Personal distance c. Social distance d. Public distance 36. A female nurse is interviewing a male patient who is near the same age as the nurse. During the interview, the patient makes an overtly sexual comment. Which is the best response by the nurse? a. “Stop that immediately!” b. “Oh, you are too funny. Let’s keep going with the interview.” c. “Do you really think I would be interested?” d. “It makes me uncomfortable when you talk that way. Please stop.” MULTIPLE RESPONSE 1. The nurse is conducting an interview. Which of these statements is true regarding open-ended questions? (Select all that apply.) a. Elicit hard facts. b. Allow for self-expression. c. Build and enhance rapport. d. Call for short one- to two-word answers. e. Are used when narrative information is needed. 2. The nurse is conducting an interview in an outpatient clinic and is using a computer to record data. Which are the best uses of the computer in this situation? (Select all that apply.) a. Collect the patient’s data in a direct, face-to-face manner. b. Enter all the data as the patient states them. c. Ask the patient to wait as the nurse enters the data. d. Type the data into the computer after the narrative is fully explored. e. Allow the patient to see the monitor during typing. Chapter 04: The Complete Health History Jarvis: Physical Examination and Health Assessment, 8th Edition MULTIPLE CHOICE 1. The nurse is preparing to conduct a health history. Which of these statements best describes the purpose of a health history? a. To provide an opportunity for interaction between the patient and the nurse b. To provide a form for obtaining the patient’s biographic information c. To document the normal and abnormal findings of a physical assessment d. To provide a database of subjective information about the patient’s past and current health 2. When the nurse is evaluating the reliability of a patient’s responses, which of these statements would be correct? a. Patient has a history of drug abuse and therefore is not reliable. b. Patient provided consistent information and therefore is reliable. c. Patient smiled throughout interview and therefore is assumed reliable. d. Patient would not answer questions concerning stress and therefore is not reliable. 3. A 59-year-old patient tells the nurse that he has ulcerative colitis. He has been having “black stools” for the last 24 hours. How would the nurse best document his reason for seeking care? a. J.M. is a 59-year-old man seeking treatment for ulcerative colitis. b. J.M. came into the clinic complaining of having black stools for the past 24 hours. c. J.M. is a 59-year-old man who states that he has ulcerative colitis and wants it checked. d. J.M. is a 59-year-old man who states that he has been having “black stools” for the past 24 hours. 4. A patient tells the nurse that she has had abdominal pain for the past week. What would be the nurse’s best response? a. “Can you point to where it hurts?” b. “What have you had to eat in the last 24 hours?” c. “Have you ever had any surgeries on your abdomen?” d. “We’ll talk more about that later in the interview.” 5. A 29-year-old woman tells the nurse that she has “excruciating pain” in her back. Which response by the nurse would be appropriate? a. “How does your family react to your pain?” b. “The pain must be terrible. You probably pinched a nerve.” c. “I’ve had back pain myself, and it can be excruciating.” d. “How would you say the pain affects your ability to do your daily activities?” 6. In recording the childhood illnesses of a patient who denies having had any, which note by the nurse would be most accurate? a. Patient denies usual childhood illnesses. b. Patient states he was a “very healthy” child. c. Patient states his sister had measles, but he didn’t. d. Patient denies measles, mumps, rubella, chickenpox, pertussis, and strep throat. 7. A female patient tells the nurse that she has had six pregnancies, with four live births at term and two spontaneous abortions. Her four children are still living. How would the nurse record this information? a. P-6, B-4, (S)Ab-2 b. Grav 6, Term 4, (S)Ab-2, Living 4 c. Patient has had four living babies. d. Patient has been pregnant 6 times. 8. A patient tells the nurse that he is allergic to penicillin. What is the best response by the nurse? a. “Are you allergic to any other drugs?” b. “How often have you received penicillin?” c. “Describe what happens to you when you take penicillin.” d. “I’ll write your allergy on your chart so you won’t receive any penicillin.” 9. The nurse is taking a family history. Which specific disease or problem should be included in the assessment? a. Emphysema b. Head trauma c. Mental illness d. Fractured bones 10. What does the review of systems provide the nurse? a. Physical findings r/t each system b. Information regarding health promotion practices c. An opportunity to teach the patient medical terms d. Information necessary for the nurse to diagnose the patient’s medical problem 11. What information obtained by the nurse regarding a patient’s skin should the nurse record in the patient’s health history? a. Skin appears dry. b. No lesions are obvious. c. Patient denies any color change. d. Lesion is noted on the lateral aspect of the right arm. 12. The nurse is obtaining a history from a 30-year-old male patient and is concerned about health promotion activities. Which of these questions would be appropriate to use to assess health promotion activities for this patient? a. “Do you perform testicular self-examinations?” b. “Have you ever noticed any pain in your testicles?” c. “Have you had any problems with passing urine?” d. “Do you have any history of sexually transmitted infections?” 13. Which of these responses might the nurse expect during a functional assessment of a patient whose leg is in a cast? a. “I broke my right leg in a car accident 2 weeks ago.” b. “The pain is decreasing, but I still need to take acetaminophen.” c. “I check the color of my toes every evening just like I was taught.” d. “I’m able to transfer myself from the wheelchair to the bed without help.” 14. In response to a question about stress, a 39-year-old woman tells the nurse that her husband and mother both died in the past year. Which response by the nurse is most appropriate? a. “This has been a difficult year for you.” b. “I don’t know how anyone could handle that much stress in 1 year!” c. “What did you do to cope with the loss of both your husband and mother?” d. “That is a lot of stress; now let’s go on to the next section of your history.” 15. In response to a question regarding the use of alcohol, a patient asks the nurse why the nurse needs to know. What is the reason for needing this information? a. This information is necessary to determine the patient’s reliability. b. Alcohol can interact with all medications and can make some diseases worse. c. The nurse needs to be able to teach the patient about the dangers of alcohol use. d. This information is not necessary unless a drinking problem is obvious. 16. The mother of a 16-month-old toddler tells the nurse that her daughter has an earache. What would be an appropriate response by the nurse? a. “Maybe she is just teething.” b. “I will check her ear for an ear infection.” c. “Are you sure she is really having pain?” d. “Describe what she is doing to indicate she is having pain.” 17. During an assessment of a patient’s family history, the nurse constructs a genogram. Which statement best describes a genogram? a. List of diseases present in a person’s near relatives b. Graphic family tree that uses symbols to depict the gender, relationship, and age of immediate family members c. Drawing that depicts the patient’s family members up to five generations back d. Description of the health of a person’s children and grandchildren 18. A 5-year-old boy is being admitted to the hospital to have his tonsils removed. Which information should the nurse collect before this procedure? a. Child’s birth weight b. Age at which he crawled c. Whether the child has had the measles d. Child’s reactions to previous hospitalizations 19. As part of the health history of a 6-year-old boy at a clinic for a sports physical examination, the nurse reviews his immunization record and notes that his last measles-mumps-rubella (MMR) vaccination was at 15 months of age. What should the nurse recommend? a. No further MMR immunizations are needed. b. MMR vaccination needs to be repeated at 4 to 6 years of age. c. MMR immunization needs to be repeated every 4 years until age 21. d. A recommendation cannot be made until the physician is consulted. 20. In obtaining a review of systems on a “healthy” 7-year-old girl, what should the health care provider be sure to include? a. Last glaucoma examination b. Frequency of breast self-examinations c. Date of her last electrocardiogram d. Limitations r/t her involvement in sports activities 21. When the nurse asks for a description of who lives with a child, the method of discipline, and the support system of the child, what part of the assessment is being performed? a. Family history b. Review of systems c. Functional assessment d. Reason for seeking care 22. The nurse is performing a functional assessment on an 82-year-old patient who recently had a stroke. Which of these questions would be most important to ask? a. “Do you wear glasses?” b. “Are you able to dress yourself?” c. “Do you have any thyroid problems?” d. “How many times a day do you have a bowel movement?” 23. The nurse is preparing to do a functional assessment. Which statement best describes the purpose of a functional assessment? a. The functional assessment assesses how the individual is coping with life at home. b. It determines how children are meeting developmental milestones. c. The functional assessment can identify any problems with memory the individual may be experiencing. d. It helps determine how a person is managing day-to-day activities. 24. The nurse is asking a patient for his reason for seeking care and asks about the signs and symptoms he is experiencing. Which of these is an example of a symptom? a. Chest pain b. Clammy skin c. Serum potassium level at 4.2 mEq/L d. Body temperature of 100 F 25. A patient is describing his symptoms to the nurse. Which of these statements reflects a description of the setting of his symptoms? a. “It is a sharp, burning pain in my stomach.” b. “I also have the sweats and nausea when I feel this pain.” c. “I think this pain is telling me that something bad is wrong with me.” d. “This pain happens every time I sit down to use the computer.” 26. During an assessment, the nurse uses the CAGE test. The patient answers “yes” to two of the questions. What could this be indicating? a. The patient is an alcoholic. b. The patient is annoyed at the questions. c. The patient should be thoroughly examined for possible alcohol withdrawal symptoms. d. The nurse should suspect alcohol abuse and continue with a more thorough substance-abuse assessment. 27. The nurse is incorporating a person’s spiritual values into the health history. Which of these questions illustrates the “community” portion of the FICA (faith and belief, importance and influence, community, and addressing or applying in care) questions? a. “Do you believe in God?” b. “Are you a part of any religious or spiritual congregation?” c. “Do you consider yourself to be a religious or spiritual person?” d. “How does your religious faith influence the way you think about your health?” 28. The nurse is preparing to complete a health assessment on a 16-year-old girl whose parents have brought her to the clinic. Which instruction would be appropriate for the parents before the interview begins? a. “It would help to interview the three of you together.” b. “While I interview your daughter, will you step out to the waiting room and complete these family health history questionnaires?” c. “Please stay during the interview; you can answer for her if she does not know the answer.” d. “While I interview your daughter, will you please stay in the room and complete these family health history questionnaires?” 29. The nurse is assessing a new patient who has recently immigrated to the United States. Which question is appropriate to add to the health history? a. “Why did you come to the United States?” b. “When did you come to the United States and from what country?” c. “What made you leave your native country?” d. “Are you planning to return to your home?” MULTIPLE RESPONSE 1. The nurse is assessing a patient’s headache pain. Which questions reflect one or more of the critical characteristics of symptoms that should be assessed? (Select all that apply.) a. “Where is the headache pain?” b. “Did you have these headaches as a child?” c. “On a scale of 1 to 10, how bad is the pain?” d. “How often do the headaches occur?” e. “What makes the headaches feel better?” f. “Do you have any family history of headaches?” 2. The nurse is conducting a developmental history on a 5-year-old child. Which questions are appropriate to ask the parents for this part of the assessment? (Select all that apply.) a. “Can he tell time?” b. “Does he have any food allergies?” c. “Is he able to tie his shoelaces?” d. “Does he take a children’s vitamin?” e. “How much junk food does your child eat?” f. “How many teeth has he lost, and when did he lose them?” Chapter 05: Mental Status Assessment Jarvis: Physical Examination and Health Assessment, 8th Edition MULTIPLE CHOICE 1. During an examination, the nurse can assess mental status by which activity? a. Examining the patient’s electroencephalogram b. Observing the patient as he or she performs an intelligence quotient (IQ) test c. Observing the patient and inferring health or dysfunction d. Examining the patient’s response to a specific set of questions 2. The nurse is assessing the mental status of a child. Which statement about children and mental status is true? a. All aspects of mental status in children are interdependent. b. Children are highly labile and unstable until the age of 2 years. c. A child’s mental status is impossible to assess until the child develops the ability to concentrate. d. Children’s mental status is largely a function of their parents’ level of functioning until the age of 7 years. 3. The nurse is assessing a 75-year-old man. What should the nurse expect when performing the mental status portion of the assessment? a. Will have no decrease in any of his abilities, including response time. b. Will have difficulty on tests of remote memory because this ability typically decreases with age. c. May take a little longer to respond, but his general knowledge and abilities should not have declined. d. Will exhibit a decrease in his response time because of the loss of language and a decrease in general knowledge. 4. When assessing aging adults, what is one of the first things the nurse should assess before making judgments about the aging person’s mental status? a. Presence of phobias b. General intelligence c. Sensory-perceptive abilities d. Presence of irrational thinking patterns 5. The nurse is preparing to conduct a mental status examination. Which statement is true regarding the mental status examination? a. A patient’s family is the best resource for information about the patient’s coping skills. b. Gathering mental status information during the health history interview is usually sufficient. c. Integrating the mental status examination into the health history interview takes an enormous amount of extra time. d. To get a good idea of the patient’s level of functioning, performing a complete mental status examination is usually necessary. 6. A woman brings her husband to the clinic for an examination. She is particularly worried because after a recent fall, he seems to have lost a great deal of his memory of recent events. Which statement reflects the nurse’s best course of action? a. Perform a complete mental status examination. b. Refer him to a psychometrician. c. Plan to integrate the mental status examination into the history and physical examination. d. Reassure his wife that memory loss after a physical shock is normal and will soon subside. 7. The nurse is conducting a patient interview. Which statement made by the patient should the nurse more fully explore to assess the mental status during the interview? a. “I sleep like a baby.” b. “I have no health problems.” c. “I never did too good in school.” d. “I am not currently taking any medications.” 8. A patient is admitted to the unit after an automobile accident. The nurse begins the mental status examination and finds that the patient has dysarthric speech and is lethargic. How should the nurse proceed? a. Defer the rest of the mental status examination. b. Skip the language portion of the examination and proceed onto assessing mood and affect. c. Conduct an in-depth speech evaluation and defer the mental status examination to another time. d. Proceed with the examination and assess the patient for suicidal thoughts because dysarthria is often accompanied by severe depression. 9. A 19-year-old woman comes to the clinic at the insistence of her brother. She is wearing black combat boots and a black lace nightgown over the top of her other clothes. Her hair is dyed pink with black streaks throughout. She has several pierced holes in her nares and ears and is wearing an earring through her eyebrow and heavy black makeup. Which is an appropriate conclusion for the nurse draw? a. She probably does not have any problems. b. She is only trying to shock people and that her dress should be ignored. c. She has a manic syndrome because of her abnormal dress and grooming. d. More information should be gathered to decide whether her dress is appropriate. 10. A patient has been in the intensive care unit for 10 days. He has just been moved to the medical-surgical unit, and the admitting nurse is planning to perform a mental status examination. What should the nurse expect during this patient’s tests of cognitive function? a. May display some disruption in thought content. b. Will state, “I am so relieved to be out of intensive care.” c. Will be oriented to place and person, but the patient may not be certain of the date. d. May show evidence of some clouding of his level of consciousness. 11. During a mental status examination, the nurse wants to assess a patient’s affect. Which question the nurse should ask? a. “How do you feel today?” b. “Would you please repeat the following words?” c. “Have these medications had any effect on your pain?” d. “Has this pain affected your ability to get dressed by yourself?” 12. The nurse is planning to assess new memory with a patient. Which is the best way for the nurse to do this? a. Administer the FACT test. b. Ask him to describe his first job. c. Give him the Four Unrelated Words Test. d. Ask him to describe what television show he was watching before coming to the clinic. 13. A 45-year-old woman is at the clinic for a mental status assessment. Which describes the expecting findings on the Four Unrelated Words Test? a. Invents four unrelated words within 5 minutes b. Invents four unrelated words within 30 seconds c. Recalls four unrelated words after a 30-minute delay d. Recalls four unrelated words after a 60-minute delay 14. During a mental status assessment, which question by the nurse would best assess a person’s judgment? a. “Do you feel that you are being watched, followed, or controlled?” b. “Tell me what you plan to do once you are discharged from the hospital.” c. “What does the statement, ‘People in glass houses shouldn’t throw stones,’ mean to you?” d. “What would you do if you found a stamped, addressed envelope lying on the sidewalk?” 15. Which of these individuals would the nurse consider at highest risk for a suicide attempt? a. Man who jokes about death b. Woman who, during a past episode of major depression, attempted suicide c. Adolescent who just broke up with her boyfriend and states that she would like to kill herself d. Older adult man who tells the nurse that he is going to “join his wife in heaven” tomorrow and plans to use a gun 16. The nurse is assessing orientation in a 79-year-old patient. Which of these responses would lead the nurse to conclude that this patient is oriented? a. “I know my name is John. I couldn’t tell you where I am. I think it is 2010, though.” b. “I know my name is John, but to tell you the truth, I get kind of confused about the date.” c. “I know my name is John; I guess I’m at the hospital in Spokane. No, I don’t know the date.” d. “I know my name is John. I am at the hospital in Spokane. I couldn’t tell you what date it is, but I know that it is February of a new year—2010.” 17. The nurse is performing the Denver II screening test on a 12-month-old infant during a routine well-child visit. What should the nurse tell the infant’s parents about the Denver II screening test? a. Tests three areas of development: cognitive, physical, and psychological b. Will indicate whether the child has a speech disorder so that treatment can begin c. Is a screening instrument designed to detect children who are slow in development d. Is a test to determine intellectual ability and may indicate whether problems will develop later in school 18. A patient drifts off to sleep when she is not being stimulated. The nurse can easily arouse her by calling her name, but the patient remains drowsy during the conversation. What is the best description of this patient’s level of consciousness? a. Lethargic b. Obtunded c. Stuporous d. Semi-coma 19. A patient has had a cerebrovascular accident (stroke). He is trying very hard to communicate. He seems driven to speak and says, “I buy obie get spirding and take my train.” What is the best description of this patient’s problem? a. Echolalia b. Global aphasia c. Broca’s aphasia d. Wernicke’s aphasia 20. A patient repeatedly seems to have difficulty coming up with a word. He says, “I was on my way to work, and when I got there, the thing that you step into that goes up in the air was so full that I decided to take the stairs.” How should the nurse record this on his chart? a. Blocking b. Neologism c. Circumlocution d. Circumstantiality 21. During an examination, the nurse notes that a patient is exhibiting flight of ideas. Which statement by the patient is an example of flight of ideas? a. “My stomach hurts. Hurts, spurts, burts.” b. “Kiss, wood, reading, ducks, onto, maybe.” c. “I wash my hands, wash them, wash them. I usually go to the sink and wash my hands.” d. “Take this pill? The pill is red. I see red. Red velvet is soft, soft as a baby’s bottom.” 22. A patient describes feeling an unreasonable, irrational fear of snakes. His fear is so persistent that he can no longer comfortably look at even pictures of snakes and has made an effort to identify all the places he might encounter a snake and avoids them. What is the best description of this patient’s condition? a. A snake phobia b. A hypochondriac c. An obsession with snakes d. A delusion that snakes are harmful stemming from an early traumatic incident involving snakes 23. During a recent interview, a patient diagnosed with schizophrenia shows the nurse a picture of a man holding a decapitated head. He describes this picture as horrifying but then laughs loudly at the content. What is the best description of this behavior? a. Confusion b. Ambivalence c. Depersonalization d. Inappropriate affect 24. During change of shift report, the nurse hears that a patient is experiencing hallucinations. Which is an example of a hallucination? a. Man believes that his dead wife is talking to him. b. Woman hears the doorbell ring and goes to answer it, but no one is there. c. Child sees a man standing in his closet. When the lights are turned on, it is only a dry cleaning bag. d. Man believes that the dog has curled up on the bed, but when he gets closer he sees that it is a blanket. 25. A 20-year-old construction worker has been brought into the emergency department with heat stroke. He has delirium as a result of a fluid and electrolyte imbalance. When conducting the mental status examination for this patient, what should the nurse assess first? a. Affect and mood b. Memory and affect c. Cognitive abilities d. Level of consciousness 26. A 26-year-old woman was robbed and beaten a month ago. She is returning to the clinic today for a follow-up assessment. The nurse will want to ask her which of these questions? a. “How are things going with the trial?” b. “How are things going with your job?” c. “Tell me about your recent engagement!” d. “Are you having any disturbing dreams?” 27. The nurse is performing a mental status examination. Which statement is true regarding the assessment of mental status? a. Mental status assessment diagnoses specific psychiatric disorders. b. Mental disorders occur in response to everyday life stressors. c. Mental status functioning is inferred through the assessment of an individual’s behaviors. d. Mental status can be directly assessed, similar to other systems of the body (e.g., heart sounds, breath sounds). 28. A 23-year-old patient in the clinic appears anxious. Her speech is rapid, and she is fidgety and in constant motion. Which of these questions or statements would be most appropriate for the nurse to use in this situation? a. “How do you usually feel? Is this normal behavior for you?” b. “I am going to say four words. In a few minutes, I will ask you to recall them.” c. “Describe the meaning of the phrase, ‘Looking through rose-colored glasses.’” d. “Pick up the pencil in your left hand, move it to your right hand, and place it on the table.” 29. The nurse is planning health teaching for a 65-year-old woman who has had a cerebrovascular accident (stroke) and has aphasia. Which of these questions is most important to use when assessing the mental status of this patient? a. “Please count backward from 100 by 7.” b. “I will name three items and ask you to repeat them in a few minutes.” c. “Please point to articles in the room and parts of the body as I name them.” d. “What would you do if you found a stamped, addressed envelope on the sidewalk?” 30. A 30-year-old female patient is describing feelings of hopelessness and depression. She has attempted self-mutilation and has a history of suicide attempts. She describes difficulty sleeping at night and has lost 10 pounds in the past month. Which of these statements or questions is the nurse’s best response in this situation? a. “Do you have a weapon?” b. “How do other people treat you?” c. “Are you feeling so hopeless that you feel like hurting yourself now?” d. “People often feel hopeless, but the feelings resolve within a few weeks.” 31. The nurse is providing instructions to newly hired graduates for the mini–mental state examination (MMSE). Which statement best describes this examination? a. Scores below 30 indicate cognitive impairment. b. The MMSE is a good tool to evaluate mood and thought processes. c. This examination is a good tool to detect delirium and dementia and to differentiate these from psychiatric mental illness. d. The MMSE is a useful tool for an initial evaluation of mental status. Additional tools are needed to evaluate cognition changes over time. 32. The nurse discovers speech problems in a patient during an assessment. The patient has spontaneous speech, but it is mostly absent or is reduced to a few stereotypical words or sounds. This finding reflects which type of aphasia? a. Global b. Broca’s c. Dysphonic d. Wernicke’s 33. A patient repeats, “I feel hot. Hot, cot, rot, tot, got. I’m a spot.” What term should the nurse use to document this? a. Blocking b. Clanging c. Echolalia d. Neologism 34. During an interview, the nurse notes that the patient gets up several times to wash her hands even though they are not dirty. This is an example of what behavior? a. Social phobia b. Compulsive disorder c. Generalized anxiety disorder d. Posttraumatic stress disorder 35. The nurse is administering a Mini-Cog test to an older adult woman. When asked to draw a clock showing the time of 10:45, the patient drew a clock with the numbers out of order and with an incorrect time. This result indicates which finding? a. Amnesia b. Delirium c. Cognitive impairment d. Attention-deficit disorder 36. During morning rounds, the nurse asks a patient, “How are you today?” The patient responds, “You today, you today, you today!” and mumbles the words. This is an example of which speech pattern? a. Echolalia b. Clanging c. Word salad d. Perseveration MULTIPLE RESPONSE 1. The nurse is assessing a patient who is admitted with possible delirium. Which of these are manifestations of delirium? (Select all that apply.) a. Person experiences agnosia. b. Person demonstrates apraxia. c. Develops over a short period d. Person exhibits memory impairment or deficits. e. Occurs as a result of a medical condition, such as systemic infection Chapter 06: Substance Use Assessment Jarvis: Physical Examination and Health Assessment, 8th Edition MULTIPLE CHOICE 1. A woman has come to the clinic to seek help with a substance-abuse problem. She admits to using cocaine just before arriving. Which of these assessment findings would the nurse expect to find when examining this woman? a. Dilated pupils, pacing, and psychomotor agitation b. Dilated pupils, unsteady gait, and aggressiveness c. Pupil constriction, lethargy, apathy, and dysphoria d. Constricted pupils, euphoria, and decreased temperature 2. The nurse is assessing a patient who has been admitted for cirrhosis of the liver, secondary to chronic alcohol use. Which assessment finding should the nurse expect? a. Hypertension b. Ventricular fibrillation c. Bradycardia d. Mitral valve prolapse 3. The nurse is conducting a class on alcohol and the effects of alcohol on the body. What information about the number of standard drinks (each containing 14 grams of alcohol) associated with a 32% increase in breast cancer should the nurse include in the class? a.  2.1/day b.  4.2/day c.  8/week d.  15/week 4. During a session on substance abuse, the nurse is reviewing statistics with the class. For people aged 12 years and older, which illicit substance was most commonly used? a. Heroin b. Marijuana c. Crack cocaine d. Hallucinogens 5. A woman who has just discovered that she is pregnant is in the clinic for her first obstetric visit. She asks the nurse, “How many drinks a day is safe for my baby?” What is the best response by the nurse? a. “You should limit your drinking to once or twice a week.” b. “It’s okay to have up to two glasses of wine a day.” c. “As long as you avoid getting drunk, you should be safe.” d. “No amount of alcohol has been determined to be safe during pregnancy.” 6. During an assessment, the nurse asks a female patient, “How many alcoholic drinks do you have a week?” Which answer by the patient would indicate at-risk drinking? a. “I may have one or two drinks a week.” b. “I usually have three or four drinks a week.” c. “I’ll have a glass or two of wine every now and then.” d. “I have eight to ten drinks a week, but I never get drunk.” 7. The nurse is asking an adolescent about illicit substance abuse. The adolescent answers, “Yes, I’ve used marijuana at parties with my friends.” What is the next question the nurse should ask? a. “Who are these friends?” b. “Is this a regular habit?” c. “Do your parents know about this?” d. “When was the last time you used marijuana?” 8. The nurse has completed an assessment on a patient who came to the clinic for a leg injury. As a result of the assessment, the nurse has determined that the patient has at-risk alcohol use. Which action by the nurse is most appropriate at this time? a. Record the results of the assessment, and notify the physician on call. b. State, “You are drinking more than is medically safe. I strongly recommend that you quit drinking, and I’m willing to help you.” c. State, “It appears that you may have a drinking problem. Here is the telephone number of our local Alcoholics Anonymous chapter.” d. Give the patient information about a local rehabilitation clinic. 9. A patient is brought to the emergency department. He is restless, has dilated pupils, is sweating, has a runny nose and tearing eyes, and complains of muscle and joint pains. His girlfriend thought he had influenza, but she became concerned when his temperature went up to 39.4 C. She admits that he has been a heavy drug user, but he has been trying to stop on his own. The nurse suspects that the patient is experiencing withdrawal symptoms from which substance? a. Alcohol b. Heroin c. Cocaine d. Sedatives 10. The nurse is reviewing aspects of substance abuse in preparation for a seminar. Which of these statements illustrates the concept of tolerance to an illicit substance? a. A physiologic dependence on a substance. b. A person requires an increased amount of the substance to produce the same effect. c. A person requires daily use of the substance to function and is unable to stop using it. d. A person experiences a syndrome of physiologic symptoms if the substance is not used. 11. The nurse is admitting a patient with an addiction to alcohol to a treatment center. Which screening tool should the nurse use to assess for symptoms of alcohol withdrawal? a. TWEAK b. AUDIT c. SMAST-G d. CIWA-Ar 12. The nurse is conducting an interview with an adult male patient. Which statement made by the patient indicates an alcohol use disorder? a. “I crave alcohol but have successfully cut down on my alcohol consumption.” b. “I’ve been late to work a few times so now I limit myself to 2 drinks/day and stick to it.” c. “I usually stay out longer and drink more than I intended but I still make it into work on time.” d. “I have a strong urge to drink and I’ve tried to stop drinking several times but it doesn’t last long.’ MULTIPLE RESPONSE 1. When reviewing the use of alcohol by older adults, the nurse notes that older adults have several characteristics that can increase the risk for alcohol use. Which would increase the bioavailability of alcohol in the blood for longer periods in the older adult? (Select all that apply.) a. Increased muscle mass b. Decreased blood pressure c. Increased cardiac output d. Decreased liver metabolism e. Decreased kidney functioning 2. A patient with a known history of heavy alcohol use has been admitted to the ICU after he was found unconscious outside a bar. The nurse closely monitors him for symptoms of withdrawal. Which of these symptoms may occur during this time? (Select all that apply.) a. Bradycardia b. Coarse tremor of the hands c. Transient hallucinations d. Somnolence e. Sweating 3. A patient visits the clinic to ask about smoking cessation. He has smoked heavily for 30 years and wants to stop “cold turkey.” He asks the nurse, “What symptoms can I expect if I do this?” Which of these symptoms should the nurse share with the patient as possible symptoms of nicotine withdrawal? (Select all that apply.) a. Headaches b. Hunger c. Sleepiness d. Restlessness e. Nervousness f. Sweating Chapter 07: Domestic and Family Violence Assessment Jarvis: Physical Examination and Health Assessment, 8th Edition MULTIPLE CHOICE 1. As a mandatory reporter of older adult abuse, which must be present before a nurse should notify the authorities? a. Statements from victim b. Statements from witnesses c. Proof of abuse and/or neglect d. Suspicion of older adult abuse and/or neglect 2. During a home visit, the nurse notices that an older adult woman is caring for her bedridden husband. The woman states that this is her duty, she does the best she can, and her children come to help when they are in town. Her husband is unable to care for himself, and she appears thin, weak, and exhausted. The nurse notices that several of his prescription medication bottles are empty. What term best describes this situation? a. Physical abuse b. Financial neglect c. Psychological abuse d. Unintentional physical neglect 3. The nurse is caring for a 17-year-old female patient. In which situation should the nurse screen the patient for intimate partner violence (IPV)? a. When intimate partner violence is suspected b. When a history of abuse in the family is known c. As a routine part of each health care encounter d. As part of the exam for a female with an unexplained injury 4. Which term refers to a wound produced by the tearing or splitting of body tissue, usually from blunt impact over a bony surface? a. Hematoma b. Abrasion c. Contusion d. Laceration 5. During an examination, the nurse notices a patterned injury on a patient’s back. What would cause such an injury? a. Blunt force b. Friction abrasion c. Stabbing from a kitchen knife d. Whipping from an extension cord 6. What should the nurse include when documenting IPV and older adult abuse? a. Photographic documentation of the injuries b. Summary of the abused patient’s statements c. General description of injuries in the progress notes d. Verbatim documentation of every statement made by the victim 7. A female patient has denied any abuse when answering the questions on an abuse assessment screening tool, but what finding by the nurse during the interview process is associated with IPV? a. Asthma b. Confusion c. Depression d. Frequent colds 8. The nurse is assessing bruising on an injured patient. Which color indicates a new bruise that is less than 2 hours old? a. Red b. Purple-blue c. Greenish-brown d. Brownish-yellow 9. The nurse suspects abuse when a 10-year-old child is taken to the urgent care center for a leg injury. Which is the best way for the nurse to document the findings in the patient’s chart? a. Rely on photographs of the injuries. b. Document what the child’s caregiver tells the nurse. c. Record what the nurse observes during the conversation. d. Use the words the child has said to describe how the injury occurred. 10. During an interview, a woman has answered “yes” to two of the Slapped, Threatened, and Throw (STaT) questions. What should the nurse say next? a. “So you were abused?” b. “Do you know what caused this abuse?” c. “I need to report this abuse to the authorities.” d. “Tell me about the abuse in your relationship.” 11. The nurse is examining a 3-year-old child who was brought to the emergency department after a fall. Which bruise, if found, would be of most concern? a. Bruises on the knee b. Bruises on the elbow c. Bruises on the abdomen d. Bruises on both of the shins 12. The nurse is caring for an 8-year-old child who has several bruises of varying colors (some red, some bluish-green, and some brownish-yellow) the size of a hand on the buttocks. What action should the nurse take next? a. Notify the child’s caregivers of the findings. b. Document that the bruises appear to be caused by spanking. c. When the child is alone, ask “How did you get these sore areas on your butt?” d. Inform the child “You can tell me who did this to you and we will not allow them to see or hurt you again.” 13. The nurse is caring for several patients on a pediatric unit. Which patient should the nurse be most concerned about possible abuse? a. A 4-month-old with bruises on the arms b. A 2-year-old with bruises on the knees c. An 8-year-old with a broken right arm d. A 15-year-old football player with a broken leg 14. The nurse is caring for several patients. Which patient is at highest risk for Intimate Partner Violence (IPV)? a. An Asian female who speaks no English b. A female multi-racial illegal immigrant c. A non-Hispanic white female living in poverty d. A female American Indian living above the poverty line MULTIPLE RESPONSE 1. The nurse assesses an older woman and suspects physical abuse. Which questions are appropriate for screening for abuse? (Select all that apply.) a. “Has anyone made you afraid, touched you in ways that you did not want, or hurt you physically?” b. “Are you being abused?” c. “Have you relied on people for any of the following: bathing, dressing, shopping, banking, or meals?” d. “Have you been upset because someone talked to you in a way that made you feel shamed or threatened?” e. “Has anyone tried to force you to sign papers or to use your money against your will?” Chapter 08: Assessment Techniques and Safety in the Clinical Setting Jarvis: Physical Examination and Health Assessment, 8th Edition MULTIPLE CHOICE 1. When performing a physical assessment, what technique should the nurse always perform first? a. Palpation b. Inspection c. Percussion d. Auscultation 2. The nurse is preparing to perform a physical assessment. Which statement is true about the inspection phase of the physical assessment? a. Usually yields little information b. Takes time and reveals a surprising amount of information c. May be somewhat uncomfortable for the expert practitioner d. Requires a quick glance at the patient’s body systems before proceeding with palpation 3. The nurse is assessing a patient’s skin during an office visit. What part of the hand and technique should be used to best assess the patient’s skin temperature? a. Fingertips b. Dorsal surface of the hand c. Ulnar portion of the hand d. Palmar surface of the hand 4. Which of these techniques uses the sense of touch to assess texture, temperature, moisture, and swelling when the nurse is assessing a patient? a. Palpation b. Inspection c. Percussion d. Auscultation 5. The nurse is preparing to assess a patient’s abdomen by palpation. How should the nurse proceed? a. Avoid palpation of reportedly “tender” areas because palpation in these areas may cause pain. b. Palpate a tender area quickly to avoid any discomfort that the patient may experience. c. Start the assessment with deep palpation, while encouraging the patient to relax and take deep breaths. d. Begin the assessment with light palpation to detect surface characteristics and to accustom the patient to being touched. 6. The nurse would use bimanual palpation technique in which situation? a. Palpating the thorax of an infant b. Palpating the kidneys and uterus c. Assessing pulsations and vibrations d. Assessing the presence of tenderness and pain 7. The nurse is preparing to percuss the abdomen of a patient. What characteristic of the underlying tissue does percussion assess? a. Turgor b. Texture c. Density d. Consistency 8. The nurse is reviewing percussion techniques with a new graduate nurse. Which action performed by the graduate nurse while percussing requires the nurse to intervene? a. Percussing once over each area b. Striking with the fingertip, not the finger pad c. Using the wrist to make the strikes, not the arm d. Quickly lifting the striking finger after each stroke 9. While percussing over the liver of a patient, the nurse notices a dull sound. What should the nurse do? a. Consider this a normal finding. b. Palpate this area for an underlying mass. c. Reposition the hands, and attempt to percuss in this area again. d. Consider this finding as abnormal, and refer the patient for additional treatment. 10. The nurse is unable to identify any changes in sound when percussing over the abdomen of an obese patient. What should the nurse do next? a. Ask the patient to take deep breaths to relax the abdominal musculature. b. Consider this finding as normal, and proceed with the abdominal assessment. c. Increase the amount of strength used when attempting to percuss over the abdomen. d. Decrease the amount of strength used when attempting to percuss over the abdomen. 11. The nurse hears bilateral loud, long, and low tones when percussing over the lungs of a 4-year-old child. How should the nurse proceed? a. Palpate over the area for increased pain and tenderness. b. Ask the child to take shallow breaths, and percuss over the area again. c. Refer the child to a specialist because of an increased amount of air in the lungs. d. Consider this finding as normal for a child this age, and proceed with the examination. 12. A patient has suddenly developed shortness of breath and appears to be in significant respiratory distress. After calling the physician and placing the patient on oxygen, which of these actions is the best for the nurse to take when further assessing the patient? a. Count the patient’s respirations. b. Bilaterally percuss the thorax, noting any differences in percussion tones. c. Call for a chest x-ray, and wait for the results before beginning an assessment. d. Inspect the thorax for any new masses and bleeding associated with respirations. 13. The nurse is teaching a class on basic assessment skills. Which of these statements is true regarding the stethoscope and its use? a. Slope of the earpieces should point posteriorly (toward the occiput). b. It blocks out extraneous room noise but does not magnify sound. c. The tubing length should be 22 inches to dampen the distortion of sound. d. Fit and quality of the stethoscope are not as important as its ability to magnify sound. 14. The nurse is preparing to use a stethoscope for auscultation. Which statement is true regarding the diaphragm of the stethoscope? a. Used to listen for high-pitched sounds b. Used to listen for low-pitched sounds c. Should be lightly held against the person’s skin to block out low-pitched sounds d. Should be lightly held against the person’s skin to listen for extra heart sounds and murmurs 15. The nurse is preparing to auscultate the abdomen. How should the nurse proceed? a. Warm the endpiece of the stethoscope by placing it in warm water. b. Leave the gown on the patient to ensure that he or she does not get chilled during the examination. c. Ensure that the bell side of the stethoscope is turned to the “on” position. d. Check the temperature of the room and offer blankets to the patient if he or she feels cold. 16. The nurse will use which technique of assessment to determine the presence of crepitus, swelling, and pulsations? a. Palpation b. Inspection c. Percussion d. Auscultation 17. The nurse is preparing to use an otoscope for an examination. Which statement is true regarding the otoscope? a. Often used to direct light onto the sinuses b. Used to examine the structures of the internal ear c. Uses a short, broad speculum to help visualize the ear d. Directs light into the ear canal and onto the tympanic membrane 18. An examiner is using an ophthalmoscope to examine a patient’s eyes. The patient has astigmatism and is nearsighted. Which of these techniques by the examiner would indicate that the examination is being correctly performed? a. Rotating the lens selector dial to bring the object into focus b. Using the large full circle of light when assessing pupils that are not dilated c. Rotating the lens selector dial to the black numbers to compensate for astigmatism d. Using the grid on the lens aperture dial to visualize the external structures of the eye 19. The nurse is unable to palpate the right radial pulse on a patient. What should the nurse do next? a. Auscultate over the area with a fetoscope. b. Use a goniometer to measure the pulsations. c. Use a Doppler device to check for pulsations over the area. d. Check for the presence of pulsations with a stethoscope. 20. The nurse is preparing to perform a physical assessment. The correct action by the nurse is reflected by which statement? a. Performs the examination from the left side of the bed b. Examines tender or painful areas first to help relieve the patient’s anxiety c. Follows the same examination sequence, regardless of the patient’s age or condition d. Organizes the assessment to ensure that the patient does not change positions too often 21. An adult male is at the clinic for a physical examination. He states that he is “very anxious” about the physical examination. What steps can the nurse take to make him more comfortable? a. Appear unhurried and confident when examining him. b. Stay in the room when he undresses in case he needs assistance. c. Ask him to change into an examining gown and to take off his undergarments. d. Defer measuring vital signs until the end of the examination, which allows him time to become comfortable. 22. When performing a physical examination, safety must be considered to protect the examiner and the patient against the spread of infection. Which of these statements describes the most appropriate action the nurse should take when performing a physical examination? a. Washing one’s hands after removing gloves is not necessary, as long as the gloves are still intact. b. Hands are washed before and after every physical patient encounter. c. Hands are washed before the examination of each body system to prevent the spread of bacteria from one part of the body to another. d. Gloves are worn throughout the entire examination to demonstrate to the patient concern regarding the spread of infectious diseases. 23. The nurse is examining a patient’s lower leg and notices a draining ulceration. Which of these actions is most appropriate in this situation? a. Wash hands and then contact the physician. b. Continue to examine the ulceration and then wash hands. c. Wash hands, put on gloves, and continue with the examination of the ulceration. d. Wash hands, proceed with rest of the physical examination, and perform the examination of the leg ulceration last. 24. During the examination, offering some brief teaching about the patient’s body or the examiner’s findings is often appropriate. Which one of these statements by the nurse is most appropriate? a. “Your atrial dysrhythmias are under control.” b. “You have pitting edema and mild varicosities.” c. “Your pulse is 80 beats per minute, which is within the normal range.” d. “I’m using my stethoscope to listen for any crackles, wheezes, or rubs in your lungs.” 25. While performing the physical examination, the nurse shares information and briefly teaches the patient. Why does the nurse do this? a. To help the examiner feel more comfortable and gain control of the situation b. To build rapport and increase the patient’s confidence in the examiner c. To assist the patient in understanding his or her disease process and treatment modalities d. To aid the patient to identify questions about his or her disease and the potential areas of needed education 26. The nurse is preparing to examine an infant. At what point in the examination should the nurse attempt to elicit the Moro reflex? a. When the infant is sleeping b. At the end of the examination c. Before auscultation of the thorax d. At about the middle of the examination 27. Which should the nurse do when preparing to perform a physical examination on an infant? a. Have the parent remove all clothing except the diaper on a boy. b. Instruct the parent to feed the infant immediately before the examination. c. Encourage the infant to suck on a pacifier during abdominal auscultation. d. Ask the parent to leave the room briefly when assessing the infant’s vital signs. 28. A 6-month-old infant has been brought to the well-child clinic for a checkup. She is currently sleeping. What should the nurse do first when beginning the examination? a. Wake the infant before beginning the examination. b. Examine the infant’s hips before the infant wakes up. c. Auscultate the lungs and heart while the infant is still sleeping. d. Begin with the assessment of the eye and continue with the remainder of the examination in a head-to-toe approach. 29. A 2-year-old child has been brought to the clinic for a well-child checkup. What is the best way for the nurse to begin the assessment? a. Ask the parent to place the child on the examining table. b. Have the parent remove all of the child’s clothing before the examination. c. Allow the child to keep a security object such as a toy or blanket during the examination. d. Initially focus the interactions on the child, essentially ignoring the parent until the child’s trust has been obtained. 30. The nurse is examining a 2-year-old child and asks, “May I listen to your heart now?” Which critique of the nurse’s technique is most accurate? a. Asking questions enhances the child’s autonomy. b. Asking the child for permission helps develop a sense of trust. c. This question is an appropriate statement because children at this age like to have choices. d. Children at this age like to say, “No.” The examiner should not offer a choice when no choice is available. 31. The nurse is preparing to examine a 4-year-old child. Which action by the nurse is appropriate for this age group? a. Explain the procedures in detail to alleviate the child’s anxiety. b. Give the child feedback and reassurance during the examination. c. Do not ask the child to remove his or her clothes because children at this age are usually very private. d. Perform an examination of the ear, nose, and throat first, and then examine the thorax and abdomen. 32. What action by the nurse is appropriate when examining a 16-year-old male teenager? a. Discuss health teaching with the parent because the teen is unlikely to be interested in promoting wellness. b. Ask his parent to stay in the room during the history and physical examination to answer any questions and to alleviate his anxiety. c. Talk to him in the same manner as one would talk to a younger child because a teen’s level of understanding may not match his or her speech. d. Provide feedback that his body is developing normally, and discuss the wide variation among teenagers on the rate of growth and development. 33. When examining an older adult, the nurse should use which technique? a. Avoid touching the patient too much. b. Attempt to perform the entire physical examination during one visit. c. Speak loudly and slowly because most aging adults have hearing deficits. d. Arrange the sequence of the examination to allow as few position changes as possible. 34. What is the most important step that the nurse can take to prevent the transmission of microorganisms in the hospital setting? a. Wear protective eye wear at all times. b. Wear gloves whenever in direct contact with patients. c. Wash hands before and after contact with each patient. d. Clean the stethoscope with an alcohol swab between patients. 35. Which of these statements is true regarding the use of Standard Precautions in the health care setting? a. Standard Precautions apply to all body fluids, including sweat. b. Alcohol-based hand rub should be used if hands are visibly dirty. c. Standard Precautions are intended for use with all patients, regardless of their risk or presumed infection status. d. Standard Precautions are to be used only when nonintact skin, excretions containing visible blood, or expected contact with mucous membranes is present. 36. The nurse is preparing to assess a hospitalized patient who is experiencing significant shortness of breath. How should the nurse proceed with the assessment? a. Have the patient lie down to obtain an accurate cardiac, respiratory, and abdominal assessment. b. Obtain a thorough history and physical assessment from the patient’s family member. c. Immediately perform a complete history and physical assessment to obtain baseline information. d. Examine the body areas appropriate to the problem and perform the rest of the complete assessment after the problem has resolved. 37. When examining an infant, the nurse should examine which area first? a. Ear b. Nose c. Throat d. Abdomen 38. While auscultating heart sounds, the nurse hears a murmur. Which of these instruments should be used to assess this murmur? a. Electrocardiogram b. Bell of the stethoscope c. Diaphragm of the stethoscope d. Palpation with the nurse’s palm of the hand 39. During an examination of a patient’s abdomen, the nurse notes that the abdomen is rounded and firm to the touch. During percussion, the nurse notes a drum-like quality of the sounds across the quadrants. How should the nurse interpret this type of sound? a. Constipation b. Air-filled areas c. Presence of a tumor d. Presence of dense organs 40. The nurse is preparing to examine a 6-year-old child. Which action is most appropriate? a. The child is asked to undress from the waist up. b. The thorax, abdomen, and genitalia are examined before the head. c. The nurse should keep in mind that a child at this age will have a sense of modesty. d. Talking about the equipment being used is avoided because doing so may increase the child’s anxiety. 41. During auscultation of a patient’s heart sounds, the nurse hears an unfamiliar sound. Which action should the nurse take? a. Ask the patient how he or she is feeling. b. Document the findings in the patient’s record. c. Wait 10 minutes, and auscultate the sound again. d. Ask another nurse to double check the finding. MULTIPLE RESPONSE 1. The nurse is preparing to palpate the thorax and abdomen of a patient. Which of these statements describes the correct technique for this procedure? (Select all that apply.) a. Identify any tender areas and palpate them last. b. Warm the hands first before touching the patient. c. Use the palms of the hands to assess temperature of the skin. d. Start with light palpation to detect surface characteristics. e. For deep palpation, use one long continuous palpation when assessing the liver. f. Use the fingertips to examine skin texture, swelling, pulsation, and presence of lumps. Chapter 09: General Survey and Measurement Jarvis: Physical Examination and Health Assessment, 8th Edition MULTIPLE CHOICE 1. The nurse is performing a general survey. Which action is a component of the general survey? a. Observing the patient’s body stature and nutritional status b. Interpreting the subjective information the patient has reported c. Measuring the patient’s temperature, pulse, respirations, and blood pressure d. Observing specific body systems while performing the physical assessment 2. Which of these guidelines should a nurse follow when measuring a patient’s weight? a. The patient is always weighed wearing only his or her undergarments. b. The type of scale does not matter, as long as the weights are similar from day to day. c. The patient may leave on his or her jacket and shoes as long as these are documented next to the weight. d. Attempts should be made to weigh the patient at approximately the same time of day, if a sequence of weights is necessary. 3. When performing an examination, the nurse should consider a child’s physical growth to be the best indicator of which aspect of health? a. General health b. Genetic makeup c. Nutritional status d. Activity and exercise patterns 4. A 1-month-old infant has a head measurement of 34 cm and has a chest circumference of 32 cm. Based on the interpretation of these findings, what action should the nurse take? a. Refer the infant to a physician for further evaluation. b. Consider these findings normal for a 1-month-old infant. c. Expect the chest circumference to be greater than the head circumference. d. Ask the parent to return in 2 weeks to re-evaluate the head and chest circumferences. 5. The nurse is assessing an 80-year-old male patient. Which assessment findings would be considered normal? a. Increase in body weight from his younger years b. Additional deposits of fat in the cheeks and forearms c. Presence of kyphosis and flexion in bilateral knees and hips d. Change in overall body proportion, including a longer trunk and shorter extremities 6. The nurse is preparing to measure the length, weight, chest, and head circumference of a 6-month-old infant. Which measurement technique is correct? a. Measuring the infant’s length by using a tape measure b. Weighing the infant by placing him or her on an electronic standing scale c. Measuring the chest circumference at the nipple line with a tape measure d. Measuring the head circumference by wrapping the tape measure over the nose and cheekbones 7. A 60-year-old male patient has been treated for pneumonia for the past 6 weeks. He is seen today in the clinic for an “unexplained” weight loss of 10 pounds over the last 6 weeks. Which is an appropriate rationale for this patient’s weight loss? a. Chronic diseases such as hypertension cause weight loss. b. Weight loss is probably the result of unhealthy eating habits. c. Unexplained weight loss often accompanies short-term illnesses. d. Weight loss is probably the result of a mental health dysfunction. 8. When assessing a 75-year-old patient who has asthma, the nurse notes that he assumes a tripod position, leaning forward with arms braced on the chair. How should the nurse interpret these findings? a. Assume that the patient is eager and interested in participating in the interview. b. Evaluate the patient for abdominal pain, which may be exacerbated in the sitting position. c. Assume that the patient is having difficulty breathing and assist him to a supine position. d. Recognize that a tripod position is often used when a patient is having respiratory difficulties. 9. In a patient with acromegaly, which assessment finding will the nurse expect to find? a. Sternal deformity and hyperextensible joints b. Growth retardation and a delayed onset of puberty c. Overgrowth of bone in the face, head, hands, and feet d. Increased height and weight and delayed sexual development 10. The nurse is performing a general survey of a patient. Which finding is considered normal? a. Body mass index (BMI) of 20 b. When standing, the patient’s base is narrow. c. The patient appears older than his stated age. d. Arm span (fingertip to fingertip) is greater than the height. 11. Which of these specific measurements is the best index of a child’s general health? a. Body mass index b. Height and weight c. Head circumference d. Chest circumference 12. The nurse is assessing an 8-year-old child whose growth rate measures below the third percentile for a child his age. He appears significantly younger than his stated age and is chubby with infantile facial features. Which condition does this child likely have? a. Acromegaly b. Marfan syndrome c. Hypopituitary dwarfism d. Achondroplastic dwarfism 13. During an examination, the nurse notices that a female patient has a round “moon” face, central trunk obesity, and a cervical hump. Her skin is fragile with bruises. The nurse determines that the patient likely has which condition? a. Gigantism b. Acromegaly c. Cushing syndrome d. Marfan syndrome Chapter 10: Vital Signs Jarvis: Physical Examination and Health Assessment, 8th Edition MULTIPLE CHOICE 1. The nurse should measure rectal temperatures in which of these patients? a. Older adult b. Comatose adult c. School-age child d. Patient receiving oxygen by nasal cannula 2. The nurse is teaching a student nurse about the different types of thermometers. When teaching the student about the advantages of the tympanic membrane thermometer (TMT), which statement should the nurse include? a. “Measuring temperature using the TMT is inexpensive.” b. “The rapid measurement of the TMT is useful for uncooperative younger children.” c. “Using the TMT is the most accurate method for measuring body temperature in newborn infants.” d. “Studies strongly support the use of the TMT in children under the age 6 years.” 3. When assessing an older adult, the nurse should recognize that which vital sign changes occur with aging? a. Increase in pulse rate b. Widened pulse pressure c. Increase in body temperature d. Decrease in diastolic blood pressure . 4. The nurse is examining a patient who is complaining of “feeling cold.” Which is a mechanism of heat loss in the body? a. Exercise b. Radiation c. Metabolism d. Food digestion 5. When measuring a patient’s body temperature, the nurse should keep in mind that what can influence the temperature? a. Constipation b. Diurnal cycle c. Nocturnal cycle d. Patient’s emotional state 6. When evaluating the temperature of older adults, the nurse should remember which aspect about an older adult’s body temperature? a. The body temperature of the older adult is lower than that of a younger adult. b. An older adult’s body temperature is approximately the same as that of a young child. c. Body temperature depends on the type of thermometer used. d. In the older adult, the body temperature varies widely because of less effective heat control mechanisms. 7. Which of these actions illustrates the correct technique the nurse should use when assessing oral temperature with a glass thermometer? a. Wait 30 minutes if the patient has ingested hot or iced liquids. b. Leave the thermometer in place for 3 to 4 minutes if the patient is afebrile. c. Shake the glass thermometer down to 37.5 C before taking the patient’s temperature. d. Place the thermometer in front of the tongue and ask the patient to close his or her lips. 8. What technique should the nurse use to accurately assess a rectal temperature in an adult? a. Use a lubricated blunt tip thermometer. b. Insert the thermometer 2 to 3 inches into the rectum. c. Leave the thermometer in place up to 8 minutes if the patient is febrile. d. Wait 2 to 3 minutes if the patient has recently smoked a cigarette. 9. Which technique is correct when the nurse is assessing the radial pulse of a patient? a. Palpate for 1 minute, if the rhythm is irregular. b. Palpate for 15 seconds and multiply by 4, if the rhythm is regular. c. Palpate for 2 full minutes to detect any variation in amplitude. d. Palpate for 10 seconds and multiply by 6, if the rhythm is regular and the patient has no history of cardiac abnormalities. 10. When assessing a patient’s pulse, the nurse should also notice which of these characteristics? a. Force b. Pallor c. Capillary refill time d. Timing in the cardiac cycle 11. When assessing the pulse of a 6-year-old boy, the nurse notices that his heart rate varies with his respiratory cycle, speeding up at the peak of inspiration and slowing to normal with expiration. What action should the nurse take next? a. Notify the physician. b. Record this finding as normal. c. Check the child’s blood pressure and note any variation with respiration. d. Document that this child has bradycardia and continue with the assessment. 12. When assessing the force, or strength, of a pulse, what should the nurse recall about the pulse? a. Is a reflection of the heart’s stroke volume b. Typically recorded on a 0- to 2-point scale c. Demonstrates elasticity of the blood vessel wall d. Reflects the blood volume in the arteries during diastole 13. The nurse is assessing the vital signs of a 20-year-old male marathon runner and documents the following vital signs: temperature–36 C; pulse–48 beats per minute; respirations–14 breaths per minute; blood pressure–104/68 mm Hg. Which statement is true concerning these results? a. The patient is experiencing tachycardia. b. These are normal vital signs for a healthy, athletic adult. c. The patient’s pulse rate is not normal—his physician should be notified. d. On the basis of these readings, the patient should return to the clinic in 1 week. 14. The nurse is assessing the vital signs of a 3-year-old patient who appears to have an irregular respiratory pattern. How should the nurse assess this child’s respirations? a. Respirations should be counted for 1 full minute if the nurse suspects an abnormality. b. Child’s pulse and respirations should be simultaneously checked for 30 seconds and then multiplied by 2. c. Child’s respirations should be checked for a minimum of 5 minutes to identify any variations in his or her respiratory pattern. d. Patient’s respirations should be counted for 15 seconds and then multiplied by 4 to obtain the number of respirations per minute. 15. A patient’s blood pressure is 118/82 mm Hg. He asks the nurse, “What do the numbers mean?” Which is the best reply by the nurse? a. “The numbers are within the normal range and are nothing to worry about.” b. “The bottom number is the diastolic pressure and reflects the stroke volume of the heart.” c. “The top number is the systolic blood pressure and reflects the pressure of the blood against the arteries when the heart contracts.” d. “The concept of blood pressure is difficult to understand. The primary thing to be concerned about is the top number, or the systolic blood pressure.” 16. While measuring a patient’s blood pressure, the nurse should recall that which is a factor that influences a patient’s blood pressure? a. Pulse rate b. Pulse pressure c. Vascular output d. Peripheral vascular resistance 17. A nurse is helping at a health fair at a local mall. What should the nurse keep in mind when taking blood pressures on a variety of people? a. After menopause, blood pressure readings in women are usually lower than those taken in men. b. The blood pressure of an African-American adult is usually higher than that of a non-Hispanic White adult of the same age. c. Blood pressure measurements in people who are overweight should be the same as those of people who are at a normal weight. d. A teenager’s blood pressure reading will be lower than that of an adult. 18. The nurse notices that a colleague is preparing to check the blood pressure of a patient who is obese by using a standard-sized blood pressure cuff. How would this likely affect the blood pressure reading? a. Yield a falsely low blood pressure b. Yield a falsely high blood pressure c. Be the same, regardless of cuff size d. Vary as a result of the technique of the person performing the assessment 19. A student is late for his appointment and has rushed across campus to the health clinic. How should the nurse proceed? a. Allow 5 minutes for him to relax and rest before checking his vital signs. b. Check the blood pressure in both arms, expecting a difference in the readings because of his recent exercise. c. Immediately monitor his vital signs on his arrival at the clinic and then 5 minutes later, recording any differences. d. Check his blood pressure in the supine position, which will provide a more accurate reading and will allow him to relax at the same time. 20. The nurse will perform a palpated pressure before auscultating blood pressure. What is the reason for this? a. More clearly hear the Korotkoff sounds. b. Detect the presence of an auscultatory gap. c. Avoid missing a falsely elevated blood pressure. d. More readily identify phase IV of the Korotkoff sounds. 21. The nurse is taking an initial blood pressure reading on a 72-year-old patient with documented hypertension. How should the nurse proceed? a. Cuff should be placed on the patient’s arm and inflated 30 mm Hg above the patient’s pulse rate. b. Cuff should be inflated to 200 mm Hg in an attempt to obtain the most accurate systolic reading. c. Cuff should be inflated 30 mm Hg above the point at which the palpated pulse disappears. d. After confirming the patient’s previous blood pressure readings, the cuff should be inflated 30 mm Hg above the highest systolic reading recorded. 22. The nurse has collected the following information on a patient: palpated blood pressure–180 mm Hg; auscultated blood pressure–170/100 mm Hg; apical pulse–60 beats per minute; radial pulse–70 beats per minute. What is the patient’s pulse pressure? a. 10 b. 70 c. 80 d. 100 23. When auscultating the blood pressure of a 25-year-old patient, the nurse notices that the phase I Korotkoff sounds begin at 200 mm Hg. At 100 mm Hg, the Korotkoff sounds muffle. At 92 mm Hg, the Korotkoff sounds disappear. How should the nurse record this patient’s blood pressure? a. 200/92 b. 200/100 c. 100/200/92 d. 200/100/92 24. A patient is seen in the clinic for complaints of “fainting episodes that started last week.” How should the nurse proceed with the examination? a. Blood pressure readings are taken in both the arms and the thighs. b. The patient is assisted to a lying position, and his blood pressure is taken. c. His blood pressure is recorded in the lying, sitting, and standing positions. d. His blood pressure is recorded in the lying and sitting positions; these numbers are then averaged to obtain a mean blood pressure. 25. A 70-year-old man has a blood pressure of 150/90 mm Hg in a lying position, 130/80 mm Hg in a sitting position, and 100/60 mm Hg in a standing position. How should the nurse evaluate these findings? a. These readings are a normal response and attributable to changes in the patient’s position. b. The change in blood pressure readings is called orthostatic hypotension. c. The blood pressure reading in the lying position is within normal limits. d. The change in blood pressure readings is considered within normal limits for the patient’s age. 26. The nurse is helping another nurse take a blood pressure reading on a patient’s thigh. Which action is correct regarding thigh pressure? a. Either the popliteal or femoral vessels should be auscultated to obtain a thigh pressure. b. The best position to measure thigh pressure is the supine position with the knee slightly bent. c. If the blood pressure in the arm is high in an adolescent, then it should be compared with the thigh pressure. d. The thigh pressure is lower than the pressure in the arm, which is attributable to the distance away from the heart and the size of the popliteal vessels. 27. The nurse is preparing to measure the vital signs of a 6-month-old infant. Which action by the nurse is correct? a. Respirations are measured; then pulse and temperature. b. Vital signs should be measured more frequently than in an adult. c. Procedures are explained to the parent, and the infant is encouraged to handle the equipment. d. The nurse should first perform the physical examination to allow the infant to become more familiar with her and then measure the infant’s vital signs. 28. A 4-month-old child is at the clinic for a well-baby checkup and immunizations. Which of these actions is most appropriate when the nurse is assessing an infant’s vital signs? a. The infant’s radial pulse should be palpated, and the nurse should notice any fluctuations resulting from activity or exercise. b. The nurse should auscultate an apical rate for 1 minute and then assess for any normal irregularities, such as sinus dysrhythmia. c. The infant’s blood pressure should be assessed by using a stethoscope with a large diaphragm piece to hear the soft muffled Korotkoff sounds. d. The infant’s chest should be observed and the respiratory rate counted for 1 minute; the respiratory pattern may vary significantly. 29. The nurse is conducting a health fair for older adults. Which statement is true regarding vital sign measurements in aging adults? a. The pulse is more difficult to palpate because of the stiffness of the blood vessels. b. An increased respiratory rate and a shallower inspiratory phase are expected findings. c. A decreased pulse pressure occurs from changes in the systolic and diastolic blood pressures. d. Changes in the body’s temperature regulatory mechanism leave the older person more likely to develop a fever. 30. The nurse is assessing children in a pediatric clinic. Which statement is true regarding the measurement of blood pressure in children? a. Blood pressure guidelines for children are based on age. b. Phase II Korotkoff sounds are the best indicator of systolic blood pressure in children. c. Using a Doppler device is recommended for accurate blood pressure measurements until adolescence. d. The disappearance of phase V Korotkoff sounds can be used for the diastolic reading in children. 31. What type of blood pressure measurement error is most likely to occur if the nurse does not check for the presence of an auscultatory gap? a. Diastolic blood pressure may not be heard. b. Diastolic blood pressure may be falsely low. c. Systolic blood pressure may be falsely low. d. Systolic blood pressure may be falsely high. 32. When considering the concepts r/t blood pressure, the nurse knows that the concept of mean arterial pressure (MAP) is best described by which statement? a. MAP is the pressure of the arterial pulse. b. MAP reflects the stroke volume of the heart. c. MAP is the pressure forcing blood into the tissues, averaged over the cardiac cycle. d. MAP is an average of the systolic and diastolic blood pressures and reflects tissue perfusion. 33. A 75-year-old man with a history of hypertension was recently changed to a new antihypertensive drug. He reports feeling dizzy at times. How should the nurse evaluate his blood pressure? a. Blood pressure and pulse should be recorded in the supine, sitting, and standing positions. b. The patient should be directed to walk around the room and his blood pressure assessed after this activity. c. Blood pressure and pulse are assessed at the beginning and at the end of the examination. d. Blood pressure is taken on the right arm and then 5 minutes later on the left arm. 34. The nurse is counting an infant’s respirations. Which technique is correct? a. Watching the chest rise and fall b. Observing the movement of the abdomen c. Placing a hand across the infant’s chest d. Using a stethoscope to listen to the breath sounds 35. When checking for proper blood pressure cuff size, which guideline is correct? a. The standard cuff size is appropriate for all sizes. b. The length of the rubber bladder should equal 80% of the arm circumference. c. The width of the rubber bladder should equal 80% of the arm circumference. d. The width of the rubber bladder should equal 40% of the arm circumference. MULTIPLE RESPONSE 1. While measuring a patient’s blood pressure, the nurse uses the proper technique to obtain an accurate reading. Which of these situations will result in a falsely high blood pressure reading? (Select all that apply.) a. The person supports his or her own arm during the blood pressure reading. b. The blood pressure cuff is too narrow for the extremity. c. The arm is held above level of the heart. d. The cuff is loosely wrapped around the arm. e. The person is sitting with his or her legs crossed. f. The nurse does not inflate the cuff high enough. COMPLETION 1. What is the pulse pressure for a patient whose blood pressure is 158/96 mm Hg and whose pulse rate is 72 beats per minute? Chapter 11: Pain Assessment Jarvis: Physical Examination and Health Assessment, 8th Edition MULTIPLE CHOICE 1. The nurse is evaluating a patient’s pain. Which is an example of acute pain? a. Fibromyalgia b. Arthritic pain c. Kidney stones d. Lower back pain 2. The nurse is teaching a class on pain at a local retirement community. Which statement about the pain experienced by older adults should the nurse include in the instructions? a. “Older adults must learn to tolerate pain.” b. “Pain is a normal process of aging and is to be expected.” c. “Pain indicates a pathologic condition or an injury and is not a normal process of aging.” d. “Older individuals perceive pain to a lesser degree than do younger individuals.” 3. A 4-year-old boy is brought to the emergency department by his mother. She says he points to his stomach and says, “It hurts so bad.” Which pain assessment tool would be the best choice when assessing this child’s pain? a. Descriptor scale b. Numeric rating scale c. Brief pain inventory d. Faces Pain Scale—Revised (FPS-R) 4. A patient states that the pain medication is “not working” and rates his postoperative pain at a 10 on a 1-to-10 scale. Which of these assessment findings indicates an acute pain response to poorly controlled pain? a. Confusion b. Depression c. Hyperventilation d. Increased blood pressure and pulse 5. A 60-year-old woman has developed reflexive sympathetic dystrophy after arthroscopic repair of her shoulder. Which is a key feature of this condition? a. Affected extremity will eventually regain its function. b. Pain is felt at one site but originates from another location. c. Patient’s pain will be associated with nausea, pallor, and diaphoresis. d. Slightest touch, such as a sleeve brushing against her arm, causes severe and intense pain. 6. The nurse is assessing a patient’s pain. What should the nurse know is the most reliable indicator of pain? a. Subjective report b. Physical examination c. Patient’s vital signs d. Results of a computerized axial tomographic scan 7. A patient has had arthritic pain in her hips for several years since a hip fracture. She is able to move around in her room and has not offered any complaints so far this morning. However, when asked, she states that her pain is “bad this morning” and rates it at an 8 on a 1-to-10 scale. What is the likely reason for this? a. The patient is addicted to her pain medications and cannot obtain pain relief. b. The patient does not want to trouble the nursing staff with her complaints. c. The patient is not in pain but rates it high to receive pain medication. d. The patient has experienced chronic pain for years and has adapted to it. 8. The nurse is reviewing the principles of pain. Which type of pain is due to an abnormal processing of the pain impulse through the peripheral or central nervous system? a. Visceral b. Referred c. Cutaneous d. Neuropathic 9. When assessing the quality of a patient’s pain, the nurse should ask which question? a. “When did the pain start?” b. “Is the pain a stabbing pain?” c. “Is it a sharp pain or dull pain?” d. “What does your pain feel like?” 10. When assessing a patient’s pain, the nurse records that the patient has visceral pain. Which condition would the patient have stated in order for the nurse document visceral pain? a. A hip fracture b. Cholecystitis c. Second-degree burns d. Pain after a leg amputation 11. The nurse is reviewing the principles of nociception. During which phase of nociception does the conscious awareness of a painful sensation occur? a. Perception b. Modulation c. Transduction d. Transmission 12. When assessing the intensity of a patient’s pain, which question by the nurse is appropriate? a. “What does your pain feel like?” b. “How much pain do you have now?” c. “How does pain limit your activities?” d. “What makes your pain better or worse?” 13. A patient is complaining of severe knee pain after twisting it during a basketball game and is requesting pain medication. Which action by the nurse is appropriate? a. Completing the physical examination first and then giving the pain medication b. Telling the patient that the pain medication must wait until after the x-ray images are completed c. Evaluating the full range of motion of the knee and then medicating for pain d. Administering pain medication and then proceeding with the assessment 14. The nurse knows that which statement is true regarding the pain experienced by infants? a. Infants feel pain less than adults do. b. The FPS-R can be used to assess pain in infants. c. A procedure that induces pain in adults will also induce pain in the infant. d. Pain in infants can only be assessed by physiologic changes, such as an increased heart rate. 15. A patient has been admitted to the hospital with vertebral fractures r/t osteoporosis. She is in extreme pain. How should the nurse document this type of pain? a. Referred b. Cutaneous c. Visceral d. Deep somatic MULTIPLE RESPONSE 1. During assessment of a patient’s pain, the nurse is aware that certain nonverbal behaviors are associated with chronic pain. Which of these behaviors are associated with chronic pain? (Select all that apply.) a. Bracing b. Rubbing c. Moaning d. Sleeping e. Diaphoresis f. Restlessness 2. During an admission assessment of a patient with dementia, the nurse assesses for pain because the patient has recently had several falls. Which of these are appropriate for the nurse to assess in a patient with dementia? (Select all that apply.) a. Ask the patient, “Do you have pain?” b. Have the patient rate pain on a 1-to-10 scale. c. Assess the patient’s breathing independent of vocalization. d. Note whether the patient is calling out, groaning, or crying. e. Observe the patient’s body language for pacing and agitation. Chapter 12: Nutrition Assessment Jarvis: Physical Examination and Health Assessment, 8th Edition MULTIPLE CHOICE 1. The nurse recognizes which of these people is at greatest risk for undernutrition? a. 30-year-old man b. 50-year-old woman c. 5-month-old infant d. 20-year-old college student 2. When assessing a patient’s nutritional status, what does the nurse need to recognize that sufficient nutrients need to do for optimal nutrition? a. Provide for the minimum body needs. b. Provide an excess of daily body requirements. c. Provide for daily body requirements but do not support increased metabolic demands. d. Provide for daily body requirements and support increased metabolic demands. 3. The nurse is providing nutrition information to the mother of a 1-year-old child. Which of these statements represents accurate information for this age group? a. Maintaining adequate fat and caloric intake is important for a 1-year-old child. b. The recommended dietary allowances for an infant are the same as for an adolescent. c. The baby’s growth is minimal at this age; therefore, caloric requirements are decreased. d. The baby should be placed on skim milk to decrease the risk for coronary artery disease when he or she grows older. 4. A pregnant woman is interested in breastfeeding her baby and asks several questions about the topic. Which statement by the nurse is appropriate? a. “Babies who are breastfed often require supplemental vitamins.” b. “Breastfeeding is best when also supplemented with bottle-feedings.” c. “Breastfeeding is recommended for infants for the first 2 years of life.” d. “Breast milk provides the nutrients necessary for growth, as well as natural immunity.” 5. A mother and her 13-year-old daughter express their concern r/t the daughter’s recent weight gain and her increase in appetite. Which of these statements represents information the nurse should discuss with them? a. Dieting and exercising are necessary at this age. b. Snacks should be high in protein, iron, and calcium. c. Teenagers who have a weight problem should not be allowed to snack. d. A low-calorie diet is important to prevent the accumulation of fat. 6. The nurse is assessing a 30-year-old unemployed immigrant from an underdeveloped country who has been in the United States for 1 month. Which of these problems r/t his nutritional status might the nurse expect to find? a. Obesity b. Hypotension c. Osteomalacia d. Coronary artery disease 7. For the first time, the nurse is seeing a patient who has no history of nutrition-related problems. Which activity should the initial nutritional screening include? a. Anthropometric measures b. Calorie count of nutrients c. Complete physical examination d. Measurement of weight and weight history 8. A patient is asked to indicate on a form how many times he eats a specific food. This method describes which of these tools for obtaining dietary information? a. Food diary b. Calorie count c. 24-hour recall d. Food-frequency questionnaire DIF: Cognitive Level: Understanding (Comprehension) MSC: Client Needs: Health Promotion and Maintenance 9. During a nutritional assessment, why is it important for the nurse to ask a patient what medications he or she is taking? a. Certain drugs can affect the metabolism of nutrients. b. The nurse needs to assess the patient for allergic reactions. c. Medications need to be documented in the record for the physician’s review. d. Medications can affect one’s memory and ability to identify food eaten in the last 24 hours. 10. A patient tells the nurse that his food simply does not have any taste anymore. What is the best response by the nurse? a. “That must be really frustrating.” b. “When did you first notice this change?” c. “My food doesn’t always have a lot of taste either.” d. “Sometimes that happens, but your taste will come back.” 11. The nurse is performing a nutritional assessment on a 15-year-old girl who tells the nurse that she is “so fat.” Assessment reveals that she is 5 feet 4 inches and weighs 110 pounds. What is an appropriate response by the nurse? a. “How much do you think you should weigh?” b. “Don’t worry about it; you’re not that overweight.” c. “The best thing for you would be to go on a diet.” d. “I used to always think I was fat when I was your age.” 12. The nurse is discussing appropriate foods with the mother of a 3-year-old child. Which of these foods are recommended? a. Foods easy to hold such as hot dogs, nuts, and grapes b. Foods that the child will eat, no matter what the food c. Any food, as long as the rest of the family is eating it, too d. Finger foods and nutritious snacks that cannot cause choking 13. The nurse is reviewing the nutritional assessment of an 82-year-old widowed patient. Which of these factors will most likely affect the nutritional status of this patient? a. Increase in taste and smell. b. Living alone on a fixed income. c. Change in cardiovascular status. d. Increase in gastrointestinal motility and absorption. 14. When considering a nutritional assessment, what should the nurse be aware is/are one of the most common anthropometric measurements? a. Height and weight b. Leg circumference c. Chest and waist measurements d. Skinfold thickness of the biceps 15. If a 29-year-old woman weighs 156 pounds, and the nurse determines her ideal body weight to be 120 pounds, then how would the nurse classify the woman’s weight? a. Obese b. Mildly overweight c. Suffering from malnutrition d. Within appropriate range of ideal weight 16. In teaching a patient how to determine total body fat at home, what should the nurse instruct the patient to measure? a. Height and weight b. Frame size and weight c. Waist and hip circumferences d. Mid-upper arm circumference and arm span 17. The nurse is evaluating patients for obesity-related diseases by calculating the waist-to-hip ratios. Which one of these patients would be at increased risk? a. 29-year-old woman whose waist measures 33 inches and hips measure 36 inches b. 32-year-old man whose waist measures 34 inches and hips measure 36 inches c. 38-year-old man whose waist measures 35 inches and hips measure 38 inches d. 46-year-old woman whose waist measures 30 inches and hips measure 38 inches 18. A 50-year-old woman with an elevated total cholesterol level is visiting the clinic to find out about her laboratory results. What would be important for the nurse to include in the patient teaching in relation to these tests? a. The risks of undernutrition should be included. b. Offer methods to reduce the stress in her life. c. Provide information regarding a diet low in saturated fat. d. This condition is hereditary; there is little she can do to change the levels. 19. In performing an assessment on a 49-year-old woman who has imbalanced nutrition as a result of dysphagia, which data would the nurse expect to find? a. Increase in hair growth b. Weight 10% to 20% over ideal c. Sore, inflamed buccal cavity d. Inadequate nutrient or food intake 20. A 21-year-old woman has been on a low-protein liquid diet for the past 2 months. She has had adequate intake of calories and appears well nourished. After further assessment, what would the nurse expect to find? a. Poor skin turgor b. Decreased serum albumin c. Increased lymphocyte count d. Triceps skinfold less than standard 21. The nurse is performing a nutritional assessment on an 80-year-old patient. What is one of the many physiologic changes that can directly affect the nutritional status of the older adult? a. Slowed gastrointestinal motility b. Hyperstimulation of the salivary glands c. Increased sensitivity to spicy and aromatic foods d. Decreased gastrointestinal absorption causing esophageal reflux 22. Which of these interventions is most appropriate when the nurse is planning nutritional interventions for a healthy, active 74-year-old woman? a. Decreasing the amount of carbohydrates to prevent lean muscle catabolism b. Increasing the amount of soy and tofu in her diet to promote bone growth and reverse osteoporosis c. Increasing the number of calories she is eating because of the increased energy needs of the older adult d. Decreasing the number of calories she is eating because of the decrease in energy requirements from the loss of lean body mass 23. A 16-year-old girl is being seen at the clinic for gastrointestinal complaints and weight loss. The nurse determines that many of her complaints may be r/t erratic eating patterns, eating predominantly fast foods, and high caffeine intake. In this situation, which is most appropriate when collecting current dietary intake information? a. Scheduling a time for direct observation of the adolescent during meals b. Asking the patient for a 24-hour diet recall, and assuming it to be reflective of a typical day for her c. Having the patient complete a food diary for 3 days, including 2 weekdays and 1 weekend day d. Using the food frequency questionnaire to identify the amount of intake of specific foods 24. Which of these conditions is due to an inadequate intake of both protein and calories? a. Bulimia b. Obesity c. Marasmus d. Kwashiorkor 25. During an assessment of a patient who has been homeless for several years, the nurse notices that his tongue is magenta in color, which is an indication of a deficiency in what mineral and/or vitamin? a. Iron b. Riboflavin c. Vitamin C d. Vitamin D and calcium 26. A 50-year-old patient has been brought to the emergency department after a housemate found that the patient could not get out of bed alone. He has lived in a group home for years but for several months has not participated in the activities and has stayed in his room. The nurse assesses for signs of undernutrition, and an x-ray study reveals that he has osteomalacia. What deficiency is likely the cause? a. Iron b. Vitamin C c. Riboflavin d. Vitamin D and calcium 27. An older adult patient in a nursing home has been receiving tube feedings for several months. During an oral examination, the nurse notes that patient’s gums are swollen, ulcerated, and bleeding in some areas. The nurse suspects that the patient has what condition? a. Rickets b. Vitamin A deficiency c. Linoleic-acid deficiency d. Vitamin C deficiency 28. The nurse is assessing the body weight as a percentage of ideal body weight on an adolescent patient who was admitted for suspected anorexia nervosa. The patient’s usual weight was 125 pounds, but today she weighs 98 pounds. After calculating the patient’s ideal body weight, what should the nurse conclude? a. The patient is experiencing mild malnutrition. b. The patient is experiencing moderate malnutrition. c. The patient is experiencing severe malnutrition. d. The patient is still within expected parameters with her current weight. MULTIPLE RESPONSE 1. The nurse is assessing a patient who is obese for signs of metabolic syndrome. This condition is diagnosed when three or more certain risk factors are present. Which of these assessment findings are risk factors for metabolic syndrome? (Select all that apply.) a. Triglyceride level of 120 mg/dL b. Blood pressure reading of 110/80 mm Hg c. Blood pressure reading of 140/90 mm Hg d. Fasting plasma glucose level less than 100 mg/dL e. Fasting plasma glucose level greater than or equal to 110 mg/dL COMPLETION 1. A patient has been unable to eat solid food for 2 weeks and is in the clinic today complaining of weakness, tiredness, and hair loss. The patient states that her usual weight is 175 pounds, but today she weighs 161 pounds. What is her recent weight change percentage? Chapter 13: Skin, Hair, and Nails Jarvis: Physical Examination and Health Assessment, 8th Edition MULTIPLE CHOICE 1. The nurse educator is preparing an education module on skin, hair, and nails for the nursing staff. Which of these statements about the epidermal layer of the skin should be included in the module? a. Highly vascular b. Thick and tough c. Thin and nonstratified d. Replaced every 4 weeks 2. The nurse educator is preparing an education module on skin, hair, and nails for the nursing staff. Which of these statements about the dermal layer of the skin should be included in the module? a. Contains mostly fat cells b. Consists mostly of keratin c. Is replaced every 4 weeks d. Contains sensory receptors 3. The nurse is examining a patient who tells the nurse, “I sure sweat a lot, especially on my face and feet but it doesn’t have an odor.” The nurse knows that this is likely r/t a disorder with what part of the body? a. Eccrine glands b. Apocrine glands c. Disorder of the stratum corneum d. Disorder of the stratum germinativum 4. A newborn infant is in the clinic for a well-baby checkup. The nurse observes the infant for the possibility of fluid loss because of which of these factors? a. Subcutaneous fat deposits are high in the newborn. b. Sebaceous glands are overproductive in the newborn. c. The newborn’s skin is more permeable than that of the adult. d. The amount of vernix caseosa dramatically rises in the newborn. 5. The nurse is bathing an 80-year-old man and notices that his skin is wrinkled, thin, lax, and dry. This finding would be r/t which factor in the older adult? a. Increased vascularity of the skin b. Increased numbers of sweat and sebaceous glands c. An increase in elastin and a decrease in subcutaneous fat d. An increased loss of elastin and a decrease in subcutaneous fat 6. During the aging process, the hair can look gray or white and begin to feel thin and fine. What should the nurse understand causes this? a. Increased adipose tissue b. Increase in the vascularity of the scalp c. Decrease in the number of functioning phagocytes d. Decrease in the number of functioning melanocytes 7. During an examination, the nurse finds that a patient has excessive dryness of the skin. How should the nurse document this finding? a. Xerosis b. Pruritus c. Alopecia d. Seborrhea 8. A 22-year-old woman comes to the clinic because of severe sunburn and states, “I was out in the sun for just a couple of minutes.” The nurse begins a medication review with her, paying special attention to which medication class? a. Tetracyclines for acne b. Proton pump inhibitors for heartburn c. Nonsteroidal anti-inflammatory drugs for pain d. Thyroid replacement hormone for hypothyroidism 9. A woman is leaving on a trip to Hawaii and has come in for a checkup. During the examination the nurse learns that she has diabetes and is prescribed oral hypoglycemic agents. What should the nurse include in this patient’s teaching? a. Increased possibility of bruising b. Importance of sunscreen and avoiding direct sunlight c. Lack of availability of glucose-monitoring equipment d. Skin sensitivity as a result of exposure to salt water 10. A 13-year-old girl is interested in obtaining information about the cause of her acne. What should the nurse include in the information about acne? a. It is contagious. b. It has no known cause. c. It is caused by increased sebum production. d. It has been found to be r/t poor hygiene. 11. A 75-year-old woman who has a history of diabetes and peripheral vascular disease has been trying to remove a corn on the bottom of her foot with a pair of scissors. Why is it important that the nurse encourage her to stop trying to remove the corn with scissors? a. The woman could be at increased risk for infection and lesions because of her chronic disease. b. With her diabetes, she has increased circulation to her foot, and it could cause severe bleeding. c. She is 75 years old and is unable to see; consequently, she places herself at greater risk for self-injury with the scissors. d. With her peripheral vascular disease, her range of motion is limited and she may not be able to reach the corn safely. 12. The nurse keeps in mind that a thorough skin assessment is extremely important. What can the skin provide important information about? a. Support systems b. Circulatory status c. Socioeconomic status d. Psychological wellness 13. A patient comes in for a physical examination in late July and states that she was “freezing to death” while waiting for her examination. The nurse notes that her skin is pale and cool. What should the nurse understand is the likely cause? a. Venous pooling b. Peripheral vasodilation c. Peripheral vasoconstriction d. Decreased arterial perfusion 14. A patient comes to the clinic and tells the nurse that he has been confined to his recliner chair for approximately 3 days with his feet down and he asks the nurse to evaluate his feet. What should the nurse expect to find during the assessment? a. Pallor b. Coolness c. Distended veins d. Prolonged capillary filling time 15. A patient is especially worried about an area of skin on her feet that has turned white. The health care provider has told her that her condition is vitiligo. What should the nurse include in the teaching of vitiligo? a. It is associated with an excess of melanin pigment. b. It is a result of excess apocrine glands in her feet. c. It is caused by the complete absence of melanin pigment in an area. d. It is r/t impetigo and can be treated with a prescription ointment. 16. A patient tells the nurse that he has noticed that one of his moles has started to burn and bleed. When assessing his skin, the nurse pays special attention to the danger signs for pigmented lesions and is concerned with which additional finding? a. Color variation b. Border regularity c. Symmetry of lesions d. Diameter of less than 6 mm 17. A patient comes to the clinic and states that he has noticed that his skin is redder than normal. The nurse understands that this condition is erythema. What is the likely cause? a. Decreased amounts of bilirubin in the blood b. Excess blood in the underlying blood vessels c. Decreased perfusion to the surrounding tissues d. Excess blood in the dilated superficial capillaries 18. During a skin assessment, the nurse notices that a Mexican-American patient has skin that is yellowish-brown; however, the skin on the hard and soft palate is pink and the patient’s scleras are white. From these findings, what can the nurse rule out? a. Pallor b. Jaundice c. Cyanosis d. Iron deficiency 19. A black patient is in the intensive care unit because of impending shock after an accident. The nurse expects to find what characteristics in this patient’s skin? a. Ruddy blue b. Generalized pallor c. Ashen, gray, or dull d. Patchy areas of pallor 20. An older adult woman is brought to the emergency department after being found lying on the kitchen floor for 2 days; she is extremely dehydrated. What would the nurse expect to see during the examination? a. Pale mucous membranes b. Smooth mucous membranes and lips c. White patches on the mucous membranes d. Dry mucous membranes and cracked lips 21. A 42-year-old woman states that she has noticed several small, slightly raised, bright red dots on her chest. On examination, the nurse confirms the presence of these “dots.” How should the nurse document these findings? a. Anasarca b. Scleroderma c. Senile angiomas d. Latent myeloma 22. A 65-year-old man with emphysema and bronchitis has come to the clinic for a follow-up appointment. On assessment, the nurse might expect to see which finding? a. Anasarca b. Scleroderma c. Pedal erythema d. Clubbing of the nails 23. A newborn infant has Down syndrome. During the skin assessment, the nurse notices a transient mottling in the trunk and extremities in response to the cool temperature in the examination room. The infant’s mother also notices the mottling and asks what it is. What should the nurse tell the mother that this mottling is called? a. Carotenemia b. Acrocyanosis c. Café au lait d. Cutis marmorata 24. A 35-year-old pregnant woman comes to the clinic for a monthly appointment. During the assessment, the nurse notices that she has a brown patch of hyperpigmentation on her face. The nurse continues the skin assessment. What other finding should the nurse expect? a. Xerosis b. Chloasma c. Keratoses d. Acrochordons 25. A man has come in to the clinic for a skin assessment because he is worried he might have skin cancer. During the skin assessment the nurse notices several areas of pigmentation that look greasy, dark, and “stuck on” his skin. Which is the best description of these? a. Senile lentigines, which do not become cancerous b. Seborrheic keratoses, which do not become cancerous c. Acrochordons, which are precursors to squamous cell carcinoma d. Actinic keratoses, which are precursors to basal cell carcinoma 26. A 70-year-old woman who loves to garden has small, flat, brown macules over her arms and hands. She asks, “What causes these liver spots?” Which is the best response by the nurse? a. “They are signs of decreased hematocrit r/t anemia.” b. “Those are due to the destruction of melanin in your skin from exposure to the sun.” c. “They are clusters of melanocytes that appear after extensive exposure to sunlight.” d. “Those are areas of hyperpigmentation r/t decreased perfusion and vasoconstriction.” 27. The nurse notices that a patient has a solid, elevated, circumscribed lesion that is less than 1 cm in diameter. How should the nurse document this finding? a. A bulla b. A wheal c. A nodule d. A papule 28. The nurse just noted from the medical record that the patient has a lesion that is confluent in nature. What should the nurse expect to find upon examination? a. Lesions that run together b. Annular lesions that have grown together c. Lesions arranged in a line along a nerve route d. Lesions that are grouped or clustered together 29. A patient has had a “terrible itch” for several months that he has been continuously scratching. What might the nurse expect to find upon physical examination? a. A keloid b. A fissure c. Keratosis d. Lichenification 30. A physician has diagnosed a patient with purpura. After leaving the room, a nursing student asks the nurse what the physician saw that led to that diagnosis. What is the best response by the nurse? a. “Blue dilation of blood vessels in a star-shaped linear pattern on the legs.” b. “Fiery red, star-shaped marking on the cheek that has a solid circular center.” c. “Confluent and extensive patch of petechiae and ecchymoses on the feet.” d. “Tiny areas of hemorrhage that are less than 2 mm, round, discrete, and dark red in color.” 31. A mother has noticed that her son, who has been to a new babysitter, has some blisters and scabs on his face and buttocks. On examination, the nurse notices moist, thin-roofed vesicles with a thin erythematous base. What does the nurse suspect? a. Eczema b. Impetigo c. Herpes zoster d. Diaper dermatitis 32. The nurse notices that a school-aged child has bluish-white, red-based spots in her mouth that are elevated approximately 1 to 3 mm. What other signs would the nurse expect to find in this patient? a. Pink, papular rash on the face and neck b. Pruritic vesicles over her trunk and neck c. Hyperpigmentation on the chest, abdomen, and back of the arms d. Red-purple, maculopapular, blotchy rash behind the ears and on the face 33. The nurse is assessing the skin of a patient who has acquired immunodeficiency syndrome (AIDS) and notices multiple patchlike lesions on the temple and beard area that are faint pink in color. What does the nurse suspect? a. Angiomas b. Herpes zoster c. Measles (rubeola) d. Kaposi’s sarcoma 34. A 45-year-old farmer comes in for a skin evaluation and complains of hair loss on his head. His hair seems to be breaking off in patches, and he notices some scaling on his head. What does the nurse suspect? a. Folliculitis b. Tinea capitis c. Toxic alopecia d. Seborrheic dermatitis 35. A mother brings her 10-year-old daughter into the clinic for an examination of the scalp and hair. She states that the child has developed irregularly shaped patches with broken-off, stublike hair in some places; she is worried that this condition could be some form of premature baldness. What is the best response by the nurse? a. “This looks like folliculitis which can be treated with an antibiotic.” b. “This sounds like traumatic alopecia which can be treated with antifungal medications.” c. “This appears to be tinea capitis which is highly contagious and needs immediate attention.” d. “This appears to be trichotillomania. Does your daughter have a habit of absentmindedly twirling her hair?” 36. The nurse has discovered decreased skin turgor in a patient and knows that this finding is expected in which condition? a. Severe obesity b. Severe dehydration c. Childhood growth spurts d. Connective tissue disorders such as scleroderma 37. While performing an assessment of a 65-year-old man with a history of hypertension and coronary artery disease, the nurse notices the presence of bilateral pitting edema in the lower legs. The skin is puffy and tight but normal in color. No increased redness or tenderness is observed over his lower legs, and the peripheral pulses are equal and strong. In this situation, the nurse suspects that the likely cause of the edema is which condition? a. Heart failure b. Venous stasis c. Local inflammation d. Peripheral arterial insufficiency 38. A 40-year-old woman reports a change in mole size, accompanied by color changes, itching, burning, and bleeding over the past month. She has a dark complexion and has no family history of skin cancer, but she has had many blistering sunburns in the past. What is the best response by the nurse? a. Tell the patient to watch the lesion and report back in 2 months. b. Refer the patient because of the suggestion of melanoma on the basis of her symptoms. c. Ask additional questions regarding environmental irritants that may have caused this condition. d. Tell the patient that these signs suggest a compound nevus, which is very common in young to middle-aged adults. 39. The nurse is assessing for clubbing of the fingernails. Which is the best description of clubbing? a. Nail bases that are firm and slightly tender b. Curved nails with a convex profile and ridges across the nails c. Nail bases that feel spongy with an angle of the nail base of 150 degrees d. Nail bases with an angle of 180 degrees or greater and nail bases that feel spongy 40. The nurse is assessing a patient with liver disease for jaundice. Which of these assessment findings is indicative of true jaundice? a. Yellow patches in the outer sclera b. Yellow color of the sclera that extends up to the iris c. Skin that appears yellow when examined under low light d. Yellow deposits on the palms and soles of the feet where jaundice first appears 41. The nurse is assessing for inflammation in a dark-skinned person. Which technique is the best? a. Assessing the skin for cyanosis and swelling b. Palpating the skin for edema and increased warmth c. Assessing the oral mucosa for generalized erythema d. Palpating for tenderness and local areas of ecchymosis 42. A few days after a summer hiking trip, a 25-year-old man comes to the clinic with a rash. On examination, the nurse notes that the rash is red, macular, with a bull’s eye pattern across his midriff and behind his knees. What does the nurse suspect? a. Eczema b. Rubeola c. Lyme disease d. Medication allergy 43. A 52-year-old woman has a papule on her nose that has rounded, pearly borders and a central red ulcer. She said she first noticed it several months ago and that it has slowly grown larger. The nurse suspects which condition? a. Acne b. Melanoma c. Basal cell carcinoma d. Squamous cell carcinoma 44. A father brings in his 2-month-old infant to the clinic because the infant has had diarrhea for the last 24 hours. He says his baby has not been able to keep any formula down and that the diarrhea has been at least every 2 hours. The nurse suspects dehydration. Where should the nurse test for skin mobility and turgor? a. Over the sternum b. On the forehead c. On the forearms d. Over the abdomen 45. A semiconscious woman is brought to the emergency department after she was found on the floor in her kitchen. Her face, nail beds, lips, and oral mucosa are a bright cherry-red color. What should the nurse suspect is the likely cause of these findings? a. Uremia b. Carotenemia c. Polycythemia d. Carbon monoxide poisoning 46. A patient has been admitted for severe psoriasis. The nurse expects to see what finding in the patient’s fingernails? a. Pitting b. Paronychia c. Beau lines d. Splinter hemorrhages MULTIPLE RESPONSE 1. The nurse is preparing for a certification course on skin care and needs to be familiar with the various lesions that may be identified on assessment of the skin. Which of the following definitions are correct? (Select all that apply.) a. Papule: Hypertrophic scar. b. Vesicle: Known as a friction blister. c. Bulla: Elevated, circumscribed lesion filled with turbid fluid (pus). d. Nodule: Solid, elevated, and hard or soft growth that is larger than 1 cm. e. Petechiae: Tiny punctate hemorrhages, 1 to 3 mm, round and discrete, dark red, purple, or brown in color. 2. A patient has been admitted to a hospital after the staff in the nursing home noticed a pressure ulcer in his sacral area. The nurse examines the pressure ulcer and determines that it is a stage II ulcer. Which of these findings are characteristic of a stage II pressure ulcer? (Select all that apply.) a. Intact skin appears red but is not broken. b. Patches of eschar cover parts of the wound. c. Ulcer extends into the subcutaneous tissue. d. Open blister areas have a red-pink wound bed. e. Localized redness in light skin will blanch with fingertip pressure. f. Partial thickness skin erosion is observed with a loss of epidermis or dermis. Chapter 14: Head, Face, and Neck, and Regional Lymphatics Jarvis: Physical Examination and Health Assessment, 8th Edition MULTIPLE CHOICE 1. A physician tells the nurse that a patient’s vertebra prominens is tender and asks the nurse to reevaluate the area in 1 hour. What area of the body will the nurse assess? a. At the level of the C7 vertebra b. At the level of the T11 vertebra c. At the level of the L5 vertebra d. At the level of the S3 vertebra 2. A mother brings her 2-month-old daughter in for an examination and says, “My daughter rolled over against the wall, and now I have noticed that she has this spot that is soft on the top of her head. Is something terribly wrong?” How should the nurse respond? a. “Perhaps that could be a result of your dietary intake during pregnancy.” b. “Your baby may have craniosynostosis, a disease of the sutures of the skull.” c. “That ‘soft spot’ may be an indication of cretinism or congenital hypothyroidism.” d. “That ‘soft spot’ is normal, and actually allows for growth of the brain during the first year of your baby’s life.” 3. The nurse notices that a patient’s palpebral fissures are not symmetric. On examination, the nurse may find that damage has occurred to which cranial nerve (CN)? a. V b. VII c. XI d. XIII 4. A patient is unable to differentiate between sharp and dull stimulation to both sides of her face. What does the nurse suspect? a. Bell palsy b. Scleroderma c. Damage to the trigeminal nerve d. Frostbite with resultant paresthesia to the cheeks 5. When examining the face of a patient, what are the two pairs of salivary glands that are accessible for examination? a. Occipital; submental b. Parotid; submandibular c. Submandibular; occipital d. Sublingual; parotid 6. A patient comes to the clinic complaining of neck and shoulder pain and is unable to turn her head. Which nerve does the nurse suspect is damaged and how should the nurse proceed with the examination? a. XII; assess for a positive Romberg sign. b. XI; palpate the anterior and posterior triangles. c. XI; have patient shrug their shoulders against resistance. d. XII; percuss the sternomastoid and submandibular neck muscles. 7. When examining a patient’s CN function, what muscles should the nurse assess to assess the function of CN XI? a. Sternomastoid and trapezius b. Spinal accessory and omohyoid c. Trapezius and sternomandibular d. Sternomandibular and spinal accessory 8. A patient’s laboratory data reveal an elevated thyroxine (T4) level. What gland should the nurse assess? a. Thyroid b. Parotid c. Adrenal d. Parathyroid 9. A patient says that she has recently noticed a lump in the front of her neck below her “Adam’s apple” that seems to be getting bigger. During the assessment, what finding would lead the nurse to suspect that this may not be a cancerous thyroid nodule? a. It is tender. b. It is mobile and soft. c. It disappears when the patient smiles. d. It is hard and fixed to the surrounding structures. 10. The nurse notices that a patient’s submental lymph nodes are enlarged. In an effort to identify the cause of the node enlargement, what should the nurse assess? a. Infraclavicular area b. Supraclavicular area c. Area distal to the enlarged node d. Area proximal to the enlarged node 11. The nurse is explaining to a student nurse the four areas in the body where lymph nodes are accessible. Which areas should the nurse include in her explanation to the student? a. Head, breasts, groin, and abdomen b. Arms, breasts, inguinal area, and legs c. Head and neck, arms, breasts, and axillae d. Head and neck, arms, inguinal area, and axillae 12. A mother brings her newborn in for an assessment and asks, “Is there something wrong with my baby? His head seems so big.” Which statement is true regarding the relative proportions of the head and trunk of the newborn? a. At birth, the head is one fifth the total length. b. Head circumference should be greater than chest circumference at birth. c. The head size reaches 90% of its final size when the child is 3 years old. d. When the anterior fontanel closes at 2 months, the head will be more proportioned to the body. 13. A patient, an 85-year-old woman, is complaining about the fact that the bones in her face have become more noticeable. What explanation should the nurse give her? a. Diets low in protein and high in carbohydrates may cause enhanced facial bones. b. Bones can become more noticeable if the person does not use a dermatologically approved moisturizer. c. More noticeable facial bones are probably due to a combination of factors r/t aging, such as decreased elasticity, subcutaneous fat, and moisture in her skin. d. Facial skin becomes more elastic with age. This increased elasticity causes the skin to be more taught, drawing attention to the facial bones. 14. A patient reports excruciating headache pain on one side of his head, especially around his eye, forehead, and cheek that has lasted approximately to 2 hours, occurring once or twice each day. What should the nurse suspect? a. Hypertension b. Cluster headaches c. Tension headaches d. Migraine headaches 15. A patient complains that while studying for an examination he began to notice a severe headache in the frontotemporal area of his head that is throbbing and is somewhat relieved when he lies down. He tells the nurse that his mother also had these headaches. What should the nurse suspect? a. Hypertension b. Cluster headaches c. Tension headaches d. Migraine headaches 16. A 19-year-old college student is brought to the emergency department with a severe headache he describes as, “Like nothing I’ve ever had before.” His temperature is 40 C, and he has a stiff neck. The nurse looks for other signs and symptoms of which problem? a. Head injury b. Cluster headache c. Migraine headache d. Meningeal inflammation 17. During a well-baby checkup, the nurse notices that a 1-week-old infant’s face looks small compared with his cranium, which seems enlarged. On further examination, the nurse also notices dilated scalp veins and downcast or “setting sun” eyes. The nurse suspects which condition? a. Craniotabes b. Microcephaly c. Hydrocephalus d. Caput succedaneum 18. The nurse needs to palpate the temporomandibular joint for crepitation. Where is this joint located? a. Just below the hyoid bone and posterior to the tragus b. Just below the vagus nerve and posterior to the mandible c. Just below the temporal artery and anterior to the tragus d. Just below the temporal artery and anterior to the mandible 19. A patient has come in for an examination and states, “I have this spot in front of my ear lobe on my cheek that seems to be getting bigger and is tender. What do you think it is?” The nurse notes swelling below the angle of the jaw. What does the nurse suspect? a. Inflammation of the thyroid gland b. Inflammation of the parotid gland c. Infection in the occipital lymph node d. Infection in the submental lymph node 20. A male patient with a history of acquired immunodeficiency syndrome (AIDS) has come in for an examination and states, “I think that I have the mumps.” What should the nurse examine first? a. Thyroid gland b. Parotid gland c. Cervical lymph nodes d. Mouth and skin for lesions 21. The nurse suspects that a patient has hyperthyroidism, and the laboratory data indicate that the patient’s T4 and T3 hormone levels are elevated. Which of these findings would the nurse most likely find on examination? a. Dyspnea b. Tachycardia c. Constipation d. Atrophied nodular thyroid gland 22. A visitor from Poland who does not speak English seems to be somewhat apprehensive about the nurse examining his neck. How should the nurse proceed that would allow the patient to feel more comfortable with the nurse examining his thyroid gland? a. Behind with the nurse’s hands placed firmly around his neck b. The side with the nurse’s eyes averted toward the ceiling and thumbs on his neck c. The front with the nurse’s thumbs placed on either side of his trachea and his head tilted forward d. The front with the nurse’s thumbs placed on either side of his trachea and his head tilted backward 23. A patient’s thyroid gland is enlarged, and the nurse is preparing to auscultate the thyroid gland for the presence of a bruit. What technique should the nurse use to assess for a bruit. a. Palpate the thyroid while the patient is swallowing. b. Auscultate the thyroid with the bell of the stethoscope. c. Palpate the thyroid while the patient holds their breath. d. Auscultate the thyroid with the diaphragm of the stethoscope. 24. The nurse notices that an infant has a large, soft lump on the side of his head and that his mother is very concerned. The mother tells the nurse that she noticed the lump approximately 8 hours after her baby’s birth and that it seems to be getting bigger. What is a possible explanation for this? a. Hydrocephalus b. Craniosynostosis c. Cephalhematoma d. Caput succedaneum 25. During an admission assessment, the nurse notices that a male patient has an enlarged and rather thick skull. The nurse suspects acromegaly. What additional finding would the nurse assess for to confirm this suspicion? a. Exophthalmos b. Bowed long bones c. Acorn-shaped cranium d. Coarse facial features 26. When examining children affected with Down syndrome (trisomy 21), what should the nurse look for r/t this disorder? a. Ear dysplasia b. Long, thin neck c. Protruding thin tongue d. Narrow and raised nasal bridge 27. A patient visits the clinic because he has recently noticed that the left side of his mouth is paralyzed. He states that he cannot whistle but the nurse notes he can still raise his eyebrows. What does the nurse suspect? a. Bell palsy b. Cushing syndrome c. Parkinson syndrome d. Experienced a cerebrovascular accident (CVA) or stroke 28. A woman comes to the clinic and states, “I’ve been sick for so long! My eyes have gotten so puffy, and my eyebrows and hair have become coarse and dry.” For what condition should the nurse assess for other signs and symptoms? a. Cachexia b. Myxedema c. Graves disease d. Parkinson syndrome 29. During an examination of a female patient, the nurse notes lymphadenopathy and suspects an acute infection. How do acutely infected lymph nodes typically appear? a. Clumped b. Unilateral c. Firm but freely movable d. Soft and nontender 30. The physician reports that a patient with a neck tumor has a tracheal shift. The nurse should understand that what is occurring to the patient’s trachea? a. Pushed downward b. Pulled to the affected side c. Pushed to the unaffected side d. Pulled downward in a rhythmic pattern 31. During an assessment of an infant, the nurse notes that the fontanels are depressed and sunken. The nurse suspects which condition? a. Rickets b. Dehydration c. Mental retardation d. Increased intracranial pressure 32. The nurse is performing an assessment on a 7-year-old child who has symptoms of chronic watery eyes, sneezing, and clear nasal drainage. The nurse notices the presence of a transverse line across the bridge of the nose, dark blue shadows below the eyes, and a double crease on the lower eyelids. What does the nurse suspect is the cause of these signs and symptoms? a. Chronic allergies b. Lymphadenopathy c. Nasal congestion d. Upper respiratory infection 33. While performing a well-child assessment on a 5 year old, the nurse notes the presence of palpable, bilateral, cervical, and inguinal lymph nodes. They are approximately 0.5 cm in size, round, mobile, and nontender. What do these findings lead the nurse to conclude? a. The child has chronic allergies. b. The child likely has an infection. c. These are normal findings for a well child of this age. d. These findings indicate a need for additional evaluation. 34. The nurse has just completed a lymph node assessment on a 60-year-old healthy female patient. When palpating the nodes on this healthy 60-year-old adult, how did the lymph nodes feel? a. Fixed b. Nonpalpable c. Rubbery, discrete, and mobile d. Large, firm, and fixed to the tissue 35. During an examination of a patient in her third trimester of pregnancy, the nurse notices that the patient’s thyroid gland is slightly enlarged. No enlargement had been previously noticed. What does the nurse suspect? a. An iodine deficiency b. Early signs of goiter c. A normal enlargement of the thyroid gland during pregnancy d. Possible thyroid cancer and the need for further evaluation 36. During an examination, the nurse knows that the best way to palpate the lymph nodes in the neck is described by which statement? a. Using gentle pressure, palpate with both hands to compare the two sides. b. Using strong pressure, palpate with both hands to compare the two sides. c. Gently pinch each node between one’s thumb and forefinger, and then move down the neck muscle. d. Using the index and middle fingers, gently palpate by applying pressure in a rotating pattern. 37. During a well-baby checkup, a mother is concerned because her 2-month-old infant cannot hold her head up when she is pulled to a sitting position. Which response by the nurse is appropriate? a. “Head control is usually achieved by 4 months of age.” b. “You shouldn’t be trying to pull your baby up like that until she is older.” c. “Head control should be achieved by this time.” d. “This inability indicates possible nerve damage to the neck muscles.” 38. During an examination of a 3-year-old child, the nurse notices a bruit over the left temporal area. What should the nurse do? a. Check for the bruit again in 1 hour. b. Stop the examination, and notify the physician. c. Continue the examination because a bruit is a normal finding for this age. d. Notify the parents that a bruit has been detected in their child and requires further evaluation. 39. During an examination, the nurse finds that a patient’s left temporal artery is tortuous and feels hardened and tender, compared with the right temporal artery. The nurse suspects which condition? a. Bell palsy b. Crepitation c. Mastoiditis d. Temporal arteritis MULTIPLE RESPONSE 1. The nurse is assessing a 1-month-old infant at his well-baby checkup. Which assessment findings are appropriate for this age? (Select all that apply.) a. Absent tonic neck reflex b. Nonpalpable cervical lymph nodes c. Fontanels firm and slightly concave d. Head circumference equal to chest circumference e. Head circumference less than chest circumference f. Head circumference greater than chest circumference Chapter 15: Eyes Jarvis: Physical Examination and Health Assessment, 8th Edition MULTIPLE CHOICE 1. When examining the eye, the nurse notices that the patient’s eyelid margins approximate completely. What does the nurse understand about this assessment finding? a. It is expected. b. It may result in problems with tearing. c. It indicates increased intraocular pressure. d. It may indicate a problem with extraocular muscles. 2. During ocular examinations, what should the nurse keep in mind regarding the movement of the extraocular muscles? a. Is decreased in the older adult. b. Is stimulated by CNs III, IV, and VI. c. Is impaired in a patient with cataracts. d. Is stimulated by cranial nerves (CNs) I and II. 3. The nurse is performing an external eye examination. Which statement regarding the outer layer of the eye is true? a. The outer layer of the eye is very sensitive to touch. b. The outer layer of the eye is darkly pigmented to prevent light from reflecting internally. c. The trigeminal nerve (CN V) and the trochlear nerve (CN IV) are stimulated when the outer surface of the eye is stimulated. d. The visual receptive layer of the eye in which light waves are changed into nerve impulses is located in the outer layer of the eye. 4. When examining a patient’s eyes, what should the nurse be aware that stimulation of the sympathetic branch of the autonomic nervous system causes? a. Pupillary constriction b. Adjusts the eye for near vision c. Causes contraction of the ciliary body d. Elevates the eyelid and dilates the pupil 5. The nurse is reviewing causes of increased intraocular pressure. Which of these factors determines intraocular pressure? a. Thickness or bulging of the lens b. Posterior chamber as it accommodates increased fluid c. Contraction of the ciliary body in response to the aqueous within the eye d. Amount of aqueous produced and resistance to its outflow at the angle of the anterior chamber 6. The nurse is conducting a visual examination. Which of these statements regarding visual pathways and visual fields is true? a. The right side of the brain interprets the vision for the O.D. b. The image formed on the retina is upside down and reversed from its actual appearance in the outside world. c. Light rays are refracted through the transparent media of the eye before striking the pupil. d. Light impulses are conducted through the optic nerve to the temporal lobes of the brain. 7. The nurse is testing a patient’s visual accommodation. How is accommodation assessed? a. Pupillary dilation when looking at a distant object b. Involuntary blinking in the presence of bright light c. Pupillary constriction when looking at a near object d. Changes in peripheral vision in response to bright light 8. A mother asks when her newborn infant’s eyesight will be developed. What is the best response by the nurse? a. “Vision is not totally developed until 2 years of age.” b. “Infants develop the ability to focus on an object at approximately 8 months of age.” c. “By approximately 3 months of age, infants develop more coordinated eye movements and can fixate on an object.” d. “Most infants have uncoordinated eye movements for the first year of life.” 9. The nurse is reviewing the age-related changes in the eye for a class. Which of these physiologic changes is responsible for presbyopia? a. Loss of lens elasticity b. Degeneration of the cornea c. Decreased adaptation to darkness d. Decreased distance vision abilities 10. Which of these assessment findings would the nurse expect to see when examining the eyes of a black patient? a. Increased night vision b. Dark retinal background c. Increased photosensitivity d. Narrowed palpebral fissures 11. A 52-year-old patient describes the presence of occasional floaters or spots moving in front of his eyes. How should the nurse proceed? a. Examine the retina to determine the number of floaters. b. Presume the patient has glaucoma and refer him for further testing. c. Consider these to be abnormal findings, and refer him to an ophthalmologist. d. Understand that floaters are usually insignificant and are caused by condensed vitreous fibers. 12. The nurse is preparing to assess the visual acuity of a 16-year-old patient. How should the nurse proceed? a. Perform the confrontation test. b. Ask the patient to read the print on a handheld Jaeger card. c. Use the Snellen chart positioned 20 feet away from the patient. d. Determine the patient’s ability to read newsprint at a distance of 12 to 14 inches. 13. A patient’s vision is recorded as 20/30 when the Snellen eye chart is used. How should the nurse interpret these results? a. At 30 feet the patient can read the entire chart. b. The patient can read at 20 feet what a person with normal vision can read at 30 feet. c. The patient can read the chart from 20 feet in the O.S. and 30 feet in the O.D. d. The patient can read from 30 feet what a person with normal vision can read from 20 feet. 14. A patient is unable to read even the largest letters on the Snellen chart. The nurse should take which action next? a. Refer the patient to an ophthalmologist or optometrist for further evaluation. b. Assess whether the patient can count the nurse’s fingers when they are placed in front of his or her eyes. c. Ask the patient to put on his or her reading glasses and attempt to read the Snellen chart again. d. Shorten the distance between the patient and the chart until the letters are seen, and record that distance. 15. A patient’s vision is recorded as 20/80 in each eye. How does the nurse interpret this finding? a. Patient has presbyopia. b. Patient as poor vision. c. Patient has acute vision. d. Patient has normal vision. 16. When performing the corneal light reflex assessment, the nurse notes that the light is reflected at 2 o’clock in each eye. What should the nurse do regarding this finding? a. Record this as a normal finding. b. Refer the individual for further evaluation. c. Document this finding as an asymmetric light reflex. d. Perform the confrontation test to validate the findings. 17. The nurse is performing the diagnostic positions test. Which result is a normal finding? a. Convergence of the eyes b. Parallel movement of both eyes c. Nystagmus in extreme superior gaze d. Slight amount of lid lag when moving the eyes from a superior to an inferior position 18. During an assessment of the sclera of a black patient, the nurse would consider which of these an expected finding? a. Yellow fatty deposits over the cornea b. Presence of small brown macules on the sclera c. Pallor near the outer canthus of the lower lid d. Yellow color of the sclera that extends up to the iris 19. A 60-year-old man is at the clinic for an eye examination. The nurse suspects that he has ptosis of one eye. How should the nurse check for this? a. Perform the confrontation test. b. Assess the individual’s near vision. c. Observe the distance between the palpebral fissures. d. Perform the corneal light test, and look for symmetry of the light reflex. 20. During an examination of the eye, the nurse would expect what normal finding when assessing the lacrimal apparatus? a. Presence of tears along the inner canthus b. Blocked nasolacrimal duct in a newborn infant c. Absence of drainage from the puncta when pressing against the inner orbital rim d. Slight swelling over the upper lid and along the bony orbit if the individual has a cold 21. When assessing the pupillary light reflex, the nurse should use which technique? a. Shine a penlight from directly in front of the patient, and inspect for pupillary constriction. b. Ask the patient to follow the penlight in eight directions, and observe for bilateral pupil constriction. c. Shine a light across the pupil from the side, and observe for direct and consensual pupillary constriction. d. Ask the patient to focus on a distant object. Then ask the patient to follow the penlight to approximately 7 cm from the nose. 22. The nurse is assessing a patient’s eyes for the accommodation response and would expect to see which normal finding? a. Dilation of the pupils b. Consensual light reflex c. Disconjugate movement of the eyes d. Convergence of the axes of the eyes 23. In using the ophthalmoscope to assess a patient’s eyes, the nurse notices a red glow in the patient’s pupils. Based on this finding, what should the nurse do? a. Suspect that an opacity is present in the lens or cornea. b. Check the light source of the ophthalmoscope to verify that it is functioning. c. Consider the red glow a normal reflection of the ophthalmoscope light off the inner retina. d. Continue with the ophthalmoscopic examination, and refer the patient for further evaluation. 24. The nurse is examining a patient’s retina with an ophthalmoscope. Which finding is considered normal? a. Optic disc that is a yellow-orange color. b. Presence of pigmented crescents in the macular area. c. Optic disc margins that are blurred around the edges. d. Presence of the macula located on the nasal side of the retina. 25. A 2-week-old infant can fixate on an object but cannot follow a light or bright toy. What should the nurse do? a. Consider this a normal finding. b. Continue with the examination, and assess visual fields. c. Assess the pupillary light reflex for possible blindness. d. Expect that a 2-week-old infant should be able to fixate and follow an object. 26. The nurse is assessing color vision of a male child. Which statement is correct? a. Color vision should be checked annually until the age of 18 years. b. Color vision screening should begin at the child’s 2-year checkup. c. The nurse should ask the child to identify the color of his or her clothing. d. Testing for color vision should be done once between the ages of 4 and 8 years. 27. The nurse is performing an eye-screening clinic at a daycare center. When examining a 2-year-old child, the nurse suspects that the child has a “lazy eye”. What should the nurse do next? a. Examine the external structures of the eye. b. Assess visual acuity with the Snellen eye chart. c. Assess the child’s visual fields with the confrontation test. d. Test for strabismus by performing the corneal light reflex test. 28. The nurse is performing an eye assessment on an 80-year-old patient. Which of these findings is considered abnormal? a. Decrease in tear production b. Unequal pupillary constriction in response to light c. Presence of arcus senilis observed around the cornea d. Loss of the outer hair on the eyebrows attributable to a decrease in hair follicles 29. The nurse notices the presence of periorbital edema when performing an eye assessment on a 70-year-old patient. What should the nurse do next? a. Check for the presence of exophthalmos. b. Suspect that the patient has hyperthyroidism. c. Ask the patient if he or she has a history of heart failure. d. Assess for blepharitis, which is often associated with periorbital edema. 30. What is the nurse assessing for when he or she directs a light across the iris of a patient’s eye from the temporal side? a. Drainage from dacryocystitis b. Presence of conjunctivitis over the iris c. Presence of shadows, which may indicate glaucoma d. Scattered light reflex, which may be indicative of cataracts 31. In a patient who has anisocoria, what would the nurse expect to observe? a. Dilated pupils b. Excessive tearing c. Pupils of unequal size d. Uneven curvature of the lens 32. A patient comes to the emergency department after a boxing match, and his O.S. is almost swollen shut. He has bruises on his face and neck. He says he is worried because he “can’t see well” from his O.S. The physician suspects retinal detachment. What finding would support this suspicion? a. Loss of central vision b. Loss of peripheral vision c. Sudden loss of pupillary constriction and accommodation d. Shadow or diminished vision in one quadrant or one half of the visual field 33. A patient comes into the clinic reporting pain in her O.D. On examination, the nurse sees a pustule at the lid margin that is painful to touch, red, and swollen. What is the correct term for this finding? a. Chalazion b. Hordeolum c. Blepharitis d. Dacryocystitis 34. A 68-year-old woman is in the eye clinic for a checkup. She tells the nurse that she has been having trouble reading the paper, sewing, and even seeing the faces of her grandchildren. On examination, the nurse notes that she has some loss of central vision but her peripheral vision is normal. What do these findings suggest? a. Macular degeneration b. Vision that is normal for someone her age c. The beginning stages of cataract formation d. Increased intraocular pressure or glaucoma 35. A patient comes into the emergency department after an accident at work. A machine blew dust into his eyes, and he was not wearing safety glasses. The nurse examines his corneas by shining a light from the side across the cornea. What findings would suggest that he has suffered a corneal abrasion? a. Smooth and clear corneas b. Opacity of the lens behind the cornea c. Bleeding from the areas across the cornea d. Shattered look to the light rays reflecting off the cornea 36. An ophthalmic examination reveals papilledema. What does this finding indicate? a. Retinal detachment b. Diabetic retinopathy c. Acute-angle glaucoma d. Increased intracranial pressure 37. During a physical education class, a student is hit in the eye with the end of a baseball bat. When examined in the emergency department, the nurse notices the presence of blood in the anterior chamber of the eye. What does this finding indicate? a. Hypopyon b. Hyphema c. Pterygium d. A corneal abrasion 38. During an assessment, the nurse notices that an older adult patient has tears rolling down his face from his O.S. Closer examination shows that the lower lid is loose and rolling outward. The patient complains of his eye feeling “dry and itchy.” Which action by the nurse is correct? a. Documenting the finding as ptosis b. Assessing for other signs of ectropion c. Assessing the eye for a possible foreign body d. Contacting the prescriber; these are signs of basal cell carcinoma 1. During an examination, a patient states that she was diagnosed with open-angle glaucoma 2 years ago. The nurse assesses for characteristics of open-angle glaucoma. Which of these are characteristics of open-angle glaucoma? (Select all that apply.) a. Immediate treatment is needed. b. Virtually no symptoms are exhibited. c. Vision loss begins with peripheral vision. d. Patient may experience sensitivity to light, nausea, and halos around lights. e. Patient experiences tunnel vision in the late stages. f. Open-angle glaucoma causes sudden attacks of increased pressure that cause blurred vision. Chapter 16: Ears Jarvis: Physical Examination and Health Assessment, 8th Edition MULTIPLE CHOICE 1. The nurse is examining a patient’s ears and notices cerumen in the external canal. Which of these statements about cerumen is correct? a. Wet, honey-colored cerumen is a sign of infection. b. The presence of cerumen is indicative of poor hygiene. c. The purpose of cerumen is to protect and lubricate the ear. d. Cerumen is necessary for transmitting sound through the auditory canal. 2. When examining the ear with an otoscope, how should the tympanic membrane look? a. Light pink with a slight bulge b. Pearly gray and slightly concave c. Whitish with black flecks or dots d. Pulled in at the base of the cone of light 3. A patient with a middle ear infection asks the nurse, “What does the middle ear do?” Which is the best response by the nurse? a. It helps maintain balance. b. It interprets sounds as they enter the ear. c. It conducts vibrations of sounds to the inner ear. d. It increases the amplitude of sound for the inner ear to function. 4. The nurse is reviewing the function of the cranial nerves (CNs). Which CN is responsible for conducting nerve impulses to the brain from the organ of Corti? a. I b. III c. VIII d. XI 5. The nurse is assessing a patient who may have hearing loss. Which of these statements is true concerning air conduction? a. Air conduction is the normal pathway for hearing. b. Amplitude of sound determines the pitch that is heard. c. Vibrations of the bones in the skull cause air conduction. d. Loss of air conduction is called a conductive hearing loss. 6. A patient has been identified as having a sensorineural hearing loss. What would be important for the nurse to do during the assessment of this patient? a. Speak loudly so the patient can hear the questions. b. Assess for middle ear infection as a possible cause. c. Ask the patient what medications he is currently taking. d. Look for the source of the obstruction in the external ear. 7. During an interview, the patient states he has the sensation that “everything around him is spinning.” What part of the ear should the nurse recognize is responsible for this sensation? a. Cochlea b. CN VIII c. Labyrinth d. Organ of Corti 8. A patient in her first trimester of pregnancy is diagnosed with rubella. Which of these statements is correct regarding the significance of this in relation to the infant’s hearing? a. Rubella may affect the mother’s hearing but not the infant’s. b. Rubella can damage the infant’s organ of Corti, which will impair hearing. c. Rubella can impair the development of cranial nerve VIII and thus affect hearing. d. Rubella is especially dangerous to the infant’s hearing in the second trimester of pregnancy. 9. The mother of a 2-year-old is concerned because her son has had three ear infections in the past year. What would be an appropriate response by the nurse? a. “It is unusual for a small child to have frequent ear infections unless something else is wrong.” b. “We need to check the immune system of your son to determine why he is having so many ear infections.” c. “Ear infections are not uncommon in infants and toddlers because they tend to have more cerumen in the external ear.” d. “Your son’s eustachian tube is shorter and wider than yours because of his age, which allows for infections to develop more easily.” 10. A 31-year-old patient tells the nurse that he has noticed a progressive loss in his hearing. He says that it does seem to help when people speak louder or if he turns up the volume of a television or radio. What is the most likely cause of this hearing loss? a. Presbycusis b. Otosclerosis c. Trauma to the bones d. Frequent ear infections 11. A 70-year-old patient tells the nurse that he has noticed that he is having trouble hearing, especially in large groups. He says “I can’t always tell where the sound is coming from” and that the words often sound “mixed up.” What might the nurse suspect as the cause for this change? a. Atrophy of the apocrine glands b. Cilia becoming coarse and stiff c. Nerve degeneration in the inner ear d. Scar tissue in the tympanic membrane 12. During an assessment of a 20-year-old Asian patient, the nurse notices that he has dry, flaky, white cerumen in his canal. What is the significance of this finding? a. It represents poor hygiene. b. It is probably the result of lesions from eczema in his ear. c. It is a normal finding, and no further follow-up is necessary. d. It could be indicative of change in cilia; the nurse should assess for hearing loss. 13. The nurse is taking the history of a patient who may have a perforated eardrum. What would be an important question in this situation? a. “Do you ever notice ringing or crackling in your ears?” b. “When was the last time you had your hearing checked?” c. “Have you ever been told that you have any type of hearing loss?” d. “Is there any relationship between the ear pain and the discharge you mentioned?” 14. A 31-year-old patient tells the nurse that he has noticed pain in his left ear when people speak loudly to him. What does this finding indicate? a. A cerumen impaction b. Normal for people of his age c. Possible middle ear infection d. A characteristic of recruitment 15. While discussing the history of a 6-month-old infant, the mother tells the nurse that she took a significant amount of aspirin while she was pregnant. What question would the nurse want to include in the history? a. “Does your baby seem to startle with loud noises?” b. “Has your baby had any surgeries on her ears?” c. “Have you noticed any drainage from her ears?” d. “How many ear infections has your baby had since birth?” 16. The nurse is performing an otoscopic examination on an adult. Which of these actions is correct? a. Tilt the person’s head forward during the examination. b. Once the speculum is in the ear, releasing the traction. c. Pulling the pinna up and back before inserting the speculum. d. Using the smallest speculum to decrease the amount of discomfort. 17. The nurse is assessing a 16-year-old patient who has suffered head injuries from a recent motor vehicle accident. Which of these statements indicates the most important reason for assessing for any drainage from the ear canal? a. The auditory canal may be occluded from increased cerumen. b. If the drum has ruptured, then purulent drainage will result. c. Bloody or clear watery drainage can indicate a basal skull fracture. d. Foreign bodies from the accident may cause occlusion of the ear canal. 18. In performing a voice test to assess hearing, which of these actions would the nurse perform? a. Shield the lips so that the sound is muffled. b. Whisper a set of random numbers and letters, and then ask the patient to repeat them. c. Ask the patient to place his or her finger in their ear to occlude outside noise. d. Stand approximately 4 feet away to ensure that the patient can really hear at this distance. 19. How should the nurse perform an examination of a 2-year-old child with a suspected ear infection? a. Pull the ear up and back before inserting the speculum. b. Omit the otoscopic examination if the child has a fever. c. Ask the mother to leave the room while examining the child. d. Perform the otoscopic examination at the end of the assessment. 20. The nurse is preparing to perform an otoscopic examination of a newborn infant. Which statement is true regarding this examination? a. An injected membrane may indicate an infection. b. The eardrum will appear in the oblique position. c. The normal membrane may appear thick and opaque. d. The appearance of the membrane is identical to that of an adult. 21. The nurse assesses the hearing of a 7-month-old by clapping hands. What is the expected response? a. The infant shows no obvious response to the noise. b. The infant shows a startle and acoustic blink reflex. c. The infant turns his or her head to localize the sound. d. The infant stops any movement, and appears to listen for the sound. 22. The nurse is performing an ear examination of an 80-year-old patient. Which of these findings would be considered normal? a. High-tone frequency loss b. Thin, translucent membrane c. Shiny, pink tympanic membrane d. Increased elasticity of the pinna 23. An assessment of a 23-year-old patient reveals the following: an auricle that is tender and reddish-blue in color with small vesicles. The nurse would need to know additional information that includes which of these? a. Any change in ability to hear b. Any recent drainage from the ear c. Recent history of trauma to the ear d. Any prolonged exposure to extreme cold 24. While performing the otoscopic examination of a 3-year-old boy who has been pulling on his left ear, the nurse finds that his left tympanic membrane is bright red and that the light reflex is not visible. What do these findings indicate? a. Cholesteatoma b. A fungal infection c. An acute otitis media d. A perforation of the eardrum 25. The mother of a 2-year-old toddler is concerned about the upcoming placement of tympanostomy tubes in her son’s ears. The nurse would include which of these statements in the teaching plan? a. The tubes are placed in the inner ear. b. The tubes are used in children with sensorineural loss. c. The tubes are permanently inserted during a surgical procedure. d. The purpose of the tubes is to decrease the pressure and allow for drainage. 26. In an individual with otitis externa, which of these signs would the nurse expect to find on assessment? a. Rhinorrhea b. Periorbital edema c. Pain over the maxillary sinuses d. Enlarged superficial cervical nodes 27. When performing an otoscopic examination of a 5-year-old child with a history of chronic ear infections, the nurse sees that his right tympanic membrane is amber-yellow in color and that air bubbles are visible behind the tympanic membrane. The child reports occasional hearing loss and a popping sound with swallowing. Based on this data, what does the nurse conclude? a. An acute purulent otitis externa b. Most likely a serous otitis media c. Evidence of a resolving cholesteatoma d. Experiencing the early stages of perforation 28. The nurse is performing an assessment on a 65-year-old man. He reports a crusty nodule behind the pinna. It intermittently bleeds and has not healed over the past 6 months. On physical assessment, the nurse finds an ulcerated crusted nodule with an indurated base. Based on these findings, what does the nurse suspect? a. Most likely a keloid b. Probably a benign sebaceous cyst c. Could be a potential carcinoma, and the patient should be referred for a biopsy d. A tophus, which is common in the older adult and is a sign of gout 29. The nurse suspects that a patient has otitis media. Early signs of otitis media include which of these findings of the tympanic membrane? a. Hypomobility b. Fiery red and bulging of entire eardrum c. Retraction with landmarks clearly visible d. Flat, slightly pulled in at the center, and moves with insufflation 30. The nurse is performing a middle ear assessment on a 15-year-old patient who has had a history of chronic ear infections. When examining the right tympanic membrane, the nurse sees the presence of dense white patches. The tympanic membrane is otherwise unremarkable. It is pearly, with the light reflex at 5 o’clock and landmarks visible. What should the nurse do? a. Refer the patient for the possibility of a fungal infection. b. Recognize that these are scars caused from frequent ear infections. c. Consider that these findings may represent the presence of blood in the middle ear. d. Be concerned about the ability to hear because of this abnormality on the tympanic membrane. 31. The nurse is preparing to do an otoscopic examination on a 2-year-old child. Which one of these reflects the correct procedure? a. Pulling the pinna down b. Pulling the pinna up and back c. Slightly tilting the child’s head toward the examiner d. Instructing the child to touch their chin to their chest 32. The nurse is conducting a child safety class for new mothers. Which factor places young children at risk for ear infections? a. Family history b. Air conditioning c. Excessive cerumen d. Passive cigarette smoke 33. During an otoscopic examination, the nurse notices an area of black and white dots on the tympanic membrane and the ear canal wall. What does this finding suggest? a. Malignancy b. Viral infection c. Blood in the middle ear d. Yeast or fungal infection 34. A 17-year-old student is a swimmer on her high school’s swim team. She has had three bouts of otitis externa this season and wants to know what to do to prevent it. What should the nurse include in the instructions? a. Use a cotton-tipped swab to dry ear canals thoroughly after each swim. b. Use rubbing alcohol or 2% acetic acid eardrops after every swim. c. Irrigate the ears with warm water and a bulb syringe after each swim. d. Rinse the ears with a warmed solution of mineral oil and hydrogen peroxide. 35. During an examination, the patient states he is hearing a buzzing sound and says that it is “driving me crazy!” What does this indicate? a. Vertigo b. Pruritus c. Tinnitus d. Cholesteatoma 36. During an examination, the nurse notices that the patient stumbles a little while walking, and, when she sits down, she holds on to the sides of the chair. The patient states, “It feels like the room is spinning!” What do this signs and symptoms indicate? a. Tinnitus b. Dizziness c. Objective vertigo d. Subjective vertigo 37. A patient has been admitted after an accident at work. During the assessment, the patient is having trouble hearing and states, “I don’t know what is the matter. All of a sudden I can’t hear you out of my left ear!” What should the nurse do next? a. Irrigate the ear with rubbing alcohol. b. Notify the patient’s health care provider. c. Prepare to remove cerumen from the patient’s ear. d. Make note of this finding for the report to the next shift. MULTIPLE RESPONSE 1. The nurse is testing the hearing of a 78-year-old man and is reminded of the changes in hearing that occur with aging that include which of the following? (Select all that apply.) a. Progression of hearing loss is slow. b. The aging person has low-frequency tone loss. c. Sounds may be garbled and difficult to localize. d. Hearing loss r/t aging begins in the mid-40s. e. Hearing loss reflects nerve degeneration of the middle ear. f. The aging person may find it harder to hear consonants than vowels. Chapter 17: Nose, Mouth, and Throat Jarvis: Physical Examination and Health Assessment, 8th Edition MULTIPLE CHOICE 1. What is the primary purpose of the ciliated mucous membrane in the nose? a. To warm the inhaled air b. To filter out dust and bacteria c. To filter coarse particles from inhaled air d. To facilitate the movement of air through the nares 2. What are the projections in the nasal cavity that increase the surface area are called? a. Meatus b. Septum c. Turbinates d. Kiesselbach plexus 3. The nurse is reviewing the development of the newborn infant. Regarding the sinuses, which statement is true in relation to a newborn infant? a. Sphenoid sinuses are full size at birth. b. Maxillary sinuses reach full size after puberty. c. Frontal sinuses are fairly well developed at birth. d. Maxillary and ethmoid sinuses are the only sinuses present at birth. 4. What is the tissue that connects the tongue to the floor of the mouth called? a. Uvula b. Palate c. Papillae d. Frenulum 5. In assessing the tonsils of a 30-year-old, the nurse notices that they are involuted, granular in appearance, and appear to have deep crypts. What is the correct response to these findings? a. Refer the patient to a throat specialist. b. No response is needed; this appearance is normal for the tonsils. c. Continue with the assessment, looking for any other abnormal findings. d. Obtain a throat culture on the patient for possible streptococcal (strep) infection. 6. The nurse is obtaining a health history on a 3-month-old infant. During the interview, the mother states, “I think she is getting her first tooth because she has started drooling a lot.” What is the best response by the nurse? a. “You’re right, drooling is usually a sign of the first tooth.” b. “It would be unusual for a 3-month-old to be getting her first tooth.” c. “This could be the sign of a problem with the salivary glands.” d. “She is just starting to salivate and hasn’t learned to swallow the saliva.” 7. The nurse is assessing an 80-year-old patient. Which of these findings would be expected for this patient? a. Hypertrophy of the gums b. Increased production of saliva c. Decreased ability to identify odors d. Finer and less prominent nasal hair 8. The nurse is performing an oral assessment on a 40-year-old black patient and notices the presence of a 1-cm, nontender, grayish-white lesion on the left buccal mucosa. Which one of these statements is true about this lesion? a. It is leukoedema which is common in dark-pigmented people. b. It is indicative of cancer and should be immediately tested. c. It is the result of hyperpigmentation and is a normal finding. d. It is torus palatinus and would normally be found only in smokers. 9. While obtaining a health history, a patient tells the nurse that he has frequent nosebleeds and asks the best way to get them to stop. What would be the nurse’s best response? a. “While sitting up, place a cold compress over your nose.” b. “Sit up with your head tilted forward and pinch your nose.” c. “Allow the bleeding to stop on its own, but don’t blow your nose.” d. “Lie on your back with your head tilted back and pinch your nose.” 10. A 92-year-old patient has had a stroke. The right side of his face is drooping. The nurse might also suspect which of these assessment findings? a. Epistaxis b. Rhinorrhea c. Dysphagia d. Xerostomia 11. While obtaining a health history from the mother of a 1-year-old child, the nurse notices that the baby has had a bottle in his mouth the entire time. The mother states, “It makes a great pacifier.” What is the best response by the nurse? a. “You’re right. Bottles make very good pacifiers.” b. “Using a bottle as a pacifier is better for the teeth than thumb-sucking.” c. “It’s okay to use a bottle as long as it contains milk and not juice.” d. “Prolonged use of a bottle can increase the risk for tooth decay and ear infections.” 12. A 72-year-old patient has a history of hypertension and chronic lung disease. Which is an important question for the nurse to include in this patient’s health history? a. “Do you use a fluoride supplement?” b. “Have you had tonsillitis in the last year?” c. “At what age did you get your first tooth?” d. “Have you noticed any dryness in your mouth?” 13. The nurse is using an otoscope to assess the nasal cavity. Which of these techniques is correct? a. Avoiding touching the nasal septum with the speculum b. Inserting the speculum at least 3 cm into the vestibule c. Gently displacing the nose to the side that is being examined d. Keeping the speculum tip medial to avoid touching the floor of the nares 14. The nurse is performing an assessment on a 21-year-old patient and notices that his nasal mucosa appears pale, gray, and swollen. What would be the most appropriate question to ask the patient? a. “Have you had any symptoms of a cold?” b. “Do you have an elevated temperature?” c. “Are you aware of having any allergies?” d. “Have you been having frequent nosebleeds?” 15. The nurse is palpating the sinus areas. If the findings are normal, then the patient should report which sensation? a. No sensation b. Firm pressure c. Pain during palpation d. Pain sensation behind eyes 16. During an oral assessment of a 30-year-old black patient, the nurse notices bluish lips and a dark line along the gingival margin. What action would the nurse perform in response to this finding? a. Check the patient’s Hb for anemia. b. Assess for other signs of insufficient oxygen supply. c. Proceed with the assessment, this appearance is a normal finding. d. Ask if he has been exposed to an excessive amount of carbon monoxide. 17. During an assessment of a 20-year-old patient with a 3-day history of nausea and vomiting, the nurse notices dry mucosa and deep vertical fissures in the tongue. What do these findings indicate? a. Dehydration b. A normal oral assessment c. Irritation from gastric juices d. Side effects from nausea medication 18. A 32-year-old woman is at the clinic for “little white bumps in my mouth.” During the assessment, the nurse notes that she has a 0.5-cm white, nontender papule under her tongue and one on the mucosa of her right cheek. What would the nurse tell the patient? a. “These spots indicate an infection such as strep throat.” b. “These bumps could be indicative of a serious lesion, so I will refer you to a specialist.” c. “This condition is called leukoplakia and can be caused by chronic irritation such as with smoking.” d. “These bumps are Fordyce granules, which are sebaceous cysts and are not a serious condition.” 19. A 10-year-old is at the clinic for “a sore throat that has lasted 6 days.” Which of these findings would be consistent with an acute infection? a. Tonsils 3+/1-4+ with pale coloring b. Tonsils 3+/1-4+ with large white spots c. Tonsils 2+/1-4+ with small plugs of white debris d. Tonsils 1+/1-4+ and pink; the same color as the oral mucosa 20. Immediately after birth, the nurse is unable to suction the nares of a crying newborn. An attempt is made to pass a catheter through both nasal cavities with no success. What should the nurse do next? a. Attempt to suction again with a bulb syringe. b. Wait a few minutes, and try again once the infant stops crying. c. Recognize that this situation requires immediate intervention. d. Contact the physician to schedule an appointment for the infant at his or her next hospital visit. 21. The nurse notices that the mother of a 2-year-old boy brings him into the clinic quite frequently for various injuries and suspects there may be some child abuse involved. What should the nurse look for during an inspection of this child’s mouth? a. Swollen, red tonsils b. Ulcerations on the hard palate c. Bruising on the buccal mucosa or gums d. Small yellow papules along the hard palate 22. The nurse is assessing a 3-year-old for “drainage from the nose.” On assessment, a purulent drainage that has a very foul odor is noted from the left naris and no drainage is observed from the right naris. The child is afebrile with no other symptoms. What should the nurse do next? a. Refer to the physician for an antibiotic order. b. Have the mother bring the child back in 1 week. c. Perform an otoscopic examination of the left nares. d. Tell the mother that this drainage is normal for a child of this age. 23. During an assessment of a 26-year-old for “a spot on my lip I think is cancer,” the clinic nurse notices a group of clear vesicles with an erythematous base around them located at the lip-skin border. The patient mentions that she just returned from Hawaii. What is the most appropriate action by the nurse? a. Tell the patient she needs to see a skin specialist. b. Discuss the benefits of having a biopsy performed on any unusual lesion. c. Tell the patient that these vesicles are indicative of herpes simplex I or cold sores and that they will heal in 4 to 10 days. d. Tell the patient that these vesicles are most likely the result of a riboflavin deficiency and discuss nutrition. 24. While performing an assessment of the mouth, the nurse notices that the patient has a 1-cm ulceration that is crusted with an elevated border and located on the outer third of the lower lip. What other information would be most important for the nurse to assess? a. Nutritional status b. When the patient first noticed the lesion c. Whether the patient has had a recent cold d. Whether the patient has had any recent exposure to sick animals 25. A pregnant woman states that she is concerned about her gums because she has noticed they are swollen and have started to bleed. What would be an appropriate response by the nurse? a. “Your condition is probably due to a vitamin C deficiency.” b. “I’m not sure what causes swollen and bleeding gums, but let me know if it’s not better in a few weeks.” c. “You need to make an appointment with your dentist as soon as possible to have this checked.” d. “Swollen and bleeding gums can be caused by a change in hormonal balance during pregnancy.” 26. A 40-year-old patient who has just finished chemotherapy for breast cancer tells the nurse that she is concerned about her mouth. During the assessment the nurse finds areas of buccal mucosa that are raw and red with some bleeding, as well as other areas that have a white, cheesy coating. What do these findings indicate? a. Candidiasis b. Leukoplakia c. Koplik spots d. Aphthous ulcers 27. The nurse is assessing a patient in the hospital who has received numerous antibiotics for a lung infection and notices that his tongue appears to be black and hairy. In response to his concern, what would the nurse say? a. “We will need to get a biopsy to determine the cause.” b. “This is an overgrowth of hair and will go away in a few days.” c. “Black, hairy tongue is a fungal infection caused by all the antibiotics you have received.” d. “This is probably caused by the same bacteria you had in your lungs.” 28. The nurse is assessing a patient with a history of intravenous drug abuse. In assessing his mouth, the nurse notices a dark red confluent macule on the hard palate. This could be an early sign of what disease or disorder? a. Measles b. Leukemia c. A carcinoma d. Acquired immunodeficiency syndrome (AIDS) 29. A mother brings her 4-month-old infant to the clinic with concerns regarding a small pad in the middle of the upper lip that has been there since 1 month of age. The infant has no health problems. On physical examination, the nurse notices a 0.5-cm, fleshy, elevated area in the middle of the upper lip. No evidence of inflammation or drainage is observed. What would the nurse tell this mother? a. “This area of irritation is caused from teething and is nothing to worry about.” b. “This finding is abnormal and should be evaluated by another health care provider.” c. “This area of irritation is the result of chronic drooling and should resolve within the next month or two.” d. “This elevated area is a sucking tubercle caused from the friction of breastfeeding or bottle-feeding and is normal.” 30. A mother is concerned because her 18-month-old toddler has 12 teeth. She is wondering if this is normal for a child of this age. Which is the best response by the nurse? a. “How many teeth did you have at this age?” b. “This is a normal number of teeth for an 18 month old.” c. “Normally, by age 2 years, 16 deciduous teeth are expected.” d. “All 20 deciduous teeth are expected to erupt by age 4 years.” 31. When examining the mouth of an older patient, the nurse recognizes which finding is due to the aging process? a. Teeth appearing shorter b. Tongue that looks smoother in appearance c. Buccal mucosa that is beefy red in appearance d. Small, painless lump on the dorsum of the tongue 32. When examining the nares of a 45-year-old patient who is experiencing rhinorrhea, itching of the nose and eyes, and sneezing, the nurse notices the following: pale turbinates, swelling of the turbinates, and clear rhinorrhea. Which of these conditions is most likely the cause? a. Nasal polyps b. Acute rhinitis c. Acute sinusitis d. Allergic rhinitis 33. When assessing the tongue of an adult, what finding would be considered abnormal? a. Smooth glossy dorsal surface b. Thin white coating over the tongue c. Raised papillae on the dorsal surface d. Visible venous patterns on the ventral surface 34. The nurse is performing an assessment. Which of these findings would cause the greatest concern? a. A painful vesicle inside the cheek for 2 days b. The presence of moist, nontender Stensen’s ducts c. Stippled gingival margins that snugly adhere to the teeth d. An ulceration on the side of the tongue with rolled edges 35. A patient has been diagnosed with strep throat. The nurse is aware that without treatment, which complication may occur? a. Rubella b. Leukoplakia c. Scarlet fever d. Rheumatic fever 36. During a checkup, a 22-year-old woman tells the nurse that she uses an over-the-counter nasal spray because of her allergies. She also states that it does not work as well as it used to when she first started using it. Which is the best response by the nurse? a. “You should never use over-the-counter nasal sprays because of the risk for addiction.” b. “You should try switching to another brand of medication to prevent this problem.” c. “Continuing to use this spray is important to keep your allergies under control.” d. “Frequent use of these nasal medications irritates the lining of the nose and may cause rebound swelling.” 37. During an oral examination of a 4-year-old American-Indian child, the nurse notices that her uvula is partially split. Which of these statements is accurate? a. A bifid uvula may occur in some American-Indian groups. b. This condition is a cleft palate and is common in American Indians. c. A bifid uvula is torus palatinus, which frequently occurs in American Indians. d. This condition is due to an injury and should be reported to the authorities. 38. A patient comes into the clinic complaining of facial pain, fever, and malaise. On examination, the nurse notes swollen turbinates and purulent discharge from the nose. The patient also complains of a dull, throbbing pain in his cheeks and teeth on the right side and pain when the nurse palpates the areas. What do these findings indicate? a. Nasal polyps b. Frontal sinusitis c. Posterior epistaxis d. Maxillary sinusitis 39. A woman who is in the second trimester of pregnancy mentions that she has had “more nosebleeds than ever” since she became pregnant. What is the likely reason for this? a. Inappropriate use of nasal sprays b. A problem with the patient’s coagulation system c. Increased susceptibility to colds and nasal irritation d. Increased vascularity in the upper respiratory tract as a result of the pregnancy MULTIPLE RESPONSE 1. During an assessment, a patient mentions that “I just can’t smell like I used to. I can barely smell the roses in my garden. Why is that?” For which possible causes of changes in the sense of smell will the nurse assess? (Select all that apply.) a. Aging b. Chronic allergies c. Cigarette smoking d. Chronic alcohol use e. Herpes simplex virus I f. Frequent episodes of strep throat Chapter 18: Breasts, Axillae, and Regional Lymphatics Jarvis: Physical Examination and Health Assessment, 8th Edition MULTIPLE CHOICE 1. Which of the following statements is true regarding the internal structures of the breast? a. Fibrous, glandular, and adipose tissues b. Primarily muscle with very little fibrous tissue c. Primarily milk ducts, known as lactiferous ducts d. Glandular tissue, which supports the breast by attaching to the chest wall 2. In performing a breast examination, the nurse knows that examining the upper outer quadrant of the breast is especially important. What is the reason for this? a. It is the largest quadrant of the breast. b. It is the most common location of breast tumors. c. It is where the majority of suspensory ligaments attach. d. It is more prone to injury and calcifications than other locations in the breast. 3. In performing an assessment of a woman’s axillary lymph system, the nurse should assess which of these nodes? a. Central, axillary, lateral, and sternal b. Pectoral, lateral, anterior, and sternal c. Central, lateral, pectoral, and subscapular d. Lateral, pectoral, axillary, and suprascapular 4. If a patient reports a recent breast infection, then the nurse should expect to find what type of node enlargement? a. Nonspecific b. Ipsilateral axillary c. Inguinal and cervical d. Contralateral axillary 5. A 9-year-old girl is in the clinic for a sport physical examination. After some initial shyness she finally asks, “Am I normal? I don’t seem to need a bra yet, but I have some friends who do. What if I never get breasts?” Which response by the nurse is best? a. “Don’t worry, you still have plenty of time to develop.” b. “I know just how you feel, I was a late bloomer myself. Just be patient, and they will grow.” c. “You will probably get your periods before you notice any significant growth in your breasts.” d. “I understand that it is hard to feel different from your friends. Breasts usually develop between 8 and 10 years of age.” 6. A patient contacts the office and tells the nurse that she is worried about her 10-year-old daughter having breast cancer. She describes a unilateral enlargement of the right breast with associated tenderness. She is worried because the left breast is not enlarged. Which response by the nurse to the mother is best? a. “Breast development is usually fairly symmetric your daughter should be examined right away.” b. “You should bring in your daughter right away because breast cancer is fairly common in preadolescent girls.” c. “Although an examination of your daughter would rule out a problem, her breast development is most likely normal.” d. “It is unusual for breasts that are first developing to feel tender because they haven’t developed much fibrous tissue.” 7. A 14-year-old girl is anxious about not having reached menarche. When taking the health history, the nurse should ascertain which of the following? a. The age her mother developed breasts b. The age she began to develop pubic hair c. The age she began developing axillary hair d. The age the girl began to develop breasts 8. A woman is in the family planning clinic seeking birth control information. She states that her breasts “change all month long” and that she is worried that this is unusual. What is the best response by the nurse? a. “Continual changes in your breasts are unusual. The breasts of nonpregnant women usually stay pretty much the same all month long.” b. “Breast changes in response to stress are very common and you should assess your life for stressful events.” c. “Because of the changing hormones during the monthly menstrual cycle, cyclic breast changes are common.” d. “Breast changes normally occur only during pregnancy. You should get a pregnancy test done as soon as possible.” 9. A woman has just learned that she is pregnant. What should the nurse teach this patient about changes in her breasts? a. She can expect her areolae to become larger and darker in color. b. Breasts may begin secreting milk after the fourth month of pregnancy. c. She should inspect her breasts for visible veins and immediately report these. d. During pregnancy, breast changes are fairly uncommon; most of the changes occur after the birth. 10. The nurse is teaching a pregnant woman about breast milk. Which statement by the nurse is correct? a. “Your breast milk is immediately present after the delivery of your baby.” b. “Breast milk is rich in protein and sugars (lactose) but has very little fat.” c. “The colostrum, which is present right after birth, does not contain the same nutrients as breast milk.” d. “You may notice a thick, yellow fluid expressed from your breasts as early as the fourth month of pregnancy.” 11. A 65-year-old patient remarks that she just cannot believe that her breasts “sag so much.” She states it must be from a lack of exercise. What explanation should the nurse offer her? a. After menopause, only women with large breasts experience sagging. b. After menopause, sagging is usually due to decreased muscle mass within the breast. c. After menopause, a diet that is high in protein will help maintain muscle mass, which keeps the breasts from sagging. d. After menopause, the glandular and fat tissue atrophies, causing breast size and elasticity to diminish, resulting in breasts that sag. 12. In examining a 70-year-old male patient, the nurse notices that he has bilateral gynecomastia. Which of the following describes the nurse’s best course of action? a. Recommend that he make an appointment with his physician for a mammogram. b. Acknowledge it as benign breast enlargement which is not unusual in men. c. Explain that this condition may be the result of hormonal changes, and recommend that he see his physician. d. Explain that gynecomastia in men is usually associated with prostate enlargement and recommend that he be thoroughly screened. 13. During an examination of a 7-year-old girl, the nurse notices that the girl is showing breast budding. What should the nurse do next? a. Nothing; breast budding is a normal finding. b. Ask the young girl if her periods have started. c. Assess the girl’s weight and body mass index (BMI). d. Ask the girl’s mother at what age she started to develop breasts. 14. The nurse is preparing for a class on breast cancer. Which statement is true with regard to cultural differences in breast cancer in the United States? a. Black women have a lower incidence of aggressive, triple negative breast cancer. b. The relative 5-year survival rate for black women is lower than that for Caucasian women. c. For every stage of breast cancer, Asian/Pacific Islander women have the lowest rate of survival. d. Ashkenazi Jewish women have a significantly lower prevalence of BRCA1 and BRCA2 gene mutations. 15. During a breast health interview, a patient states that she has noticed pain in her left breast. Which statement by the nurse is most appropriate? a. “Don’t worry about the pain; breast cancer is not painful.” b. “I would like some more information about the pain in your left breast.” c. “Oh, I had pain like that after my son was born; it turned out to be a blocked milk duct.” d. “Breast pain is almost always the result of benign breast disease.” 16. During a health history interview, a female patient states that she has noticed a few drops of clear discharge from her right nipple. What should the nurse do next? a. Ask her if she is possibly pregnant. b. Immediately contact the physician to report the discharge. c. Immediately obtain a sample for culture and sensitivity testing. d. Ask the patient some additional questions about the medications she is taking. 17. During a physical examination, a 45-year-old woman states that she has had a crusty, itchy rash on her breast for approximately 2 weeks. In trying to find the cause of the rash, which question would be important for the nurse to ask? a. “Is the rash raised and red?” b. “Does it appear to be cyclic?” c. “Where did the rash first appear—on the nipple, the areola, or the surrounding skin?” d. “What was she doing when she first noticed the rash, and do her actions make it worse?” 18. A patient is newly diagnosed with benign breast disease. The nurse recognizes which statement about benign breast disease to be true? a. It makes it more difficult to examine the breasts. b. It is easily reduced with hormone replacement therapy. c. It frequently turns into cancer in a woman’s later years. d. It is usually diagnosed before a woman reaches childbearing age. 19. During an annual physical examination, a 43-year-old patient states that she does not perform monthly breast self-examinations (BSEs). She tells the nurse that she believes that mammograms “do a much better job than I ever could to find a lump.” What should the nurse include in his or her response to this patient? a. BSEs may detect lumps that appear between mammograms. b. She is correct—mammography is a good replacement for BSE. c. The American Cancer Society recommends women over 40 years old perform a monthly BSE. d. She does not need to perform BSEs as long as a physician checks her breasts annually. 20. During an interview, a patient reveals that she is pregnant. She states that she is not sure whether she will breastfeed her baby and asks for some information about this. Which of these statements by the nurse is accurate? a. “Breastfed babies tend to be more colicky.” b. “Breastfed babies eat more often than infants on formula.” c. “Breastfeeding is second nature, and every woman can do it.” d. “Breastfeeding provides the perfect food and antibodies for your baby.” 21. The nurse is reviewing risk factors for breast cancer. Which of these women have risk factors that place them at a higher risk for breast cancer? a. 37-year-old who is slightly overweight b. 42-year-old who has had ovarian cancer c. 45-year-old who has never been pregnant d. 66-year-old whose mother had breast cancer 22. During an examination of a woman, the nurse notices that her left breast is slightly larger than her right breast. Which of these statements is true about this finding? a. Asymmetry of the breasts is unusual and the patient should be referred to physician. b. Asymmetry of the breasts is common, but the nurse should verify that this finding is not new. c. Asymmetry of breast size and shape is very unusual and means she may have an inflammation or growth. d. Asymmetry of breast size and shape is probably due to breastfeeding and is nothing to worry about. 23. The nurse is assisting with a BSE clinic. Which of these women reflects abnormal findings during the inspection phase of breast examination? a. Woman whose nipples are in different planes (deviated) b. Woman whose left breast is slightly larger than her right c. Nonpregnant woman whose skin is marked with linear striae d. Pregnant woman whose breasts have a fine blue network of veins visible under the skin 24. During the physical examination, the nurse notices that a female patient has an inverted left nipple. Which statement regarding this is most accurate? a. Normal nipple inversion is usually bilateral. b. Unilateral inversion of a nipple is always a serious sign. c. Whether the inversion is a recent change should be determined. d. Nipple inversion is not significant unless accompanied by an underlying palpable mass. 25. The nurse is performing a breast examination. Which of these statements best describes the correct procedure to use when screening for nipple and skin retraction during a breast examination? a. Have the patient bend over and touch her toes. b. Have the patient lie down on her left side and observe for any retraction. c. Have the patient shift from a supine position to a standing position, and note any lag or retraction. d. Have the patient slowly lift her arms above her head, and note any retraction or lag in movement. 26. The nurse is palpating a female patient’s breasts during an examination. Which of these positions is most likely to make significant lumps more distinct during breast palpation? a. Supine with the arms raised over her head b. Sitting with the arms relaxed at her sides c. Supine with the arms relaxed at her sides d. Sitting with the arms flexed and fingertips touching her shoulders 27. Which of these clinical situations would the nurse consider to be outside normal limits? a. A patient has had one pregnancy and states that she believes she may be entering menopause. Her breast examination reveals breasts that are soft and slightly sagging. b. A patient has never been pregnant. Her breast examination reveals large pendulous breasts that have a firm, transverse ridge along the lower quadrant in both breasts. c. A patient has never been pregnant and reports that she should begin her period tomorrow. Her breast examination reveals breast tissue that is nodular and somewhat engorged. She states that the examination was slightly painful. d. A patient has had two pregnancies, and she breastfed both of her children. Her youngest child is now 10 years old. Her breast examination reveals breast tissue that is somewhat soft, and she has a small amount of thick yellow discharge from both nipples. 28. A patient states during the interview that she noticed a new lump in the shower a few days ago. It was on her left breast near her axilla. How should the nurse proceed? a. Palpate the lump first. b. Palpate the unaffected breast first. c. Avoid palpating the lump because it could be a cyst, which might rupture. d. Palpate the breast with the lump first but plan to palpate the axilla last. 29. The nurse has palpated a lump in a female patient’s right breast. The nurse documents this as a small, round, firm, distinct, lump located at 2 o’clock, 2 cm from the nipple. It is nontender and fixed. No associated retraction of the skin or nipple, no erythema, and no axillary lymphadenopathy are observed. What information is missing from the documentation? a. Size of the lump b. Shape of the lump c. Consistency of the lump d. Whether the lump is solitary or multiple 30. The nurse is conducting a class on BSE. Which of these statements indicates the proper BSE technique? a. The best time to perform BSE is in the middle of the menstrual cycle. b. A woman should perform BSEs bimonthly unless she has fibrocystic breast tissue. c. The best time to perform a BSE is 4 to 7 days after the first day of the menstrual period. d. If she suspects that she is pregnant, then the woman should not perform a BSE until after her baby is born. 31. A 55-year-old postmenopausal woman is being seen in the clinic for her annual examination. She is concerned about changes in her breasts that she has noticed over the past 5 years. She states that her breasts have decreased in size and that the elasticity has changed so that her breasts seem “flat and flabby.” Which is the best reply by the nurse? a. “This change occurs most often because of long-term use of bras that do not provide enough support to the breast tissues.” b. “Decreases in hormones after menopause causes atrophy of the glandular tissue in the breast and is a normal process of aging.” c. “This is a normal change that occurs as women get older and is due to the increased levels of progesterone during the aging process.” d. “Postural changes in the spine make it appear that your breasts have changed in shape. Exercises to strengthen the muscles of the upper back and chest wall will help prevent the changes in elasticity and size.” 32. A 43-year-old woman is at the clinic for a routine examination. She reports that she has had a lump in her right breast for years. Recently, it has begun to change in consistency and is becoming harder. She reports that 5 years ago her physician evaluated the lump and determined that it was “nothing to worry about.” The nurse’s examination validates the presence of a mass in the right upper outer quadrant at 1 o’clock, approximately 5 cm from the nipple. It is firm, mobile, and nontender, with borders that are not well defined. What is the best response by the nurse? a. “Because of the change in consistency of the lump, it should be further evaluated by a physician.” b. “The changes could be r/t your menstrual cycles. Keep track of the changes in the mass each month.” c. “The lump is probably nothing to worry about because it has been present for years and was determined to be noncancerous 5 years ago.” d. “Because you are experiencing no pain and the size has not changed, you should continue to monitor the lump and return to the clinic in 3 months.” 33. While inspecting a patient’s breasts, the nurse finds that the left breast is slightly larger than the right with the bilateral presence of Montgomery glands. How should the nurse proceed? a. Palpate over the Montgomery glands, checking for drainage. b. Consider these findings as normal, and proceed with the examination. c. Ask extensive health history questions regarding the woman’s breast asymmetry. d. Continue with the examination, and then refer the patient for further evaluation of the Montgomery glands. 34. During an examination, the nurse notes a supernumerary nipple just under the patient’s left breast. The patient tells the nurse that she always thought it was a mole. Which statement about this finding is correct? a. This variation is normal and not a significant finding. b. This finding is significant and needs further investigation. c. A supernumerary nipple also contains glandular tissue and may leak milk during pregnancy and lactation. d. The patient is correct—a supernumerary nipple is actually a mole that happens to be located under the breast. 35. While examining a 75-year-old woman, the nurse notices that the skin over her right breast is thickened and the hair follicles are exaggerated. What is this condition called? a. Dimpling b. Retraction c. Peau d’orange d. Benign breast disease 36. A new mother calls the clinic to report that part of her left breast is red, swollen, tender, very hot, and hard. She has a fever of 38.3 C. She also has had symptoms of influenza, such as chills, sweating, and feeling tired. The nurse notices that she has been breastfeeding for 1 month. From her description, what condition does the nurse suspect? a. Mastitis b. Paget disease c. Plugged milk duct d. Mammary duct ectasia 37. During a breast examination on a female patient, the nurse notices that the nipple is flat, broad, and fixed. The patient states it “started doing that a few months ago.” What does this finding suggest? a. Dimpling b. Retracted nipple c. Nipple inversion d. Deviation in nipple pointing 38. A 54-year-old man comes to the clinic with a “horrible problem.” He tells the nurse that he has just discovered a lump on his breast and is fearful of cancer. The nurse knows which statement about breast cancer in men is true? a. Breast cancer in men rarely spreads to the lymph nodes. b. Less than one percent of all breast cancers occurs in men. c. Most breast masses in men are diagnosed as gynecomastia. d. Breast masses in men are difficult to detect because of minimal breast tissue. MULTIPLE RESPONSE 1. The nurse is assessing the breasts of a 68-year-old woman and discovers a mass in the upper outer quadrant of the left breast. When assessing this mass, the nurse is aware that characteristics of a cancerous mass include which of the following? (Select all that apply.) a. Nontender mass b. Regular border c. Hard, dense, and immobile d. Rubbery texture and mobile e. Dull, heavy pain on palpation f. Irregular, poorly delineated border 2. The nurse is examining a 62-year-old man and notes that he has bilateral gynecomastia. The nurse should explore his health history for which related conditions? (Select all that apply.) a. Malnutrition b. Liver disease c. Hyperthyroidism d. Type 2 diabetes mellitus e. History of alcohol abuse Chapter 19: Thorax and Lungs Jarvis: Physical Examination and Health Assessment, 8th Edition MULTIPLE CHOICE 1. Which of these statements is true regarding the vertebra prominens? a. It is the spinous process of C7. b. It is nonpalpable in most individuals. c. It is opposite the interior border of the scapula. d. It is located next to the manubrium of the sternum. 2. When performing a respiratory assessment on a patient, the nurse notices a costal angle of approximately 90 degrees. What should the nurse recognize about this finding? a. Observed in patients with kyphosis. b. Indicative of pectus excavatum. c. A normal finding in a healthy adult. d. An expected finding in a patient with a barrel chest. 3. When assessing a patient’s lungs, what should the nurse recall about the left lung? a. Consists of two lobes. b. Is divided by the horizontal fissure. c. Primarily consists of an upper lobe on the posterior chest. d. Is shorter than the right lung because of the underlying stomach. 4. Which statement about the apices of the lungs is true? a. Are at the level of the second rib anteriorly. b. Extend 3 to 4 cm above the inner third of the clavicles. c. Are located at the sixth rib anteriorly and the eighth rib laterally. d. Rest on the diaphragm at the fifth intercostal space in the midclavicular line (MCL). 5. Where does the trachea bifurcate on the anterior chest? a. Costal angle b. Sternal angle c. Xiphoid process d. Suprasternal notch 6. During an assessment, the nurse knows that expected assessment findings in the normal adult lung include which findings? a. Adventitious sounds and limited chest expansion b. Muffled voice sounds and symmetric tactile fremitus c. Increased tactile fremitus and dull percussion tones d. Absent voice sounds and hyperresonant percussion tones 7. What are the primary muscles of respiration? a. Diaphragm and intercostals b. Sternomastoids and scaleni c. Trapezii and rectus abdominis d. External obliques and pectoralis major 8. A 65-year-old patient with a history of heart failure comes to the clinic stating “I keep waking up from sleep with shortness of breath.” Which action by the nurse is most appropriate? a. Obtain a detailed health history of the patient’s allergies and a history of asthma. b. Tell the patient to sleep on his or her right side to facilitate ease of respirations. c. Assess for other signs and symptoms of paroxysmal nocturnal dyspnea. d. Assure the patient that paroxysmal nocturnal dyspnea is normal and will probably resolve within the next week. 9. When assessing tactile fremitus, the nurse recalls that it is normal to feel tactile fremitus most intensely over which location? a. Between the scapulae b. Third intercostal space, MCL c. Over the lower lobes, posterior side d. Fifth intercostal space, midaxillary line (MAL) 10. The nurse is reviewing the technique of palpating for tactile fremitus with a new graduate. Which statement by the graduate nurse reflects a correct understanding of tactile fremitus? a. “Is caused by moisture in the alveoli.” b. “Is caused by sounds generated from the larynx.” c. “Reflects the blood flow through the pulmonary arteries.” d. “Indicates that air is present in the subcutaneous tissues.” 11. During percussion, the nurse hears a dull percussion note elicited over a lung lobe. What is the most likely cause of this finding? a. Shallow breathing b. Normal lung tissue c. Decreased adipose tissue d. Increased density of lung tissue 12. The nurse is observing the auscultation technique of a student nurse. What is the correct method to use when progressing from one auscultatory site on the thorax to another? a. Side-to-side comparison b. Top-to-bottom comparison c. Posterior-to-anterior comparison d. Interspace-by-interspace comparison 13. When auscultating the lungs of an adult patient, the nurse notes that low-pitched, soft breath sounds are heard over the posterior lower lobes, with inspiration being longer than expiration. How should the nurse interpret these findings? a. Normal sounds auscultated over the trachea. b. Bronchial breath sounds that are normal in that location. c. Vesicular breath sounds that are normal in that location. d. Bronchovesicular breath sounds that are normal in that location. 14. The nurse is auscultating the chest in an adult. Which technique is correct? a. Instructing the patient to take deep, rapid breaths b. Instructing the patient to breathe in and out through his or her nose c. Firmly holding the diaphragm of the stethoscope against the skin of the chest d. Lightly holding the bell of the stethoscope against the skin on the chest to avoid friction 15. The nurse is percussing over the lungs of a patient with pneumonia. If the patient has atelectasis, what sound will the nurse hear? a. Tympany b. Dullness c. Resonance d. Hyperresonance 16. During auscultation of the lungs, the nurse expects decreased breath sounds to be heard in which situation? a. When adventitious sounds are present b. When the bronchial tree is obstructed c. In conjunction with whispered pectoriloquy d. In conditions of consolidation, such as pneumonia 17. Which is a normal finding when assessing the respiratory system of an older adult? a. Increased thoracic expansion b. Decreased mobility of the thorax c. Decreased anteroposterior diameter d. Bronchovesicular breath sounds throughout the lungs 18. A mother brings her 3-month-old infant to the clinic for evaluation of a cold. She tells the nurse that he has had “a runny nose for a week.” When performing the physical assessment, the nurse notes that the child has nasal flaring and sternal and intercostal retractions. What should the nurse do next? a. Ask the mother if the infant has had trouble with feedings. b. Assure the mother that these signs are normal symptoms of a cold. c. Recognize that these are serious signs, and contact the physician. d. Perform a complete cardiac assessment because these signs are probably indicative of early heart failure. 19. When assessing the respiratory system of a 4-year-old child, which of these findings would the nurse expect? a. Crepitus palpated at the costochondral junctions b. Presence of bronchovesicular breath sounds in the peripheral lung fields c. No diaphragmatic excursion as a result of a child’s decreased inspiratory volume d. Irregular respiratory pattern and a respiratory rate of 40 breaths per minute at rest 20. When inspecting the anterior chest of an adult, the nurse should include which assessment? a. Diaphragmatic excursion b. Symmetric chest expansion c. Presence of breath sounds d. Shape and configuration of the chest wall 21. The nurse would most likely hear fine crackles in which patient or situation? a. A pregnant woman b. A healthy 5-year-old child c. The immediate newborn period d. A patient with a pneumothorax 22. During an assessment of an adult, the nurse has noted unequal chest expansion and recognizes that this occurs in which situation? a. In an obese patient b. When part of the lung is obstructed or collapsed c. When bulging of the intercostal spaces is present d. When accessory muscles are used to augment respiratory effort 23. During auscultation of the lungs of an adult patient, the nurse notices the presence of bronchophony. The nurse should assess for signs of which condition? a. Asthma b. Emphysema c. Airway obstruction d. Pulmonary consolidation 24. The nurse is reviewing the characteristics of breath sounds. Which statement about bronchovesicular breath sounds is true? a. Musical in quality b. Expected near the major airways c. Usually caused by a pathologic disease d. Similar to bronchial sounds except shorter in duration 25. The nurse is listening to the breath sounds of a patient with severe asthma. Air passing through narrowed bronchioles would produce which of these adventitious sounds? a. Wheezes b. Bronchophony c. Bronchial sounds d. Whispered pectoriloquy 26. A patient has a long history of chronic obstructive pulmonary disease (COPD). During the assessment, the nurse will most likely observe which of these? a. Unequal chest expansion b. Increased tactile fremitus c. Atrophied neck and trapezius muscles d. Anteroposterior-to-transverse diameter ratio of 1:1 27. A teenage patient comes to the emergency department with complaints of an inability to breathe and a sharp pain in the left side of his chest. The assessment findings include cyanosis, tachypnea, tracheal deviation to the right, decreased tactile fremitus on the left, hyperresonance on the left, and decreased breath sounds on the left. What do these findings suggest? a. Bronchitis b. Pneumothorax c. Acute pneumonia d. Asthmatic attack 28. An adult patient with a history of allergies comes to the clinic complaining of wheezing and difficulty in breathing when working in his yard. The assessment findings include tachypnea, the use of accessory neck muscles, prolonged expiration, intercostal retractions, decreased breath sounds, and expiratory wheezes. What do these findings suggest? a. Asthma b. Atelectasis c. Lobar pneumonia d. Heart failure 29. The nurse is assessing the lungs of an older adult. Which of these changes are normal in the respiratory system of the older adult? a. Decrease in small airway closure occurs, leading to problems with atelectasis. b. Severe dyspnea is experienced on exertion, resulting from changes in the lungs. c. Respiratory muscle strength increases to compensate for a decreased vital capacity. d. Lungs are less elastic and distensible, which decreases their ability to collapse and recoil. 30. A woman in her 26th week of pregnancy states that she is “not really short of breath” but feels that she is aware of her breathing and the need to breathe. What is the best reply by the nurse? a. “The diaphragm becomes fixed during pregnancy, making it difficult to take in a deep breath.” b. “The increase in estrogen levels during pregnancy often causes a decrease in the diameter of the rib cage and makes it difficult to breathe.” c. “What you are experiencing is normal. Some women may interpret this as shortness of breath, but it is a normal finding and nothing is wrong.” d. “This increased awareness of the need to breathe is normal as the fetus grows because of the increased oxygen demand on the mother’s body, which results in an increased respiratory rate.” 31. A 35-year-old recent immigrant is being seen in the clinic for symptoms of a cough associated with rust-colored sputum, low-grade afternoon fevers, and night sweats for the past 2 months. Based on these findings, what is the most likely cause? a. Pneumonia b. Bronchitis c. Tuberculosis d. Pulmonary edema 32. A 70-year-old patient is being seen in the clinic for severe exacerbation of his heart failure. Which of these findings is the nurse most likely to observe in this patient? a. Fever, dry nonproductive cough, and diminished breath sounds b. Rasping cough, thick mucoid sputum, wheezing, and bronchitis c. Productive cough, dyspnea, weight loss, anorexia, and tuberculosis d. Shortness of breath, orthopnea, paroxysmal nocturnal dyspnea, and ankle edema 33. A patient comes to the clinic reporting a cough that is worse at night but not as bad during the day. What does the nurse suspect? a. Pneumonia b. Postnasal drip or sinusitis c. Exposure to irritants at work d. Chronic bronchial irritation from smoking 34. During a morning assessment, the nurse notices that the patient’s sputum is frothy and pink. Which condition could this finding indicate? a. Croup b. Tuberculosis c. Viral infection d. Pulmonary edema 35. During auscultation of breath sounds, the nurse should correctly use the stethoscope in which of the following ways? a. Listening to at least one full respiration in each location b. Listening as the patient inhales and then going to the next site during exhalation c. If the patient is modest, listening to sounds over his or her clothing or hospital gown d. Instructing the patient to breathe in and out rapidly while listening to the breath sounds 36. A patient has been admitted to the emergency department with a possible medical diagnosis of pulmonary embolism. The nurse expects to see which assessment findings related to this condition? a. Absent or decreased breath sounds b. Productive cough with thin, frothy sputum c. Chest pain that is worse on deep inspiration and dyspnea d. Diffuse infiltrates with areas of dullness upon percussion 37. During palpation of the anterior chest wall, the nurse notices a coarse, crackling sensation over the skin surface. What does this finding indicate? a. Crepitus b. Friction rub c. Tactile fremitus d. Adventitious sounds 38. The nurse is auscultating the lungs of a patient who had been sleeping and notices short, popping, crackling sounds that stop after a few breaths. What does this finding indicate? a. Fine wheezes b. Vesicular breath sounds c. Fine crackles and may be a sign of pneumonia d. Atelectatic crackles that do not have a pathologic cause 39. A patient has been admitted to the emergency department for a suspected drug overdose. His respirations are shallow, with an irregular pattern, at a rate of 12 respirations per minute. The nurse interprets this respiration pattern as which of the following? a. Bradypnea b. Hypoventilation c. Cheyne-Stokes respirations d. Chronic obstructive breathing 40. A patient with pleuritis has been admitted to the hospital and reports pain with breathing. What other key assessment finding would the nurse expect to find upon auscultation? a. Stridor b. Crackles c. Wheezing d. Friction rub MULTIPLE RESPONSE 1. The nurse is assessing voice sounds during a respiratory assessment. Which of these findings indicates a normal assessment? (Select all that apply). a. As the patient says a long “ee-ee-ee” sound, the examiner hears a long “aaaaaa” sound. b. As the patient says a long “ee-ee-ee” sound, the examiner also hears a long “ee-ee-ee” sound. c. As the patient repeatedly says “ninety-nine,” the examiner clearly hears the words “ninety-nine.” d. Voice sounds are faint, muffled, and almost inaudible when the patient whispers “one, two, three” in a very soft voice. e. When the patient speaks in a normal voice, the examiner can hear a sound but cannot exactly distinguish what is being said. Chapter 20: Heart and Neck Vessels Jarvis: Physical Examination and Health Assessment, 8th Edition MULTIPLE CHOICE 1. What is the sac that surrounds and protects the heart is called? a. Myocardium b. Pericardium c. Endocardium d. Pleural space 2. The direction of blood flow through the heart is best described by which of these? a. Vena cava  right atrium  right ventricle  lungs  pulmonary artery  left atrium  left ventricle b. Right atrium  right ventricle  pulmonary artery  lungs  pulmonary vein  left atrium  left ventricle c. Aorta  right atrium  right ventricle  lungs  pulmonary vein  left atrium  left ventricle  vena cava d. Right atrium  right ventricle  pulmonary vein  lungs  pulmonary artery  left atrium  left ventricle 3. The nurse is reviewing the anatomy and physiologic functioning of the heart. Which statement best describes what is meant by atrial kick? a. The atria contract during systole and attempt to push against closed valves. b. Contraction of the atria at the beginning of diastole can be felt as a palpitation. c. Atrial kick is the pressure exerted against the atria as the ventricles contract during systole. d. The atria contract toward the end of diastole and push the remaining blood into the ventricles. 4. When listening to heart sounds, which valve closures are heard best at the base of the heart? a. Aortic and pulmonic b. Mitral and pulmonic c. Mitral and tricuspid d. Tricuspid and aortic 5. Which of these statements describes the closure of the valves in a normal cardiac cycle? a. The pulmonic valve closes slightly before the aortic valve. b. The aortic valve closes slightly before the tricuspid valve. c. Both the tricuspid and pulmonic valves close at the same time. d. The tricuspid valve closes slightly later than the mitral valve. 6. What component of the conduction system is referred to as the pacemaker of the heart? a. Bundle of His b. Bundle branches c. Sinoatrial (SA) node d. Atrioventricular (AV) node 7. Which sequence does the electrical stimulus of the cardiac cycle follow? a. AV node  SA node  bundle of His b. Bundle of His  AV node  SA node c. SA node  AV node  bundle of His  bundle branches d. AV node  SA node  bundle of His  bundle branches 8. The findings from an assessment of a 70-year-old patient with swelling in his ankles include jugular venous pulsations 5 cm above the sternal angle when the head of his bed is elevated 45 degrees. What does this finding indicate? a. Decreased fluid volume b. Increased cardiac output c. Narrowing of jugular veins d. Elevated pressure r/t heart failure 9. A 25-year-old woman in her fifth month of pregnancy has a blood pressure of 100/70 mm Hg. In reviewing her previous examination, the nurse notes that her blood pressure in her second month was 124/80 mm Hg. In evaluating this change, what does the nurse know to be true? a. This decline in blood pressure is the result of peripheral vasodilatation and is an expected change. b. Because of increased cardiac output, the blood pressure should be higher at this time. c. This change in blood pressure is not an expected finding because it means a decrease in cardiac output. d. This decline in blood pressure means a decrease in circulating blood volume, which is dangerous for the fetus. 10. In assessing a 70-year-old man, the nurse finds the following: blood pressure 140/100 mm Hg; heart rate 104 beats per minute and slightly irregular; and the split S2 heart sound. Which of these findings can be explained by the expected hemodynamic changes r/t age? a. Increase in resting heart rate b. Increase in systolic blood pressure c. Decrease in diastolic blood pressure d. Increase in diastolic blood pressure 11. A 45-year-old man is in the clinic for a routine physical examination. During the recording of his health history, the patient states that he has been having difficulty sleeping. “I’ll be sleeping great, and then I wake up and feel like I can’t get my breath.” Which is the best response by the nurse? a. “When was your last electrocardiogram?” b. “It’s probably because it’s been so hot at night.” c. “Do you have any history of problems with your heart?” d. “Have you had a recent sinus infection or upper respiratory infection?” 12. In assessing a patient’s major risk factors for heart disease, which would the nurse want to include when taking a history? a. Family history, hypertension, stress, and age b. Personality type, high cholesterol, diabetes, and smoking c. Smoking, hypertension, obesity, diabetes, and high cholesterol d. Alcohol consumption, obesity, diabetes, stress, and high cholesterol 13. The mother of a 3-month-old infant states that her baby has not been gaining weight. With further questioning, the nurse finds that the infant falls asleep after nursing and wakes up after a short-time hungry again. What other information would the nurse want to have? a. Infant’s sleeping position b. Sibling history of eating disorders c. Amount of background noise when eating d. Presence of dyspnea or diaphoresis when sucking 14. What should the nurse do when assessing the carotid arteries of an older patient with cardiovascular disease? a. Palpate the artery in the upper one-third of the neck. b. Simultaneously palpate both arteries to compare amplitude. c. Listen with the bell of the stethoscope to assess for bruits. d. Instruct the patient to take slow deep breaths during auscultation. 15. During an assessment of a 68-year-old man with a recent onset of right-sided weakness, the nurse hears a blowing, swishing sound with the bell of the stethoscope over the left carotid artery. What does this finding indicate? a. Valvular disorder b. Blood flow turbulence c. Fluid volume overload d. Ventricular hypertrophy 16. During an inspection of the precordium of an adult patient, the nurse notices the chest moving in a forceful manner along the sternal border. What does this finding most likely indicate? a. Systolic murmur b. Diastolic murmur c. Enlargement of the left ventricle d. Enlargement of the right ventricle 17. During an assessment of a healthy adult, where would the nurse expect to palpate the apical impulse? a. Third left intercostal space at the midclavicular line b. Fourth left intercostal space at the sternal border c. Fourth left intercostal space at the anterior axillary line d. Fifth left intercostal space at the midclavicular line 18. The nurse is examining a patient who has possible cardiac enlargement. Which statement about percussion of the heart is true? a. Percussion is easier in patients who are obese. b. Percussion is a useful tool for outlining the heart’s borders. c. Only expert health care providers should attempt percussion of the heart. d. Studies show that percussed cardiac borders do not correlate well with the true cardiac border. 19. The nurse is preparing to auscultate for heart sounds. Which technique is correct? a. Listening for all possible sounds at a time at each specified area. b. Listening to the sounds at the aortic, tricuspid, pulmonic, and mitral areas. c. Listening to the sounds only at the site where the apical pulse is felt to be the strongest. d. Listening by inching the stethoscope in a rough Z pattern, from the base of the heart across and down, then over to the apex. 20. While counting the apical pulse on a 16-year-old patient, the nurse notices an irregular rhythm. His rate speeds up on inspiration and slows on expiration. What should the nurse do? a. Document this as a normal finding. b. Talk with the patient about his intake of caffeine. c. Perform an electrocardiogram after the examination. d. Refer the patient to a cardiologist for further testing. 21. What is the best description of the S1 heart sound? a. Indicates the beginning of diastole. b. Coincides with the carotid artery pulse. c. Louder than the S2 at the base of the heart. d. Is caused by the closure of the semilunar valves. 22. During cardiac auscultation, the nurse hears a sound immediately occurring after the S2 at the second left intercostal space. What should the nurse do to further assess this sound? a. Ask the patient to hold his or her breath while the nurse listens again. b. No further assessment is needed because the nurse knows this sound is an S3. c. Watch the patient’s respirations while listening for the effect on the sound. d. Have the patient turn to the left side while the nurse listens with the bell of the stethoscope. 23. Which of these findings would the nurse expect to notice during a cardiac assessment on a 4-year-old child? a. S3 when sitting up b. Persistent tachycardia above 150 beats per minute c. Murmur at the second left intercostal space when supine d. Palpable apical impulse in the fifth left intercostal space lateral to midclavicular line 24. While auscultating heart sounds on a 7-year-old child for a routine physical examination, the nurse hears an S3, a soft murmur at the left midsternal border, and a venous hum when the child is standing. What would be a correct interpretation of these findings? a. These findings can all be normal in a child. b. An S3 is indicative of heart disease in children. c. The venous hum most likely indicates an aneurysm. d. These findings are indicative of congenital problems. 25. During the precordial assessment of a patient who is 8 months pregnant, the nurse palpates the apical impulse at the fourth left intercostal space lateral to the midclavicular line. What does this finding indicate? a. Right ventricular hypertrophy b. Increased blood flow through the internal mammary artery c. Displacement of the heart from elevation of the diaphragm d. Increased volume and size of the heart as a result of pregnancy 26. In assessing for an S4 heart sound, what part of the stethoscope should the nurse use and in what location? a. Bell of the stethoscope at the base with the patient leaning forward b. Diaphragm of the stethoscope in the aortic area with the patient sitting c. Diaphragm of the stethoscope in the pulmonic area with the patient supine d. Bell of the stethoscope at the apex with the patient in the left lateral position 27. A 70-year-old patient with a history of hypertension has a blood pressure of 180/100 mm Hg and a heart rate of 90 beats per minute. The nurse hears an extra heart sound at the apex immediately before the S1. The sound is heard only with the bell of the stethoscope while the patient is in the left lateral position. Based on these findings and the patient’s history, the nurse should recognize this extra heart sound is most likely what? a. Split S1 b. Atrial gallop c. Diastolic murmur d. Summation sound 28. The nurse is performing a cardiac assessment on a 65-year-old patient 3 days after her myocardial infarction (MI). Heart sounds are normal when she is supine, but when she is sitting and leaning forward, the nurse hears a high-pitched, scratchy sound with the diaphragm of the stethoscope at the apex. It disappears on inspiration. What does the nurse suspect? a. Another MI b. Increased cardiac output c. Inflammation of the precordium d. Ventricular hypertrophy resulting from muscle damage 29. The mother of a 10-month-old infant tells the nurse that she has noticed that her son becomes blue when he is crying and that the frequency of this is increasing. He is also not crawling yet. During the examination the nurse palpates a thrill at the left lower sternal border and auscultates a loud systolic murmur in the same area. What would be the most likely cause of these findings? a. Tetralogy of Fallot b. Atrial septal defect c. Patent ductus arteriosus d. Ventricular septal defect 30. A 30-year-old woman with a history of mitral valve problems states that she has been “very tired.” She has started waking up at night and feels like her “heart is pounding.” During the assessment, the nurse palpates a thrill and lift at the fifth left intercostal space midclavicular line. In the same area, the nurse also auscultates a blowing, swishing sound right after the S1. What do these findings most likely indicate? a. Heart failure b. Aortic stenosis c. Pulmonary edema d. Mitral regurgitation 31. During a cardiac assessment on a 38-year-old patient in the hospital for “chest pain,” the nurse finds the following: jugular vein pulsations 4 cm above the sternal angle when the patient is elevated at 45 degrees, blood pressure 98/60 mm Hg, heart rate 130 beats per minute, ankle edema, difficulty breathing when supine, and an S3 on auscultation. Which of these conditions best explains the cause of these findings? a. Heart failure b. Fluid overload c. Atrial septal defect d. Myocardial infarction 32. What is the normal splitting of the S2 is associated with? a. Expiration b. Inspiration c. Exercise state d. Low resting heart rate 33. The nurse is preparing to teach a class on cardiovascular assessment. When explaining a thrill, what should the nurse include in the teaching? a. A vibration that is palpable b. Palpated in the right epigastric area c. Associated with ventricular hypertrophy d. A murmur auscultated at the third intercostal space 34. When performing a cardiovascular assessment, what should the nurse understand about an S4 heart sound? a. Heard at the onset of atrial diastole b. Often a normal finding in the older adult c. Heard at the end of ventricular diastole d. Heard best over the second left intercostal space with the individual sitting upright 35. During an assessment, the nurse notes that the patient’s apical impulse is laterally displaced and is palpable over a wide area. What does this finding indicate? a. Systemic hypertension b. Pulmonic hypertension c. Pressure overload, as in aortic stenosis d. Volume overload, as in heart failure 36. When the nurse is auscultating the carotid artery for bruits, which of these statements reflects the correct technique? a. While listening with the bell of the stethoscope, the patient is asked to take a deep breath and hold it. b. While auscultating one side with the bell of the stethoscope, the carotid artery is palpated on the other side to check pulsations. c. While lightly applying the bell of the stethoscope over the carotid artery and listening, the patient is asked to take a breath, exhale, and briefly hold it. d. While firmly placing the bell of the stethoscope over the carotid artery and listening, the patient is asked to take a breath, exhale, and briefly hold it. 37. The nurse is preparing for a class on risk factors for hypertension and reviews recent statistics. Which racial group has the highest prevalence of hypertension in the world? a. Blacks b. Whites c. Hispanics d. American Indians 38. The nurse is assessing a patient with possible cardiomyopathy and assesses the hepatojugular reflux. If heart failure is present, then the nurse should recognize which finding while pushing on the right upper quadrant of the patient’s abdomen, just below the rib cage? a. The jugular veins will not be detected during this maneuver. b. The jugular veins will remain elevated as long as pressure on the abdomen is maintained. c. An impulse will be visible at the fourth or fifth intercostal space at or inside the midclavicular line. d. The jugular veins will rise for a few seconds and then recede back to the previous level if the heart is properly working. 39. The nurse is assessing the apical pulse of a 3-month-old infant and finds that the heart rate is 135 beats per minute. How should the nurse interpret this finding? a. Normal for this age b. Lower than expected c. Higher than expected, probably as a result of crying d. Higher than expected, reflecting persistent tachycardia Chapter 21: Peripheral Vascular System and Lymphatic System Jarvis: Physical Examination and Health Assessment, 8th Edition MULTIPLE CHOICE 1. Which statement is true regarding the arterial system? a. Arteries are large-diameter vessels. b. The arterial system is a high-pressure system. c. The walls of arteries are thinner than those of the veins. d. Arteries can greatly expand to accommodate a large blood volume increase. 2. The nurse is reviewing the blood supply to the arm. What major artery supplies blood to the arm? a. Ulnar b. Radial c. Brachial d. Deep palmar 3. The nurse is preparing to assess the dorsalis pedis artery. Where is the correct location for palpation? a. Behind the knee b. Over the lateral malleolus c. In the groove behind the medial malleolus d. Lateral to the extensor tendon of the great toe 4. A 65-year-old patient is experiencing pain in his left calf when he exercises which disappears after resting for a few minutes. What problem in the left leg does this indicate? a. Venous obstruction b. Partial blockage of an artery c. Claudication due to venous abnormalities d. Ischemia caused by the complete blockage of an artery 5. The nurse is reviewing venous blood flow patterns. Which of these statements best describes the mechanism(s) by which venous blood returns to the heart? a. Intraluminal valves ensure unidirectional flow toward the heart. b. Contracting skeletal muscles milk blood distally toward the veins. c. High-pressure system of the heart helps facilitate venous return. d. Increased thoracic pressure and decreased abdominal pressure facilitate venous return to the heart. 6. Which vein(s) is(are) responsible for most of the venous return in the arm? a. Deep b. Ulnar c. Subclavian d. Superficial 7. A 70-year-old patient is scheduled for open-heart surgery. The surgeon plans to use the great saphenous vein for the coronary bypass grafts. The patient asks, “What happens to my circulation when this vein is removed?” How should the nurse reply? a. “Venous insufficiency is a common problem after this type of surgery.” b. “Oh, you have lots of veins—you won’t even notice that it has been removed.” c. “You will probably experience decreased circulation after the vein is removed.” d. “This vein can be removed without harming your circulation because the deeper veins in your leg are in good condition.” 8. The nurse is reviewing the risk factors for venous disease. Which of these situations best describes a person at highest risk for the development of venous disease? a. Person who has been on bed rest for 4 days b. Older adult taking anticoagulant medication c. Woman in the second month of her first pregnancy d. Person with a 30-year, 1 pack per day smoking habit 9. The nurse is teaching a review class on the lymphatic system. Which statement by a class participant indicates correct understanding of the material? a. “Lymph flow is propelled by the contraction of the heart.” b. “The flow of lymph is slow, compared with that of the blood.” c. “One of the functions of the lymph is to absorb lipids from the biliary tract.” d. “Lymph vessels have no valves; therefore, lymph fluid flows freely from the tissue spaces into the bloodstream.” 10. When performing an assessment of a patient, the nurse notices the presence of an enlarged right epitrochlear lymph node. What should the nurse do next? a. Assess the patient’s abdomen, and notice any tenderness. b. Carefully assess the cervical lymph nodes, and check for any enlargement. c. Ask additional health history questions regarding any recent ear infections or sore throats. d. Examine the patient’s lower arm and hand, and check for the presence of infection or lesions. 11. A 35-year-old man is seen in the clinic for an infection in his left foot. Which of these findings should the nurse expect to see during an assessment of this patient? a. Hard and fixed cervical nodes b. Enlarged and tender inguinal nodes c. Bilateral enlargement of the popliteal nodes d. Pelletlike nodes in the supraclavicular region 12. The nurse is examining the lymphatic system of a healthy 3-year-old child. Which finding should the nurse expect? a. Palpable superficial lymph nodes b. Excessive swelling of the lymph nodes c. No palpable nodes because of the immature immune system of a child d. Fewer and smaller sized lymph nodes compared with those of an adult 13. During an assessment of an older adult, the nurse should expect to which finding as a normal physiologic change associated with the aging process? a. Hormonal changes causing vasodilation and a resulting drop in blood pressure b. Progressive atrophy of the intramuscular calf veins, causing venous insufficiency c. Peripheral blood vessels growing more rigid with age, producing a rise in systolic blood pressure d. Narrowing of the inferior vena cava, causing low blood flow and increases in venous pressure resulting in varicosities 14. A 67-year-old patient states that he recently began to have pain in his left calf when climbing the 10 stairs to his apartment. This pain is relieved by sitting for “a couple of minutes”; then he is able to resume his activities. What do these symptoms suggest? a. Claudication b. Sore muscles c. Muscle cramps d. Venous insufficiency 15. A patient complains of leg pain that wakes him at night. He states that he “has been having problems” with his legs. He has pain in his legs when they are elevated that disappears when he dangles them. He recently noticed “a sore” on the outer aspect of the right ankle. What do these findings suggest? a. Pain r/t lymphatic abnormalities b. Problems r/t venous insufficiency c. Problems r/t arterial insufficiency d. Pain r/t musculoskeletal abnormalities 16. The nurse uses the profile sign during an assessment. What does this technique detect? a. Barrel chest b. Early clubbing c. Symmetry of the fingers d. Insufficient capillary refill 17. The nurse is performing an assessment on an adult. The adult’s vital signs are normal, and capillary refill time is 5 seconds. What should the nurse do next? a. Ask the patient about a history of frostbite. b. Suspect that the patient has venous insufficiency. c. Consider this a delayed capillary refill time, and investigate further. d. Consider this a normal capillary refill time that requires no further assessment. 18. When performing a peripheral vascular assessment on a patient, the nurse is unable to palpate the ulnar pulses. The patient’s skin is warm and capillary refill time is normal. What should the nurse do next? a. Check for the presence of claudication. b. Refer the individual for further evaluation. c. Consider this finding normal, and proceed with the peripheral vascular evaluation. d. Ask the patient if he or she has experienced any unusual cramping or tingling in the arm. 19. The nurse is assessing the pulses of a patient who has been admitted for untreated hyperthyroidism. When assessing this patient’s pulse, what should the nurse expect? a. Normal b. Bounding c. Weak, thready d. Unpalpable pedal pulse 20. The nurse is preparing to perform a modified Allen test. Which is an appropriate reason for this test? a. To measure the rate of lymphatic drainage b. To evaluate the adequacy of capillary patency before venous blood draws c. To evaluate the adequacy of collateral circulation before cannulating the radial artery d. To evaluate the venous refill rate that occurs after the ulnar and radial arteries are temporarily occluded 21. A patient has been diagnosed with venous stasis. Which of these findings would the nurse most likely observe? a. Unilateral cool foot b. Thin, shiny, atrophic skin c. Pallor of the toes and cyanosis of the nail beds d. Brownish discoloration to the skin of the lower leg 22. The nurse is attempting to assess the femoral pulse in a patient who is obese. Which of these actions would be most appropriate? a. Ask the patient to assume a prone position. b. Ask the patient to bend his or her knees to the side in a froglike position. c. The nurse firmly presses against the bone with the patient in a semi-Fowler position. d. The nurse listens with a stethoscope for pulsations; palpating the pulse in an obese person is extremely difficult. 23. When auscultating over a patient’s femoral arteries, the nurse notices the presence of a bruit on the left side. Which statement about bruits is accurate? a. Often associated with venous disease b. Occur in the presence of lymphadenopathy c. Femoral artery bruits are caused by hypermetabolic states d. Occur with turbulent blood flow, indicating partial occlusion 24. How should the nurse document mild, slight pitting edema the ankles of a pregnant patient? a. 1+/0-4+ b. 3+/0-4+ c. 4+/0-4+ d. Brawny edema 25. A patient has hard, nonpitting edema of the left lower leg and ankle. The right leg has no edema. When interpreting these findings, what should the nurse recall? a. Alterations in arterial function will cause edema. b. Nonpitting, hard edema occurs with lymphatic obstruction. c. Phlebitis of a superficial vein will cause bilateral edema. d. Long-standing arterial obstruction will cause pitting edema. 26. When assessing a patient’s pulse, the nurse notes that the amplitude is weaker during inspiration and stronger during expiration. When the nurse measures the blood pressure, the reading decreases 20 mm Hg during inspiration and increases with expiration. What do these findings indicate? a. Pulsus alternans b. Pulsus bisferiens c. Pulsus bigeminus d. Pulsus paradoxus 27. During an assessment, the nurse elevated a patient’s legs 12 inches off the table and had him wag his feet to drain off venous blood. After helping him sit up and dangle his legs over the side of the table, what should the nurse expect for a normal finding? a. Significant elevational pallor b. Venous filling within 15 seconds c. No change in the coloration of the skin d. Color returning to the feet within 20 seconds of assuming a sitting position 28. During a visit to the clinic, a woman in her seventh month of pregnancy states that her legs feel “heavy in the calf” and that she often has foot cramps at night. The nurse notices that the patient has dilated, tortuous veins apparent in her lower legs. Which condition is reflected by these findings? a. Lymphedema b. Varicose veins c. Raynaud phenomenon d. Deep vein thrombophlebitis 29. During an assessment, the nurse notices that a patient’s left arm is swollen from the shoulder down to the fingers, with nonpitting brawny edema. The right arm is normal. The patient had a left-sided mastectomy 1 year ago. Based on these findings, what does the nurse suspect? a. Lymphedema b. Venous stasis c. Arteriosclerosis d. Deep-vein thrombosis 30. The nurse is preparing to assess the ankle-brachial index (ABI) of a patient. Which statement about the ABI is true? a. Normal ABI indices are from 0.5 to 1.0. b. Normal ankle pressure is slightly lower than the brachial pressure. c. The ABI is a reliable measurement of peripheral vascular disease in individuals with diabetes. d. An ABI of 0.9 to 0.7 indicates the presence of peripheral vascular disease and mild claudication. 31. The nurse is performing a well-child checkup on a 5-year-old boy. The child has no current condition that would lead the nurse to suspect an illness. His health history is unremarkable, and he received immunizations 1 week ago. Which of these findings should be considered normal in this patient? a. Enlarged, warm, and tender nodes b. Lymphadenopathy of the cervical nodes c. Palpable firm, small, shotty, mobile, and nontender lymph nodes d. Firm, rubbery, and large nodes, somewhat fixed to the underlying tissue 32. When using a Doppler ultrasonic stethoscope, the nurse recognizes venous flow when which sound is heard? a. Low humming sound b. Swishing, whooshing sound c. Regular “lub, dub” pattern d. Steady, even, flowing sound 33. The nurse is describing a weak, thready pulse on the documentation flow sheet. Which statement is correct? a. “Easily palpable; pounds under the fingertips.” b. “Greater than normal force that suddenly collapses.” c. “Hard to palpate, may fade in and out, and is easily obliterated by pressure.” d. “Rhythm is regular, but force varies with alternating beats of large and small amplitude.” 34. During an assessment, a patient tells the nurse that her fingers often change color when she goes out in cold weather. She describes these episodes as her fingers first turning white, then blue, then red with a burning, throbbing pain. What does the nurse suspect? a. Lymphedema b. Raynaud phenomenon c. Deep-vein thrombosis d. Chronic arterial insufficiency 35. During a routine office visit, a patient takes off his shoes and shows the nurse “this awful sore that won’t heal.” On inspection, the nurse notes a 3-cm round ulcer on the left great toe, with a pale ischemic base, well-defined edges, and no drainage. Based on these findings, what does the nurse suspect? a. Varicosities b. Venous stasis ulcer c. Arterial ischemic ulcer d. Deep vein thrombophlebitis 36. The nurse is reviewing an assessment of a patient’s peripheral pulses and notices that the documentation states that the radial pulses are “2+.” The nurse recognizes that this reading indicates what type of pulse? a. Weak b. Absent c. Normal d. Bounding MULTIPLE RESPONSE 1. A patient is recovering from several hours of orthopedic surgery. During an assessment of the patient’s lower legs, the nurse will monitor for signs of acute venous symptoms. Signs of acute venous symptoms include which of the following? (Select all that apply.) a. Sudden onset b. Warm, red, and swollen calf c. Pain that is worse at the end of the day d. Aching, tired pain, with a feeling of fullness e. Pain that is relieved with elevation of the leg. f. Intense, sharp pain, with the deep muscle tender to the touch 2. A patient has been admitted with chronic arterial symptoms. During the assessment, the nurse should expect which findings? (Select all that apply.) a. Skin of the patient is pale and cool. b. His ankles have two small, weeping ulcers. c. He states that the pain gets worse when walking. d. Patient works long hours sitting at a computer desk. e. Patient has a history of diabetes and cigarette smoking. f. Patient states that the pain is worse at the end of the day. Chapter 22: Abdomen Jarvis: Physical Examination and Health Assessment, 8th Edition MULTIPLE CHOICE 1. The nurse is percussing the seventh right intercostal space at the midclavicular line over the liver. Which sound should the nurse expect to hear? a. Tympany b. Dullness c. Resonance d. Hyperresonance 2. Which structure is located in the left lower quadrant of the abdomen? a. Liver b. Duodenum c. Gallbladder d. Sigmoid colon 3. A patient is having difficulty swallowing medications and food. How should the nurse document this? a. Aphasia b. Anorexia c. Dysphasia d. Dysphagia 4. The nurse suspects that a patient has a distended bladder. How should the nurse assess for this condition? a. Percuss and palpate in the lumbar region b. Inspect and palpate in the epigastric region c. Auscultate and percuss in the inguinal region d. Percuss and palpate the midline area above the suprapubic bone 5. The nurse is aware that what change may occur in the gastrointestinal system with aging? a. Increased salivation b. Increased liver size c. Increased esophageal emptying d. Decreased gastric acid secretion 6. A 22-year-old man comes to the clinic for an examination after falling off his motorcycle and landing on his left side on the handle bars. The nurse suspects that he may have injured his spleen. Which of these statements is true regarding assessment of the spleen in this situation? a. The spleen can be enlarged as a result of trauma. b. The spleen is normally felt on routine palpation. c. If an enlarged spleen is noted, then the nurse should thoroughly palpate to determine its size. d. An enlarged spleen should not be palpated because it can easily rupture. 7. A patient’s abdomen is bulging and stretched in appearance. How should the nurse document this finding? a. Obese b. Scaphoid c. Herniated d. Protuberant 8. The nurse is describing a scaphoid abdomen. When assessing the contour of the abdomen from the rib margin to the pubic bone, what would the contour look like? a. Flat b. Convex c. Bulging d. Concave 9. While examining a patient, the nurse observes abdominal pulsations between the xiphoid process and umbilicus. What does the nurse suspect? a. Pulsations of the renal arteries b. Normal abdominal aortic pulsations c. Pulsations of the inferior vena cava d. Increased peristalsis from a bowel obstruction 10. A patient has hypoactive bowel sounds. What is a possible cause of this finding? a. Diarrhea b. Peritonitis c. Laxative use d. Gastroenteritis 11. The nurse is watching a new graduate nurse perform auscultation of a patient’s abdomen. Which statement by the new graduate shows a correct understanding of the reason auscultation precedes percussion and palpation of the abdomen? a. “We need to determine the areas of tenderness before using percussion and palpation.” b. “Auscultation prevents distortion of bowel sounds that might occur after percussion and palpation.” c. “Auscultation allows the patient more time to relax and therefore be more comfortable with the physical examination.” d. “Auscultation prevents distortion of vascular sounds, such as bruits and hums, that might occur after percussion and palpation.” 12. The nurse is listening to bowel sounds. Which of these statements is true of bowel sounds? a. Sound like two pieces of leather being rubbed together b. Are usually high-pitched, gurgling, and irregular sounds c. Are usually loud, high-pitched, rushing, and tinkling sounds d. Originate from the movement of air and fluid through the large intestine 13. The physician comments that a patient has abdominal borborygmi. What is the best description of this term? a. Hypoactive bowel sounds b. A peritoneal friction rub c. Loud gurgling bowel sounds d. Loud continual humming bowel sounds 14. During an abdominal assessment, the nurse would consider which of these findings as normal? a. Presence of a bruit in the femoral area b. Tympanic percussion note in the umbilical region c. Dull percussion note in the left upper quadrant at the midclavicular line d. Palpable spleen between the ninth and eleventh ribs in the left midaxillary line 15. The nurse is assessing the abdomen of a pregnant woman who states she has been having “acid indigestion” all the time. What does the nurse know that esophageal reflux during pregnancy can cause? a. Diarrhea b. Pyrosis c. Dysphagia d. Constipation 16. The nurse is performing an abdominal assessment. What types of percussion notes can be heard during abdominal assessment? a. Flatness, resonance, and dullness b. Resonance, dullness, and tympany c. Tympany, hyperresonance, and dullness d. Resonance, hyperresonance, and flatness 17. An older patient has been diagnosed with pernicious anemia. This disorder could be r/t what condition? a. Increased gastric acid secretion b. Decreased gastric acid secretion c. Delayed gastrointestinal emptying time d. Increased gastrointestinal emptying time 18. A patient is reporting sharp pain along the costovertebral angles. What does this symptom most often indicate? a. Ovarian infection b. Liver enlargement c. Spleen enlargement d. Kidney inflammation 19. A nurse notices that a patient has abdominal ascites. What does this finding indicate? a. Flatus b. Fibroid tumors c. Presence of feces d. Presence of fluid 20. The nurse notices that a patient has had a black, tarry stool. What should the nurse recognize may cause this finding? a. Gallbladder disease b. Overuse of laxatives c. Gastrointestinal bleeding d. Localized bleeding around the anus 21. During an abdominal assessment, the nurse elicits tenderness on light palpation in the right lower quadrant. The nurse recognizes this finding could indicate a problem with what structure? a. Spleen b. Sigmoid c. Appendix d. Gallbladder 22. The nurse is assessing the abdomen of an older adult. Which statement regarding the older adult and abdominal assessment is true? a. Abdominal tone is increased. b. Abdominal musculature is thinner. c. Abdominal rigidity with an acute abdominal condition is more common. d. The older adult with an acute abdominal condition complains more about pain than the younger person. 23. During an assessment of a newborn infant, the nurse suspects the infant has pyloric stenosis. What finding would cause the nurse to suspect this? a. Projectile vomiting b. Hypoactive bowel activity c. Palpable olive-sized mass in the right lower quadrant d. Pronounced peristaltic waves crossing from right to left 24. The nurse is reviewing the assessment of an aortic aneurysm. Which of these statements is true regarding an aortic aneurysm? a. A bruit is absent. b. Femoral pulses are increased. c. A pulsating mass is usually present. d. Most are located below the umbilicus. 25. During an abdominal assessment, the nurse is unable to hear bowel sounds in a patient’s abdomen. How long should the nurse listen before reporting absent bowel sounds? a. 1 minute b. 5 minutes c. 10 minutes d. 2 minutes in each quadrant 26. A patient is suspected of having inflammation of the gallbladder, or cholecystitis. The nurse should conduct which of these techniques to assess for this condition? a. Obturator test b. Test for Murphy sign c. Iliopsoas muscle test d. Assess for rebound tenderness 27. Just before going home, a new mother asks the nurse about the infant’s umbilical cord. Which of these statements is correct? a. “It should fall off in 10 to 14 days.” b. “It will soften before it falls off.” c. “It contains two veins and one artery.” d. “Skin will cover the area within 1 week.” 28. Which of these percussion findings would the nurse expect to find in a patient with a large amount of ascites? a. Dullness across the abdomen b. Flatness in the right upper quadrant c. Hyperresonance in the left upper quadrant d. Tympany in the right and left lower quadrants 29. A 40-year-old man states that his physician told him that he has a hernia. He asks the nurse to explain what a hernia is. Which response by the nurse is appropriate? a. “No need to worry. Most men your age develop hernias.” b. “A hernia is a loop of bowel protruding through a weak spot in the abdominal muscles.” c. “A hernia is the result of prenatal growth abnormalities that are just now causing problems.” d. “I’ll have to have your physician explain this to you.” 30. A 45-year-old man is in the clinic for a physical examination. During the abdominal assessment, the nurse percusses the abdomen and notices an area of dullness above the right costal margin of approximately 11 cm. How should the nurse proceed? a. Document the presence of hepatomegaly. b. Ask additional health history questions regarding his alcohol intake. c. Consider this finding as normal, and proceed with the examination. d. Describe this dullness as indicative of an enlarged liver, and refer him to a physician. 31. When palpating the abdomen of a 20-year-old patient, the nurse notices the presence of tenderness in the left upper quadrant with deep palpation. Which of these structures is most likely to be involved? a. Spleen b. Appendix c. Gallbladder d. Sigmoid colon 32. The nurse is reviewing information on lactose intolerance and learned that in some racial groups lactase activity (ability to digest and absorb lactose) is high at birth but declines to low levels by adulthood. Which ethnic group has the highest potential for lactose-intolerance symptoms in adulthood? a. Asians b. African Americans c. White Americans d. American Indians 33. The nurse is assessing a patient for possible peptic ulcer disease. Which condition or history often causes this disorder? a. Hypertension b. Streptococcal infections c. Recurrent constipation with frequent laxative use d. Frequent use of nonsteroidal antiinflammatory drugs 34. During the change-of-shift report, the student nurse hears that a patient has hepatomegaly. What should the student recognizes that this term means? a. Enlarged liver b. Enlarged spleen c. Distended bowel d. Excessive diarrhea 35. During an assessment, the nurse notices that a patient’s umbilicus is enlarged and everted. It is positioned midline with no change in skin color. The nurse recognizes that the patient may have which condition? a. Constipation b. Abdominal tumor c. Umbilical hernia d. Intra-abdominal bleeding 36. During an abdominal assessment, the nurse tests for a fluid wave. What condition would produce a positive fluid wave test? a. Ascites b. Splenomegaly c. Constipation d. Distended bladder 37. The nurse is preparing to examine a patient who has been complaining of right lower quadrant pain. Which technique is correct during the assessment? a. Examine the tender area first. b. Examine the tender area last. c. Avoid palpating the tender area. d. Palpate the tender area first, and then auscultate for bowel sounds. 38. During a health history, the patient tells the nurse, “I have pain all the time in my stomach. It’s worse 2 hours after I eat, but it gets better if I eat again!” Based on these symptoms, the nurse suspects that the patient has which condition? a. Appendicitis b. Gastric ulcer c. Duodenal ulcer d. Cholecystitis MULTIPLE RESPONSE 1. The nurse suspects that a patient has appendicitis. Which of these procedures are appropriate for use when assessing for appendicitis or a perforated appendix? (Select all that apply.) a. Test for fluid wave b. Test for the Murphy sign c. Test for the Blumberg sign d. Test for shifting dullness e. Perform the iliopsoas muscle test Chapter 23: Musculoskeletal System Jarvis: Physical Examination and Health Assessment, 8th Edition MULTIPLE CHOICE 1. A patient is being assessed for range-of-joint movement. The nurse asks him to move his arm in toward the center of his body. What is this movement called? a. Flexion b. Abduction c. Adduction d. Extension 2. A patient tells the nurse that she is having a hard time bringing her hand to her mouth when she eats or tries to brush her teeth. The nurse knows that for her to move her hand to her mouth, she must perform which movement? a. Flexion b. Abduction c. Adduction d. Extension 3. What are the functional units of the musculoskeletal system? a. Bones b. Joints c. Muscles d. Tendons 4. When reviewing the musculoskeletal system, the nurse should recall that hematopoiesis takes place where? a. Liver b. Spleen c. Kidneys d. Bone marrow 5. What are the fibrous bands that run directly from one bone to another, strengthen the joint, and help prevent movement in undesirable directions called? a. Bursa b. Tendons c. Cartilage d. Ligaments 6. The nurse notices that a woman in an exercise class is unable to do one-person jump rope. What does the nurse know that the shoulder must be able to do in order for one to be able to do one-person jump rope? a. Inversion b. Supination c. Protraction d. Circumduction 7. What is the articulation of the mandible and the temporal bone called? a. Intervertebral foramen b. Condyle of the mandible c. Temporomandibular joint d. Zygomatic arch of the temporal bone 8. To palpate the temporomandibular joint, where should the nurse place his or her fingers? a. The depression inferior to the tragus of the ear b. The depression superior to the tragus of the ear c. The depression anterior to the tragus of the ear d. The depression posterior to the tragus of the ear 9. Of the 33 vertebrae in the spinal column, which is correct? a. 5 lumbar b. 5 thoracic c. 7 sacral d. 12 cervical 10. If an imaginary line were drawn connecting the highest point on each iliac crest. What vertebra would that line cross? a. First sacral b. Fourth lumbar c. Seventh cervical d. Twelfth thoracic 11. The nurse is explaining to a patient that there are shock absorbers in his back to cushion the spine and to help it move. What is the nurse referring to as shock absorbers? a. Vertebral column b. Nucleus pulposus c. Vertebral foramen d. Intervertebral disks 12. The nurse is providing patient education for a man who has been diagnosed with a rotator cuff injury. When explaining the structures involved in his injury, what should the nurse include? a. Nucleus pulposus b. Medial epicondyle c. Glenohumeral joint d. Articular processes 13. During an interview the patient states, “I can feel this bump on the top of both of my shoulders—it doesn’t hurt but I am curious about what it might be.” What should the nurse tell this patient? a. “That is the subacromial bursa.” b. “That is the acromion process.” c. “That is the glenohumeral joint.” d. “That is the greater tubercle of the humerus.” 14. The nurse is checking the range of motion in a patient’s knee and knows that the knee is capable of which movement(s)? a. Circumduction b. Flexion and extension c. Inversion and eversion d. Supination and pronation 15. A patient is visiting the clinic for an evaluation of a swollen, painful knuckle. The nurse notices that the knuckle above his ring on the left hand is swollen and that he is unable to remove his wedding ring. What is the name of this patient’s affected joint? a. Tibiotalar b. Interphalangeal c. Tarsometatarsal d. Metacarpophalangeal 16. The nurse is assessing a patient’s ischial tuberosity. How should the nurse position the patient to palpate the ischial tuberosity? a. Standing b. Flexing the hip c. Flexing the knee d. Lying in the supine position 17. The nurse is examining the hip area of a patient and palpates a flat depression on the upper, lateral side of the thigh when the patient is standing. What is the nurse palpating? a. Iliac crest b. Ischial tuberosity c. Greater trochanter d. Gluteus maximus muscle 18. What is articulated with the tibia and fibula in the ankle joint? a. Talus b. Cuboid c. Calcaneus d. Cuneiform bones 19. A woman who is 8 months pregnant comments that she has noticed a change in her posture and is having lower back pain. The nurse tells her that during pregnancy, women have a posture shift to compensate for the enlarging fetus. What is the term for this shift in posture? a. Lordosis b. Scoliosis c. Ankylosis d. Kyphosis 20. An 85-year-old patient comments during his annual physical examination that he seems to be getting shorter as he ages. Why does height decrease with aging? a. The vertebral column shortens. b. Long bones tend to shorten with age. c. A significant loss of subcutaneous fat occurs. d. A thickening of the intervertebral disks develops. 21. A patient has been diagnosed with osteoporosis and asks the nurse, “What is osteoporosis?” What is the best explanation by the nurse? a. “It is the loss of bone density.” b. “It is an increase in bone matrix.” c. “It is new bone growth that is weaker.” d. “There is a decrease in phagocytic activity.” 22. The nurse is teaching a class on preventing osteoporosis to a group of perimenopausal women. Which of these actions is the best way to prevent or delay bone loss in this group? a. Assessing bone density annually b. Taking medications to prevent osteoporosis c. Performing physical activity, such as fast walking d. Taking 800 mg calcium and 200 IU vitamin D supplements daily 23. A teenage girl has arrived reporting pain in her left wrist. She was playing basketball when she fell and landed on her left hand. The nurse examines her hand. Which finding would lead the nurse to expect a fracture? a. Dull ache b. Deep pain in her wrist c. Sharp pain that increases with movement d. Dull throbbing pain that increases with rest 24. A patient is reporting pain in his joints that is worse in the morning, better after he moves around for a while, and then gets worse again if he sits for long periods. The nurse should assess for other signs of what problem? a. Tendinitis b. Osteoarthritis c. Rheumatoid arthritis d. Intermittent claudication 25. A patient states, “I can hear a crunching or grating sound when I kneel.” She also states that “it is very difficult to get out of bed in the morning because of stiffness and pain in my joints.” The nurse should assess for signs of what problem? a. Bone spur b. Tendonitis c. Crepitation d. Fluid in the knee joint 26. A patient is able to flex his right arm forward without difficulty or pain but is unable to abduct his arm because of pain and muscle spasms. What does the nurse suspect? a. Crepitation b. Rheumatoid arthritis c. Rotator cuff lesions d. A dislocated shoulder 27. A professional tennis player comes into the clinic complaining of a sore elbow. Where should the nurse assess for tenderness? a. Olecranon bursa b. Annular ligament c. Base of the radius d. Medial and lateral epicondyle 28. The nurse suspects that a patient has carpal tunnel syndrome and wants to perform the Phalen test. What instructions should the nurse give the patient to perform this test? a. Dorsiflex the foot. b. Plantarflex the foot. c. Hold both hands back to back while flexing the wrists 90 degrees for 60 seconds. d. Hyperextend the wrists with the palmar surface of both hands touching, and wait for 60 seconds. 29. An 80-year-old woman is visiting the clinic for a checkup. She states, “I can’t walk as much as I used to.” What should the nurse have the patient do to observe for motor dysfunction in her hip? a. Internally rotate her hip while she is sitting. b. Abduct her hip while she is lying on her back. c. Adduct her hip while she is lying on her back. d. Externally rotate her hip while she is standing. 30. The nurse has completed the musculoskeletal examination of a patient’s knee and has found a positive bulge sign. How does the nurse interpret this finding? a. Irregular bony margins b. Soft-tissue swelling in the joint c. Swelling from fluid in the epicondyle d. Swelling from fluid in the suprapatellar pouch 31. During an examination, the nurse asks a patient to bend forward from the waist and notices that the patient has lateral tilting. When his leg is raised straight up, the patient states pain going down his buttock into his leg. What does the nurse suspect? a. Scoliosis b. Meniscus tear c. Herniated nucleus pulposus d. Spasm of paravertebral muscles 32. The nurse is examining a 3-month-old infant. While the nurse holds his or her thumbs on the infant’s inner mid thighs and the fingers on the outside of the infant’s hips, touching the greater trochanter, the nurse adducts the legs until the his or her thumbs touch and then abducts the legs until the infant’s knees touch the table. The nurse does not notice any “clunking” sounds. How should the nurse document this finding? a. Positive Allis test b. Negative Allis test c. Positive Ortolani sign d. Negative Ortolani sign 33. During a neonatal examination, the nurse notices that the newborn infant has six toes. How should the nurse document this finding? a. Unidactyly b. Syndactyly c. Polydactyly d. Multidactyly 34. A mother brings her newborn baby boy in for a checkup; she tells the nurse that he does not seem to be moving his right arm as much as his left and that he seems to have pain when she lifts him up under the arms. The nurse suspects a fractured clavicle. What finding would support this suspicion? a. Negative Allis test b. Positive Ortolani sign c. Limited range of motion during Lasègue test d. Limited range of motion during the Moro reflex 35. A 40-year-old man has come into the clinic reporting extreme pain in his toes. The nurse notices that his toes are slightly swollen, reddened, and warm to the touch. What does the nurse suspect? a. Acute gout b. Osteoporosis c. Ankylosing spondylitis d. Degenerative joint disease 36. A young swimmer comes to the sports clinic complaining of a very sore shoulder. He was running at the pool, slipped on some wet concrete, and tried to catch himself with his outstretched hand. He landed on his outstretched hand and has not been able to move his shoulder since. What does the nurse suspect? a. Joint effusion b. Tear of rotator cuff c. Adhesive capsulitis d. Dislocated shoulder 37. A 68-year-old woman has come in for an assessment of her rheumatoid arthritis, and the nurse notices raised, firm, nontender nodules at the olecranon bursa and along the ulna. What is the appropriate term for these nodules? a. Epicondylitis b. Gouty arthritis c. Olecranon bursitis d. Subcutaneous nodules. 38. A woman who has had rheumatoid arthritis for years is starting to notice that her fingers are drifting to the side. What is term commonly used for this condition? a. Radial drift b. Ulnar deviation c. Swan-neck deformity d. Dupuytren contracture 39. A patient who has had rheumatoid arthritis for years comes to the clinic to ask about changes in her fingers. The nurse will assess for signs of what problems? a. Heberden nodes b. Bouchard nodules c. Swan-neck deformities d. Dupuytren contractures 40. A patient’s annual physical examination reveals a lateral curvature of the thoracic and lumbar segments of his spine; however, this curvature disappears with forward bending. What is this abnormality called? a. Dislocated hip b. Structural scoliosis c. Functional scoliosis d. Herniated nucleus pulposus 41. A 14-year-old boy who has been diagnosed with Osgood-Schlatter disease reports painful swelling just below the knee for the past 5 months. Which response by the nurse is appropriate? a. “If these symptoms persist, you may need arthroscopic surgery.” b. “You are experiencing degeneration of your knee, which may not resolve.” c. “Your disease is due to repeated stress on the patellar tendon. It is usually self-limited, and your symptoms should resolve with rest.” d. “Increasing your activity and performing knee-strengthening exercises will help decrease the inflammation and maintain mobility in the knee.” 42. When assessing muscle strength, the nurse observes that a patient has complete range of motion against gravity with full resistance. What grade of muscle strength should the nurse record using a 0- to 5-point scale? a. 2 b. 3 c. 4 d. 5 43. The nurse is examining a 6-month-old infant and places the infant’s feet flat on the table and flexes his knees up. The nurse notes that the right knee is significantly lower than the left. Which of these statements is true of this finding? a. This finding is a positive Allis sign and suggests hip dislocation. b. The infant probably has a dislocated patella on the right knee. c. This finding is a negative Allis sign and normal for an infant of this age. d. The infant should return to the clinic in 2 weeks to see if his condition has changed. 44. The nurse is assessing a 1-week-old infant and is testing his muscle strength. The nurse lifts the infant with hands under the axillae and notices that the infant starts to “slip” between the hands. What does the nurse suspect? a. A fractured clavicle b. Possible deformity of the spine c. Weakness of the shoulder muscles d. This is a normal finding for an infant at this age 45. The nurse is examining a 2-month-old infant and notices asymmetry of the infant’s gluteal folds. The nurse should assess for other signs of what disorder? a. Spina bifida b. Down syndrome c. Hip dislocation d. Fractured clavicle 46. The nurse should use which test to check for large amounts of fluid around the patella? a. Tinel sign b. Phalen test c. McMurray test d. Ballottement 47. A patient tells the nurse that, “All my life I’ve been called ‘knock knees’.” What is medical term for this condition? a. Genu varum b. Pes planus c. Genu valgum d. Metatarsus adductus 48. A man who has had gout for several years comes to the clinic with a problem with his toe. On examination, the nurse notices the presence of hard, painless nodules over the great toe; one has burst open with a chalky discharge. What is this called? a. Tophi b. Callus c. Bunion d. Plantar wart 49. When performing a musculoskeletal assessment, what is the correct approach? a. Proximal to distal b. Distal to proximal c. Posterior to anterior d. Anterior to posterior MULTIPLE RESPONSE 1. The nurse is assessing the joints of a woman who has stated, “I have a long family history of arthritis, and my joints hurt.” The nurse suspects that she has osteoarthritis. Which of these are symptoms of osteoarthritis? (Select all that apply.) a. Symmetric joint involvement b. Asymmetric joint involvement c. Pain with motion of affected joints d. Affected joints may have heat, redness, and swelling e. Affected joints are swollen with hard, bony protuberances Chapter 24: Neurologic System Jarvis: Physical Examination and Health Assessment, 8th Edition MULTIPLE CHOICE 1. What are the two parts of the nervous system? a. Motor and sensory b. Central and peripheral c. Peripheral and autonomic d. Hypothalamus and cerebral 2. The wife of a 65-year-old man tells the nurse that she is concerned because she has noticed a change in her husband’s personality and ability to understand. He also cries very easily and becomes angry. What part of the cerebral lobe is responsible for these behaviors? a. Frontal b. Parietal c. Occipital d. Temporal 3. Which statement concerning the areas of the brain is true? a. The cerebellum is the center for speech and emotions. b. The hypothalamus controls body temperature and regulates sleep. c. The basal ganglia are responsible for controlling voluntary movements. d. Motor pathways of the spinal cord and brainstem synapse in the thalamus. 4. The area of the nervous system that responsible for mediating reflexes? a. Medulla b. Cerebellum c. Spinal cord d. Cerebral cortex 5. While gathering equipment after an injection, a nurse accidentally received a prick from an improperly capped needle. To interpret this sensation, which of these areas must be intact? a. Corticospinal tract, medulla, and basal ganglia b. Pyramidal tract, hypothalamus, and sensory cortex c. Lateral spinothalamic tract, thalamus, and sensory cortex d. Anterior spinothalamic tract, basal ganglia, and sensory cortex 6. A patient with a lack of oxygen to his heart will have pain in his chest and possibly in the shoulder, arms, or jaw. The nurse knows that the best explanation why this occurs is which one of these statements? a. A problem exists with the sensory cortex and its ability to discriminate the location. b. The lack of oxygen in his heart has resulted in decreased amount of oxygen to the areas experiencing the pain. c. The sensory cortex does not have the ability to localize pain in the heart; consequently, the pain is felt elsewhere. d. A lesion has developed in the dorsal root, which is preventing the sensation from being transmitted normally. 7. What controls humans’ ability to perform very skilled movements such as writing? a. Basal ganglia b. Corticospinal tract c. Spinothalamic tract d. Extrapyramidal tract 8. A 30-year-old woman tells the nurse that she has been very unsteady and has had difficulty in maintaining her balance. Which area of the brain most concerns the nurse? a. Thalamus b. Brainstem c. Cerebellum d. Extrapyramidal tract 9. Which of these statements about the peripheral nervous system is correct? a. The CNs enter the brain through the spinal cord. b. Efferent fibers carry sensory input to the central nervous system through the spinal cord. c. The peripheral nerves are inside the central nervous system and carry impulses through their motor fibers. d. The peripheral nerves carry input to the central nervous system by afferent fibers and away from the central nervous system by efferent fibers. 10. A patient has a severed spinal nerve as a result of trauma. Which statement is true in this situation? a. Because there are 31 pairs of spinal nerves, no effect results if only one nerve is severed. b. The dermatome served by this nerve will no longer experience any sensation. c. The adjacent spinal nerves will continue to carry sensations for the dermatome served by the severed nerve. d. A severed spinal nerve will only affect motor function of the patient because spinal nerves have no sensory component. 11. A 21-year-old patient has a head injury resulting from trauma and is unconscious. There are no other injuries. During the assessment what would the nurse expect to find when testing the patient’s deep tendon reflexes? a. Reflexes will be normal. b. Reflexes cannot be elicited. c. All reflexes will be diminished but present. d. Some reflexes will be present, depending on the area of injury. 12. A mother of a 1-month-old infant asks the nurse why it takes so long for infants to learn to roll over. What is the reason for this? a. A demyelinating process must be occurring with her infant. b. Myelin is needed to conduct the impulses, and the neurons of a newborn are not yet myelinated. c. The cerebral cortex is not fully developed; therefore, control over motor function gradually occurs. d. The spinal cord is controlling the movement because the cerebellum is not yet fully developed. 13. During an assessment of an 80-year-old patient, the nurse notices the following: an inability to identify vibrations at her ankle and to identify the position of her big toe, a slower and more deliberate gait, and a slightly impaired tactile sensation. All other neurologic findings are normal. How should the nurse interpret these findings? a. CNS dysfunction b. Lesion in the cerebral cortex c. Normal changes attributable to aging d. Demyelination of nerves attributable to a lesion 14. A 70-year-old woman tells the nurse that every time she gets up in the morning or after she’s been sitting for a while, she gets “really dizzy” and feels like she is going to fall over. What is the best response by the nurse? a. “Have you been extremely tired lately?” b. “You probably just need to drink more liquids.” c. “I’ll refer you for a complete neurologic examination.” d. “You need to get up slowly when you’ve been lying down or sitting.” 15. During the taking of the health history, a patient tells the nurse that “it feels like the room is spinning around me.” How should the nurse document this finding? a. Vertigo b. Syncope c. Dizziness d. Seizure activity 16. When taking the health history on a patient with a seizure disorder, the nurse assesses whether the patient has an aura. Which of these would be the best question for obtaining this information? a. “Does your muscle tone seem tense or limp?” b. “After the seizure, do you spend a lot of time sleeping?” c. “Do you have any warning sign before your seizure starts?” d. “Do you experience any color change or incontinence during the seizure?” 17. While obtaining a health history of a 3-month-old infant from the mother, the nurse asks about the infant’s ability to suck and grasp the mother’s finger. What is the nurse assessing? a. Reflexes b. Intelligence c. Cranial nerves d. Cerebral cortex function 18. In obtaining a health history on a 74-year-old patient, the nurse notes that he drinks alcohol daily and that he has noticed a tremor in his hands that affects his ability to hold things. With this information, what response should the nurse make? a. “Does the tremor change when you drink alcohol?” b. “Does your family know you are drinking every day?” c. “We’ll do some tests to see what is causing the tremor.” d. “You really shouldn’t drink so much alcohol; it may be causing your tremor.” 19. A 50-year-old woman is in the clinic for weakness in her left arm and leg that she has noticed for the past week. The nurse should perform which type of neurologic examination? a. Glasgow Coma Scale b. Neurologic recheck examination c. Complete neurologic examination d. Screening neurologic examination 20. During an assessment of the cranial nerves (CNs), the nurse finds the following: asymmetry when the patient smiles or frowns, uneven lifting of the eyebrows, sagging of the lower eyelids, and escape of air when the nurse presses against the right puffed cheek. These findings indicate dysfunction of which cranial nerve(s)? a. Motor component of CN IV b. Motor component of CN VII c. Motor and sensory components of CN XI d. Motor component of CN X and sensory component of CN VII 21. The nurse is testing the function of CN XI. Which statement best describes the response the nurse should expect if this nerve is intact? a. Patient demonstrates the ability to hear normal conversation. b. When patient sticks out tongue it is midline and without tremors or deviation. c. Patient follows an object with his or her eyes without nystagmus or strabismus. d. Patient moves the head and shoulders against resistance with equal strength. 22. During the neurologic assessment of a “healthy” 35-year-old patient, the nurse asks him to relax his muscles completely. The nurse then moves each extremity through full range of motion. Which of these results would the nurse expect to find? a. Firm, rigid resistance to movement b. Mild, even resistance to movement c. Slight pain with some directions of movement d. Hypotonic muscles as a result of total relaxation. . 23. When the nurse asks a 68-year-old patient to stand with his feet together and arms at his side with his eyes closed, he starts to sway and moves his feet farther apart. How should the nurse document this finding? a. Ataxia b. Lack of coordination c. Negative Homan sign d. Positive Romberg sign 24. The nurse is performing an assessment on a 29-year-old woman who visits the clinic reporting “always dropping things and falling down.” While testing rapid alternating movements, the nurse notices that the woman is unable to pat both of her knees. Her response is extremely slow and she frequently misses. What should the nurse suspect? a. Lesion of CN IX b. Vestibular disease c. Dysfunction of the cerebellum d. Inability to understand directions 25. During the taking of the health history of a 78-year-old man, his wife states that he occasionally has problems with short-term memory loss and confusion: “He can’t even remember how to button his shirt.” When assessing his sensory system, which action by the nurse is most appropriate? a. The nurse would perform the tests, knowing that mental status does not affect sensory ability. b. The nurse would proceed with an explanation of each test, making certain that the wife understands. c. Before testing, the nurse would assess the patient’s mental status and ability to follow directions. d. The nurse would not test the sensory system as part of the examination because the results would not be valid. 26. The assessment of a 60-year-old patient has taken longer than anticipated. In testing his pain perception, the nurse decides to complete the test as quickly as possible. When the nurse applies the sharp point of the pin on his arm several times, he is only able to identify these as one “very sharp prick.” What would be the most accurate explanation for this? a. The patient has hyperesthesia as a result of the aging process. b. This response is most likely the result of the summation effect. c. The nurse was probably not poking hard enough with the pin in the other areas. d. The patient most likely has analgesia in some areas of arm and hyperalgesia in others. 27. The nurse is performing a neurologic assessment on a 41-year-old woman with a history of diabetes. When testing her ability to feel the vibrations of a tuning fork, the nurse notices that the patient is unable to feel vibrations on the great toe or ankle bilaterally, but she is able to feel vibrations on both patellae. Given this information, what would the nurse suspect? a. Hyperalgesia b. Hyperesthesia c. Peripheral neuropathy d. Lesion of sensory cortex 28. The nurse places a key in the hand of a patient and he identifies it as a penny. What term would the nurse use to describe this finding? a. Extinction b. Stereognosis c. Graphesthesia d. Tactile discrimination 29. The nurse is testing the deep tendon reflexes of a 30-year-old woman who is in the clinic for an annual physical examination. When striking the Achilles heel and quadriceps muscle, the nurse is unable to elicit a reflex. How should the nurse proceed? a. Ask the patient to lock her fingers and pull. b. Document these reflexes as 0 on a scale of 0 to 4+. c. Refer the patient to a specialist for further testing. d. Complete the examination, and then test these reflexes again. 30. In assessing a 70-year-old patient who has had a recent cerebrovascular accident, the nurse notices right-sided weakness. What might the nurse expect to find when testing his reflexes on the right side? a. Normal reflexes b. Lack of reflexes c. Diminished reflexes d. Hyperactive reflexes 31. When the nurse is testing the triceps reflex, what is the expected response? a. Flexion of the hand b. Pronation of the hand c. Flexion of the forearm d. Extension of the forearm 32. The nurse is testing superficial reflexes on an adult patient. When stroking up the lateral side of the sole and across the ball of the foot, the nurse notices the plantar flexion of the toes. How should the nurse document this finding? a. Positive Babinski sign b. Plantar reflex abnormal c. Plantar reflex present d. Plantar reflex 2+ on a scale from “0 to 4+” 33. In the assessment of a 1-month-old infant, the nurse notices a lack of response to noise or stimulation. The mother reports that in the last week he has been sleeping all of the time, and when he is awake all he does is cry. The nurse hears that the infant’s cries are very high pitched and shrill. What is the most appropriate response by the nurse? a. Refer the infant for further testing. b. Talk with the mother about eating habits. c. Do nothing; these are expected findings for an infant this age. d. Tell the mother to bring the baby back in 1 week for a recheck. 34. Which of these tests would the nurse use to check the motor coordination of an 11-month-old infant? a. Denver II b. Stereognosis c. Deep tendon reflexes d. Rapid alternating movements 35. To assess the head control of a 4-month-old infant, the nurse lifts up the infant in a prone position while supporting his chest. The nurse looks for what normal response? a. Infant raises the head and arches the back. b. Infant extends the arms and drops down the head. c. Infant flexes the knees and elbows with the back straight. d. Infant holds the head at 45 degrees and keeps the back straight. 36. While assessing a 7-month-old infant, the nurse makes a loud noise and notices the following response: abduction and flexion of the arms and legs; fanning of the fingers, and curling of the index finger and thumb in a C position, followed by the infant bringing in the arms and legs to the body. What does the nurse recall about this response? a. This response could indicate brachial nerve palsy. b. This reaction is an expected startle response at this age. c. This reflex should have disappeared between 1 and 4 months of age. d. This response is normal as long as the movements are bilaterally symmetric. 37. To test for gross motor skill and coordination of a 6-year-old child, which of these techniques would be appropriate? a. Have the child hop on one foot. b. Have the child stand on his head. c. Ask the child to touch his finger to his nose. d. Ask the child to make “funny” faces at the nurse. 38. During the assessment of an 80-year-old patient, the nurse notices that his hands show tremors when he reaches for something and his head is always nodding. No associated rigidity is observed with movement. Which of these statements is most accurate? a. These findings are normal, resulting from aging. b. These findings could be r/t hyperthyroidism. c. These findings are the result of Parkinson disease. d. This patient should be evaluated for a cerebellar lesion. 39. While the nurse is taking the history of a 68-year-old patient who sustained a head injury 3 days earlier, he tells the nurse that he is on a cruise ship and is 30 years old. What does this finding indicate? a. Great sense of humor b. Uncooperative behavior c. Decreased level of consciousness d. Inability to understand questions 40. The nurse is caring for a patient who has just had neurosurgery. To assess for increased intracranial pressure, what would the nurse include in the assessment? a. CNs, motor function, and sensory function b. Deep tendon reflexes, vital signs, and coordinated movements c. Level of consciousness, motor function, pupillary response, and vital signs d. Mental status, deep tendon reflexes, sensory function, and pupillary response 41. During an assessment of a 22-year-old woman who sustained a head injury from an automobile accident 4 hours earlier, the nurse notices the following changes: pupils were equal, but now the right pupil is fully dilated and nonreactive, and the left pupil is 4 mm and reacts to light. What do these findings suggest? a. Injury to the O.D. b. Test inaccurately performed c. Increased intracranial pressure d. Normal response after a head injury 42. A 32-year-old woman tells the nurse that she has noticed “very sudden, jerky movements” mainly in her hands and arms. She says, “They seem to come and go, primarily when I am trying to do something. I haven’t noticed them when I’m sleeping.” What do these symptoms suggest? a. Tics b. Chorea c. Athetosis d. Myoclonus 43. During an assessment of a 62-year-old man, the nurse notices the patient has a stooped posture, shuffling walk with short steps, flat facial expression, and pill-rolling finger movements. What do these findings suggest? a. Parkinsonism b. Cerebral palsy c. Cerebellar ataxia d. Muscular dystrophy 44. During an assessment of a 32-year-old patient with a recent head injury, the nurse notices that the patient responds to pain by extending, adducting, and internally rotating his arms. His palms pronate, and his lower extremities extend with plantar flexion. Which statement concerning these findings is most accurate? What do these findings indicate? a. A lesion of the cerebral cortex b. A completely nonfunctional brainstem c. Normal findings that will resolve in 24 to 48 hours d. A very ominous sign and may indicate brainstem injury 45. A 78-year-old man has a history of a cerebrovascular accident. The nurse notes that when he walks, his left arm is immobile against the body with flexion of the shoulder, elbow, wrist, and fingers and adduction of the shoulder. His left leg is stiff and extended and circumducts with each step. What type of gait disturbance is this individual experiencing? a. Scissors gait b. Cerebellar ataxia c. Parkinsonian gait d. Spastic hemiparesis 46. In a person with an upper motor neuron lesion such as a cerebrovascular accident, which of these physical assessment findings should the nurse expect? a. Hyperreflexia b. Fasciculations c. Loss of muscle tone and flaccidity d. Atrophy and wasting of the muscles 47. A 59-year-old patient has a herniated intervertebral disk. Which of the following findings should the nurse expect to see on physical assessment of this individual? a. Hyporeflexia b. Increased muscle tone c. Positive Babinski sign d. Presence of pathologic reflexes 48. A patient is unable to perform rapid alternating movements such as rapidly patting her knees. How should the nurse document this finding? a. Ataxia b. Astereognosis c. Loss of kinesthesia d. Presence of dysdiadochokinesia 49. The nurse should test the functioning of which structure(s) when determining whether a person is oriented to his or her surroundings? a. Cerebellum b. Cranial nerves c. Cerebral cortex d. Medulla oblongata 50. During an examination, the nurse notices severe nystagmus in both eyes of a patient. Which conclusion by the nurse is correct? a. A normal occurrence b. Indicates disease of the cerebellum or brainstem c. A sign that the patient is nervous about the examination d. Indicates a visual problem, and a referral to an ophthalmologist is indicated 51. What does testing kinesthesia assess? a. Fine touch b. Position sense c. Motor coordination d. Perception of vibration 52. The nurse is reviewing a patient’s medical record and notes that he is in a coma. Using the Glasgow Coma Scale, which number indicates that the patient is in a coma? a. 6 b. 12 c. 15 d. 24 53. A man who was found wandering in a park at 2 AM has been brought to the emergency department for an examination; he said he fell and hit his head. During the examination, the nurse asks him to use his index finger to touch the nurse’s finger, then his own nose, then the nurse’s finger again (which has been moved to a different location). The patient is clumsy, unable to follow the instructions, and overshoots the mark, missing the finger. What does the nurse suspect? a. Cerebral injury b. Peripheral neuropathy c. Cerebrovascular accident d. Acute alcohol intoxication 54. The nurse is assessing the neurologic status of a patient who has a late-stage brain tumor. With the reflex hammer, the nurse draws a light stroke up the lateral side of the sole of the foot and inward, across the ball of the foot. In response, the patient’s toes fan out, and the big toe shows dorsiflexion. How should the nurse interpret these findings? a. Clonus, which is a hyperactive response b. Achilles reflex, which is an expected response c. Negative Babinski sign, which is normal for adults d. Positive Babinski sign, which is abnormal for adults MULTIPLE RESPONSE 1. A 69-year-old patient has been admitted to an adult psychiatric unit because his wife thinks he is getting more and more confused. He laughs when he is found to be forgetful, saying “I’m just getting old!” After the nurse completes a thorough neurologic assessment, which findings would be indicative of Alzheimer disease? (Select all that apply.) a. Getting lost in one’s own neighborhood b. Occasionally forgetting names or appointments c. Sometimes having trouble finding the right word d. Misplacing items, such as putting dish soap in the refrigerator e. Difficulty performing familiar tasks, such as placing a telephone call f. Rapid mood swings, from calm to tears, for no apparent reason. COMPLETION 1. During the assessment of deep tendon reflexes, the nurse finds that a patient’s responses are bilaterally normal. What number is used to indicate normal deep tendon reflexes when the documenting this finding? +Correct. Chapter 25: Male Genitourinary System Jarvis: Physical Examination and Health Assessment, 8th Edition MULTIPLE CHOICE 1. Which is a structure of the external male genital? a. Testis b. Scrotum c. Epididymis d. Vas deferens 2. Which is an accessory glandular structure for the male genital organs? a. Testis b. Scrotum c. Prostate d. Vas deferens 3. Which of these statements is true regarding the penis? a. The urethral meatus is located on the ventral side of the penis. b. The prepuce is the fold of foreskin covering the shaft of the penis. c. The penis is made up of two cylindric columns of erectile tissue. d. The corpus spongiosum expands into a cone of erectile tissue called the glans. 4. When performing a genital examination on a 25-year-old man, the nurse notices deeply pigmented, wrinkled scrotal skin with large sebaceous follicles. On the basis of this information, how should the nurse proceed? a. Squeeze the glans to check for the presence of discharge. b. Consider this finding as normal, and proceed with the examination. c. Assess the testicles for the presence of masses or painless lumps. d. Obtain a more detailed history, focusing on any scrotal abnormalities the patient has noticed. 5. Which statement concerning the testes is true? a. The vas deferens is located along the inferior portion of each testis. b. The lymphatic vessels of the testes drain into the abdominal lymph nodes. c. The right testis is lower than the left because the right spermatic cord is longer. d. The cremaster muscle contracts in response to cold and draws the testicles closer to the body. 6. A male patient with possible fertility problems asks the nurse where sperm is produced. Which answer should the nurse give the patient? a. Testes b. Prostate c. Epididymis d. Vas deferens 7. A 62-year-old man states that his physician told him that he has an “inguinal hernia.” He asks the nurse to explain what a hernia is. Which response by the nurse is best? a. “Don’t worry, most men your age develop hernias.” b. “A hernia is often the result of a prenatal growth abnormality.” c. “You should talk to your physician since he or she made the initial diagnosis.” d. “A hernia is a loop of bowel protruding through a weak spot in the abdominal muscles.” 8. The mother of a 10-year-old boy asks the nurse about the recognition of puberty. How should the nurse reply? a. “Puberty usually begins around 15 years of age.” b. “The first sign of puberty is an enlargement of the testes.” c. “The penis size does not increase until about 16 years of age.” d. “The development of pubic hair precedes testicular or penis enlargement.” 9. During an examination of an aging man, the nurse recognizes that which finding is an expected or normal change? a. Enlarged scrotal sac b. Increased pubic hair c. Decreased penis size d. Increased rugae over the scrotum 10. An older man is concerned about his sexual performance. In addition to a disease, what else should the nurse explain can cause a withdrawal from sexual activity later in life? a. Decreased sperm production b. Side effects of medications c. Decreased libido with aging d. Decreased pleasure from sexual intercourse 11. A 59-year-old patient has been diagnosed with prostatitis and is being seen at the clinic for reports of burning and pain during urination. How should the nurse document this finding? a. Dysuria b. Nocturia c. Polyuria d. Hematuria 12. When the nurse is conducting a sexual history from a male adolescent, which statement would be most appropriate to use at the beginning of the interview? a. “Do you use condoms?” b. “You don’t masturbate, do you?” c. “Have you had sex in the last 6 months?” d. “Often adolescents your age have questions about sexual activity.” 13. Which of these statements is most appropriate when the nurse is obtaining a genitourinary history from an older man? a. “Do you need to get up at night to urinate?” b. “Do you experience nocturnal emissions, or ‘wet dreams’?” c. “Do you know how to perform a testicular self-examination?” d. “Has anyone ever touched your genitals when you did not want them to?” 14. When the nurse is performing a genital examination on a male patient, the patient has an erection. How should the nurse respond? a. Ask the patient if he would like someone else to examine him. b. Continue with the examination as though nothing has happened. c. Stop the examination, leave the room while stating that the examination will resume at a later time. d. Reassure the patient that this is a normal response and continue with the examination. 15. The nurse is examining the glans and knows which finding is normal for this area? a. Hair is without pest inhabitants. b. The skin is wrinkled and without lesions. c. Smegma may be present under the foreskin of an uncircumcised male. d. The meatus may have a slight discharge when the glans is compressed. 16. When performing a genitourinary assessment, the nurse notices that the urethral meatus is ventrally positioned. What does this indicate? a. Hypospadias b. A result of phimosis c. Probably due to a stricture d. Often associated with aging 17. The nurse is performing a genital examination on a male patient and notices urethral drainage. What should the nurse do when collecting urethral discharge for microscopic examination and culture? a. Ask the patient to urinate into a sterile cup. b. Ask the patient to obtain a specimen of semen. c. Insert a cotton-tipped applicator into the urethra. d. Compress the glans between the examiner’s thumb and forefinger, and collect any discharge. 18. When assessing the scrotum of a male patient, the nurse notices the presence of multiple firm, nontender, yellow 1-cm nodules. What does this finding indicate? a. Urethritis b. Sebaceous cysts c. Subcutaneous plaques d. Due to an inflammation of the epididymis 19. When performing a scrotal assessment, the nurse notices that the scrotal contents show a red glow with transillumination. How should the nurse proceed? a. Assess the patient for the presence of a hernia. b. Suspect the presence of serous fluid in the scrotum. c. Refer the patient for evaluation of a mass in the scrotum. d. Consider this finding normal and proceed with the examination. 20. When the nurse is performing a genital examination on a male patient, which action is correct? a. Auscultating for the presence of a bruit over the scrotum b. Palpating the inguinal canal only if a bulge is present in the inguinal region during inspection c. Palpating for the vertical chain of lymph nodes along the groin, inferior to the inguinal ligament d. Having the patient shift his weight onto the left (unexamined) leg when palpating for a hernia on the right side 21. The nurse is aware that which statement is true regarding the incidence of testicular cancer? a. The cure rate for testicular cancer is low. b. Testicular cancer is the most common cancer in men aged 30 to 50 years. c. The early symptoms of testicular cancer are pain and induration. d. Men with a history of cryptorchidism are at the greatest risk for the development of testicular cancer. 22. The nurse is describing how to perform a testicular self-examination to a patient. Which statement is most appropriate? a. “A good time to examine your testicles is just before you take a shower.” b. “The testicle is egg shaped and movable. It feels firm and has a lumpy consistency.” c. “If you notice an enlarged testicle or a painless lump, call your health care provider.” d. “Perform a testicular examination at least once a week to detect the early stages of testicular cancer.” 23. A 2-month-old uncircumcised infant has been brought to the clinic for a well-baby checkup. How would the nurse proceed with the genital examination? a. Elicit the cremasteric reflex. b. The glans is assessed for redness or lesions. c. Retracting the foreskin should be avoided until the infant is 3 months old. d. Any dirt or smegma that has collected under the foreskin should be noted. 24. A 2-year-old boy has been diagnosed with physiologic cryptorchidism. Considering this diagnosis, what will the nurse most likely observe during the assessment? a. Testes that are hard and painful to palpation b. Atrophic scrotum and a bilateral absence of the testis c. Absence of the testis in the scrotum, but the testis can be milked down d. Testes that migrate into the abdomen when the child squats or sits cross-legged 25. What is a common assessment finding in a boy younger than 2 years old? a. Inflamed and tender spermatic cord b. Presence of a hernia in the scrotum c. Penis that looks large in relation to the scrotum d. Presence of a hydrocele, or fluid in the scrotum 26. During an examination of an aging man, what finding would the nurse expect? a. Change in scrotal color b. Decrease in the size of the penis c. Enlargement of the testes and scrotum d. Increase in the number of rugae over the scrotal sac 27. When performing a genital assessment on a middle-aged man, the nurse notices multiple soft, moist, painless papules in the shape of cauliflower-like patches scattered across the shaft of the penis. What does this finding suggest? a. Carcinoma b. Genital warts c. Genital herpes d. Syphilitic chancres 28. When performing a genitourinary assessment on a 16-year-old male adolescent, the nurse notices a swelling in the scrotum that increases with increased intra-abdominal pressure and decreases when he is lying down. The patient reports pain when straining. What do these findings indicate? a. Femoral hernia b. Incisional hernia c. Direct inguinal hernia d. Indirect inguinal hernia 29. When the nurse is performing a testicular examination on a 25-year-old man, which finding is considered normal? a. Nontender subcutaneous plaques b. Scrotal area that is dry, scaly, and nodular c. Testes that feel oval and movable and are slightly sensitive to compression d. Single, hard, circumscribed, movable mass, less than 1 cm under the surface of the testes 30. The nurse is inspecting the scrotum and testes of a 43-year-old man. Which finding would require additional follow-up and evaluation? a. Skin on the scrotum is taut. b. Left testicle hangs lower than the right testicle. c. Scrotal skin has yellowish 1-cm nodules that are firm and nontender. d. Testes move closer to the body in response to cold temperatures. 31. A 55-year-old man is experiencing severe pain of sudden onset in the scrotal area. It is somewhat relieved by elevation. On examination the nurse notices an enlarged, red scrotum that is very tender to palpation. Distinguishing the epididymis from the testis is difficult, and the scrotal skin is thick and edematous. What do these findings suggest? a. A varicocele b. Epididymitis c. A spermatocele d. Testicular torsion 32. The nurse is performing a genitourinary assessment on a 50-year-old obese male laborer. On examination, the nurse notices a painless round swelling close to the pubis in the area of the internal inguinal ring that is easily reduced when the individual is supine. What type of hernia do these findings suggest? a. Scrotal b. Femoral c. Direct inguinal d. Indirect inguinal 33. The nurse is providing patient teaching about an erectile dysfunction drug. One of the drug’s potential side effects is prolonged, painful erection of the penis without sexual stimulation. What is the medical term for this condition? a. Orchitis b. Phimosis c. Stricture d. Priapism 34. During an examination, the nurse notices that a male patient has a red, round, superficial ulcer with a yellowish serous discharge on his penis. On palpation, the nurse finds a nontender base that feels like a small button between the thumb and fingers. What do these findings indicate? a. Genital warts b. Herpes infection c. Carcinoma lesion d. Syphilitic chancre 35. During a health history, a patient tells the nurse that he has trouble in starting his urine stream. How should the nurse document this finding? a. Urgency b. Dribbling c. Frequency d. Hesitancy 36. During a genital examination, the nurse notices that a male patient has clusters of small vesicles on the glans, surrounded by erythema. What does this finding suggest? a. Genital warts b. Genital herpes c. Peyronie disease d. Syphilitic chancres 37. During a physical examination, the nurse finds that a male patient’s foreskin is fixed and tight and will not retract over the glans. What is this condition called? a. Phimosis b. Epispadias c. Peyronie disease d. Urethral stricture MULTIPLE RESPONSE 1. A 55-year-old man is in the clinic for a yearly checkup. He is worried because his father died of prostate cancer. The nurse knows which tests should be performed at this time? (Select all that apply.) a. Urinalysis b. Prostate biopsy c. Transrectal ultrasound d. Digital rectal examination (DRE) e. Blood test for prostate-specific antigen (PSA) 2. A 16-year-old boy is brought to the clinic for a problem that he refused to let his mother see. The nurse examines him, and finds that he has scrotal swelling on the left side. He had the mumps the previous week, and the nurse suspects that he has orchitis. Which of the following assessment findings support this diagnosis? (Select all that apply.) a. Swollen testis b. Mass that transilluminates c. Scrotal skin that is reddened d. Mass that does not transilluminate e. Scrotum that is tender upon palpation f. Scrotum that is nontender upon palpation Chapter 26: Anus, Rectum, and Prostate Jarvis: Physical Examination and Health Assessment, 8th Edition MULTIPLE CHOICE 1. Which statement concerning the anal canal is true? a. Slants backward toward the sacrum b. Contains hair and sebaceous glands c. Approximately 2 cm long in the adult d. The outlet for the gastrointestinal tract 2. Which statement concerning the sphincters is correct? a. The internal sphincter is under voluntary control. b. The external sphincter is under voluntary control. c. Both sphincters remain slightly relaxed at all times. d. The internal sphincter surrounds the external sphincter. 3. The nurse is performing an examination of the anus and rectum. Which of these statements is correct and important to remember during this examination? a. The rectum is approximately 8 cm long. b. The anorectal junction cannot be palpated. c. Above the anal canal, the rectum turns anteriorly. d. There are no sensory nerves in the anal canal or rectum. 4. What structure secretes a thin, milky alkaline fluid to enhance the viability of sperm? a. Cowper gland b. Median sulcus c. Prostate gland d. Bulbourethral gland 5. A 46-year-old man requires an assessment of his sigmoid colon. Which instrument or technique is most appropriate for this examination? a. Ultrasound b. Proctoscope c. Colonoscope d. Rectal examination with an examining finger 6. The nurse is caring for a newborn infant. Thirty hours after birth, the infant passes a dark green meconium stool. What is the importance of this finding? a. The stool indicates anal patency. b. The dark green color indicates occult blood in the stool. c. Meconium stool can be reflective of distress in the newborn. d. The newborn should have passed the first stool within 12 hours of birth. 7. During the assessment of an 18-month-old infant, the mother expresses concern to the nurse about the infant’s inability to toilet train. What would be the best response by the nurse? a. “Some children are just more difficult to train, so I wouldn’t worry about it yet.” b. “Have you considered reading any of the books on toilet training? They can be very helpful.” c. “This could mean that there is a problem with your baby’s development. We’ll watch her closely for the next few months.” d. “The nerves that allow your baby to have control over the passing of stools are not developed until at least 18 to 24 months of age.” 8. A 60-year-old man has just been told that he has benign prostatic hypertrophy (BPH). He has a friend who just died from cancer of the prostate and is concerned this will happen to him. How should the nurse respond? a. “The swelling in your prostate is only temporary and will go away.” b. “We will treat you with chemotherapy so we can control the cancer.” c. “It would be very unusual for a man your age to have cancer of the prostate.” d. “The enlargement of your prostate is caused by hormonal changes, and not cancer.” 9. A 30-year-old woman is visiting the clinic because of “pain in my bottom when I have a bowel movement.” The nurse should assess for which problem? a. Pinworms b. Hemorrhoids c. Colon cancer d. Fecal incontinence 10. A patient who is visiting the clinic reports having “stomach pains for 2 weeks” and describes his stools as being “soft and black” for approximately the last 10 days. He denies taking any medications. What do these symptoms suggest? a. Excessive fat caused by malabsorption b. Absent bile pigment from liver problems c. Increased iron intake, resulting from a change in diet d. Occult blood, resulting from gastrointestinal bleeding 11. After completing an assessment of a 60-year-old white male with a family history of colon cancer, the nurse discusses with him early detection measures for colon cancer. What should the nurse include in the instructions? a. Annual proctoscopy b. Annual PSA blood test c. Colonoscopy every 10 years d. Fecal occult blood test every 6 months 12. The mother of a 5-year-old girl tells the nurse that she has noticed her daughter “scratching at her bottom a lot the last few days.” During the assessment, the nurse finds redness and raised skin in the anal area. What does this finding likely indicate? a. Pinworms b. Chickenpox c. Constipation d. Bacterial infection 13. The nurse is examining only the rectal area of a woman and should place the woman in what position? a. Prone b. Lithotomy c. Left lateral decubitus d. Bending over the table while standing 14. While performing an assessment of the perianal area of a patient, the nurse notices that the pigmentation of the anus is darker than the surrounding skin, the anal opening is closed, and a skin sac that is shiny and blue is noted. The patient mentioned that he has had pain with bowel movements and has occasionally noted some spots of blood. What does this assessment and history most likely indicate? a. Anal fistula b. Pilonidal cyst c. Rectal prolapse d. Thrombosed hemorrhoid 15. The nurse is preparing to palpate the rectum and should use which of these techniques? a. Flex the finger, and slowly insert it toward the umbilicus. b. Insert an extended index finger at a right angle to the anus. c. First instruct the patient that this procedure will be painful. d. Place the finger directly into the anus to overcome the tight sphincter. 16. While performing a rectal examination, the nurse notices a firm, irregularly shaped mass. What should the nurse do next? a. Continue with the examination, and document the finding in the chart. b. Instruct the patient to return for a repeat assessment in 1 month. c. Tell the patient that a mass was felt, but it is nothing to worry about. d. Report the finding, and refer the patient to a specialist for further examination. 17. During an assessment of the newborn, the nurse expects to see which finding when the anal area is slightly stroked? a. Jerking of the legs b. Flexion of the knees c. Quick contraction of the sphincter d. Relaxation of the external sphincter 18. A 13-year-old girl is visiting the clinic for a sports physical examination. The nurse should remember to include which of these tests in the examination? a. Valsalva maneuver b. Testing for occult blood c. Internal palpation of the anus d. Inspection of the perianal area 19. During an assessment of a 20-year-old man, the nurse finds a small palpable lesion with a tuft of hair located directly over the coccyx. What does this finding indicate? a. Carcinoma b. Rectal polyp c. Pruritus ani d. Pilonidal cyst 20. A 70-year-old man is visiting the clinic for difficulty in passing urine. In the health history, he indicates that he has to urinate frequently, especially at night. He has burning when he urinates and has noticed pain in his back. Considering this history, what might the nurse expect to find during the physical assessment? a. Asymmetric, hard, and fixed prostate gland b. Occult blood and perianal pain to palpation c. Symmetrically enlarged, soft prostate gland d. Soft nodule protruding from the rectal mucosa 21. A 40-year-old black man is in the office for his annual physical examination. Which statement regarding the PSA blood test is true? a. Should be performed at age 50 years. b. Should be performed with this visit. c. Should be performed at age 45 years. d. Is only necessary if a family history of prostate cancer exists. 22. A 62-year-old man is experiencing fever, chills, malaise, urinary frequency, and urgency. He also reports urethral discharge and a dull aching pain in the perineal and rectal area. These symptoms are most consistent with which condition? a. BPH b. Polyps c. Prostatitis d. Carcinoma of the prostate 23. During a group discussion on men’s health, what group should the nurse inform them has the highest incidence of prostate cancer? a. Blacks b. Hispanics c. Asian Americans d. American Indians 24. While palpating the prostate gland through the rectum, which finding would the nurse recognize as abnormal? a. Heart shaped b. Palpable central groove c. Tenderness to palpation d. Elastic and rubbery consistency 25. The nurse notices that a patient has had a pale, yellow, greasy stool, or steatorrhea. What is the cause of this finding? a. Occult bleeding b. Absent bile pigment c. Increased fat content d. Ingestion of bismuth preparations 26. During the health history of a patient who reports chronic constipation, the patient asks the nurse about foods to eat to avoid constipation. What should the nurse include as an example of an appropriate food? a. Yogurt b. Broccoli c. Ground beef d. Iceberg lettuce 27. While assessing a hospitalized patient who is jaundiced, the nurse notices that the patient has been incontinent of stool. The stool is loose and gray-tan in color. What does this finding indicate? a. Occult blood b. Inflammation c. Absent bile pigment d. Ingestion of iron preparations 28. During a digital examination of the rectum, the nurse notices that the patient has hard feces in the rectum. The patient complains of feeling “full,” has a distended abdomen, and states that she has not had a bowel movement “for several days.” The nurse suspects which condition? a. Rectal polyp b. Rectal abscess c. Fecal impaction d. Fecal incontinence 29. As the nurse is taking the health history, the patient states, “It really hurts back there, and sometimes it itches, too. I have even seen blood on the tissue when I have a bowel movement. Is there something there?” The nurse should expect to see which of these upon examination of the anus? a. Rectal prolapse b. Internal hemorrhoid c. External hemorrhoid that has resolved d. External hemorrhoid that is thrombosed MULTIPLE RESPONSE 1. The nurse is performing a digital examination of a patient’s prostate gland and notices that a normal prostate gland includes which of the following characteristics? (Select all that apply.) a. Fixed mobility b. Boggy with a soft consistency c. 1 cm protrusion into the rectum d. Flat shape with no palpable groove e. Heart-shaped with a palpable central groove f. Smooth surface, elastic, and rubbery consistency Chapter 27: Female Genitourinary System Jarvis: Physical Examination and Health Assessment, 8th Edition MULTIPLE CHOICE 1. During a health history, a 22-year-old woman asks, “Can I get that vaccine for human papilloma virus (HPV)? I have genital warts and I’d like them to go away!” What is the nurse’s best response? a. “The HPV vaccine is for girls and women ages 9 to 26 years, so we can start that today.” b. “This vaccine is only for girls who have not yet started to become sexually active.” c. “Let’s check with the physician to see if you are a candidate for this vaccine.” d. “The vaccine cannot protect you if you already have an HPV infection.” 2. The nurse is examining a female patient’s vestibule. What does the nurse expect to visualize? a. Urethral meatus and vaginal orifice b. Vaginal orifice and vestibular (Bartholin) glands c. Urethral meatus and paraurethral (Skene) glands d. Paraurethral (Skene) and vestibular (Bartholin) glands 3. During a speculum inspection of the vagina, what would the nurse expect to see at the end of the vaginal canal? a. Cervix b. Uterus c. Ovaries d. Fallopian tubes 4. The uterus is usually positioned tilting forward and superior to the bladder. What is this position called? a. Anteverted and anteflexed b. Retroverted and anteflexed c. Retroverted and retroflexed d. Superiorverted and anteflexed 5. An 11-year-old girl is in the clinic for a sports physical examination. The nurse notices that she has begun to develop breasts, and during the conversation the girl reveals that she is worried about her development. The nurse should use which of these techniques to best assist the young girl in understanding the expected sequence for development? a. Use the Tanner scale on the five stages of sexual development. b. Describe her development and compare it with that of other girls her age. c. Use the Jacobsen table on expected development on the basis of height and weight data. d. Reassure her that her development is within normal limits and tell her not to worry about the next step. 6. A woman who is 8 weeks pregnant is in the clinic for a checkup. The nurse reads on her chart that her cervix is softened and looks cyanotic. Based on these findings, what two signs is the patient exhibiting? a. Tanner and Hegar b. Hegar and Goodell c. Chadwick and Hegar d. Goodell and Chadwick 7. What usually occurs to the cells in the reproductive tract to cause the changes normally associated with menopause? a. Aging b. Becoming fibrous c. Estrogen dependent d. Able to respond to progesterone 8. The nurse is reviewing the changes that occur with menopause. Which changes are expected? a. Uterine and ovarian atrophy, along with thinning of vaginal epithelium b. Ovarian atrophy, increased vaginal secretions, and increasing clitoral size c. Cervical hypertrophy, ovarian atrophy, and increased acidity of vaginal secretions d. Vaginal mucosa fragility, increased acidity of vaginal secretions, and uterine hypertrophy 9. A 54-year-old woman who has just completed menopause is in the clinic today for a yearly physical examination. Which of these statements should the nurse include in patient education? a. “You can continue with hormone replacement therapy as it actually decreases your risk for breast cancer.” b. “You should be aware that you’re at increased risk for dyspareunia because of decreased vaginal secretions.” c. “You have only stopped menstruating and there are not really any other changes that you need to be concerned about.” d. “You likely may have difficulty with sexual pleasure as a result of drastic changes in the female sexual response cycle.” 10. A woman is in the clinic for an annual gynecologic examination. How should the nurse begin the interview? a. Menstrual history, because it is generally nonthreatening. b. Sexual history, because discussing it first will build rapport. c. Obstetric history, because it includes the most important information. d. Urinary system history, because problems may develop in this area as well. 11. A patient has had three pregnancies and two live births. How should the nurse record this information? a. G2; P2; AB1 b. G3; P2; AB0 c. G3; P2; AB1 d. G3; P3; AB1 12. During the interview with a female patient, the nurse gathers data that indicates the patient is perimenopausal. Which of these statements made by this patient leads to this conclusion? a. “I have noticed that my muscles ache at night when I go to bed.” b. “I will be very happy when I can stop worrying about having a period.” c. “I have been noticing that I sweat a lot more than I used to, especially at night.” d. “I have only been pregnant twice, but both times I had breast tenderness as my first symptom.” 13. A 50-year-old woman calls the clinic because she has noticed some changes in her body and breasts and wonders if these changes could be attributable to the hormone replacement therapy (HRT) she started 3 months earlier. How should the nurse respond? a. “HRT is at such a low dose that side effects are very unusual.” b. “HRT has several side effects, including fluid retention, breast tenderness, and vaginal bleeding.” c. “Vaginal bleeding with HRT is very unusual; I suggest you come into the clinic immediately to have this evaluated.” d. “It sounds as if your dose of estrogen is too high; I think you may need to decrease the amount you are taking and then call back in a week.” 14. A 52-year-old patient states that when she sneezes or coughs she “wets herself a little.” She is very concerned that something may be wrong with her. What does this finding suggest? a. Dysuria b. Hematuria c. Urge incontinence d. Stress incontinence 15. During the interview, a patient reveals that she has some vaginal discharge. She is worried that it may be a sexually transmitted infection. What would be the most appropriate response by the nurse? a. “Oh, don’t worry. Some cyclic vaginal discharge is normal.” b. “Have you been engaging in unprotected sexual intercourse?” c. “I’d like more information about the discharge. What color is it?” d. “Have you had any urinary incontinence associated with the discharge?” 16. A woman states that 2 weeks ago she had a urinary tract infection that was treated with an antibiotic. What should the nurse ask the woman? a. “Have you had excessive vaginal bleeding?” b. “Have you experienced changes in your urination patterns?” c. “Do you have any unusual vaginal discharge or itching?” d. “Have you noticed any changes in your desire for intercourse?” 17. Which statement would be most appropriate when the nurse is introducing the topic of sexual relationships during a health interview? a. “Now, it is time to talk about your sexual history. When did you first have intercourse?” b. “Most women your age have had more than one sexual partner. How many would you say you have had?” c. “Women often feel dissatisfied with their sexual relationships. Would it be okay to discuss this now?” d. “Women often have questions about their sexual relationship and how it affects their health. Do you have any questions?” 18. A 22-year-old woman has been considering using oral contraceptives. As a part of her health history, what should the nurse ask? a. “Do you have a history of heart murmurs?” b. “Will you be in a monogamous relationship?” c. “Have you carefully thought this choice through?” d. “If you smoke, how many cigarettes do you smoke per day?” 19. A married couple has come to the clinic seeking advice on pregnancy. They have been trying to conceive for 4 months and have not been successful. What should the nurse do first? a. Ascertain whether either of them has been using broad-spectrum antibiotics. b. Explain that couples are considered infertile after 1 year of unprotected intercourse. c. Immediately refer the woman to an expert in pelvic inflammatory disease—the most common cause of infertility. d. Explain that couples are considered infertile after 3 months of engaging in unprotected intercourse and that they will need a referral to a fertility expert. 20. A nurse is assessing a patient’s risk for contracting a sexually transmitted infection (STI). What is an appropriate question to ask this patient? a. “Do you have a sexually transmitted infection?” b. “You are aware of the dangers of unprotected sex, aren’t you?” c. “You know that it’s important to use condoms for protection, right?” d. “Do you use a condom with each episode of sexual intercourse?” 21. When the nurse is interviewing a preadolescent girl, which opening question would be least threatening? a. “Do you have any questions about growing up?” b. “What has your mother told you about growing up?” c. “When did you notice that your body was changing?” d. “I remember being very scared when I got my period. How do you think you’ll feel?” 22. When the nurse is discussing sexuality and sexual issues with an adolescent, a permission statement helps convey that it is normal to think or feel a certain way. Which statement is the best example of a permission statement? a. “It is okay that you have become sexually active.” b. “Girls your age often have questions about sexual activity. Do you have any questions?” c. “If it is okay with you, I’d like to ask you some questions about your sexual history.” d. “Girls your age often engage in sexual activities. It is okay to tell me if you have had intercourse.” 23. The nurse is preparing to interview a postmenopausal woman. Which of these statements is true as it applies to obtaining the health history of a postmenopausal woman? a. The nurse should screen for monthly breast tenderness. b. The nurse should ask a postmenopausal woman if she has ever had vaginal bleeding. c. Once a woman reaches menopause, the nurse does not need to ask any history questions. d. Postmenopausal women are not at risk for contracting STIs; therefore, these questions can be omitted. 24. During the examination portion of a patient’s visit, she will be in lithotomy position. Which statement reflects some things that the nurse can do to make this position more comfortable for her? a. Ask her to place her hands and arms over her head. b. Elevate her head and shoulders to maintain eye contact. c. Allow her to choose to have her feet in the stirrups or have them resting side by side on the edge of the table. d. Allow her to keep her buttocks approximately 6 inches from the edge of the table to prevent her from feeling as if she will fall off. 25. An 18-year-old patient is having her first pelvic examination. Which action by the nurse is appropriate? a. Inviting her mother to be present during the examination. b. Avoiding the lithotomy position for this first time because it can be uncomfortable and embarrassing. c. Raising the head of the examination table and giving her a mirror so that she can view the examination. d. Fully draping her, leaving the drape between her legs elevated to avoid embarrassing her with eye contact. 26. The nurse has just completed an inspection of a nulliparous woman’s external genitalia. Which of these would be a description of a finding within normal limits? a. Redness of the labia majora b. Multiple nontender sebaceous cysts c. Gaping and slightly shriveled labia majora d. Discharge that is foul smelling and irritating 27. The nurse is preparing for an internal genitalia examination of a woman. Which order of the examination is correct? a. Bimanual, speculum, and rectovaginal b. Speculum, rectovaginal, and bimanual c. Speculum, bimanual, and rectovaginal d. Rectovaginal, bimanual, and speculum 28. During an internal examination of a woman’s genitalia, the nurse will use which technique for proper insertion of the speculum? a. The woman is instructed to bear down, the speculum blades are opened and applied in a swift, upward movement. b. The woman is instructed to bear down, the width of the blades is horizontally turned, and the speculum is inserted downward at a 45-degree angle toward the small of the woman’s back. c. The blades of the speculum are inserted on a horizontal plane, turning them to a 30-degree angle while continuing to insert them. The woman is asked to bear down after the speculum is inserted. d. The blades are locked open by turning the thumbscrew. Once the blades are open, pressure is applied to the introitus and the blades are inserted downward at a 45-degree angle to bring the cervix into view. 29. The nurse is examining a 35-year-old female patient. During the health history, the nurse notices that she has had two term pregnancies, and both babies were delivered vaginally. During the internal examination, the nurse observes that the cervical os is a horizontal slit with some healed lacerations and that the cervix has some nabothian cysts that are small, smooth, and yellow. In addition, the nurse notices that the cervical surface is granular and red, especially around the os. Finally, the nurse notices the presence of stringy, opaque, odorless secretions. Which of these findings are abnormal? a. Nabothian cysts are present. b. The cervical os is a horizontal slit. c. The cervical surface is granular and red. d. Stringy and opaque secretions are present. 30. A patient calls the clinic for instructions before having a Papanicolaou (Pap) smear. What is an appropriate response by the nurse? a. “If you are menstruating, please use pads to avoid placing anything into the vagina.” b. “Avoid intercourse, inserting anything into the vagina, or douching within 24 hours of your appointment.” c. “We would like you to use a mild saline douche before your examination. You may pick this up in our office.” d. “If you suspect that you have a vaginal infection, please gather a sample of the discharge to bring with you.” 31. During an examination, which tests will the nurse collect to screen for cervical cancer? a. Endocervical specimen, cervical scrape, and vaginal pool b. Endocervical specimen, vaginal pool, and acetic acid wash c. Cervical scrape, acetic acid wash, saline mount (wet prep) d. Endocervical specimen, potassium hydroxide (KOH) preparation, and acetic acid wash 32. When performing the bimanual examination, the nurse notices that the cervix feels smooth and firm, is round, and is fixed in place (does not move). When cervical palpation is performed, the patient complains of some pain. How should the nurse interpret these findings? a. These findings are all within normal limits. b. Pain may occur during palpation of the cervix. c. Cervical consistency should be soft and velvety—not firm. d. The cervix should move when palpated; an immobile cervix may indicate malignancy. 33. The nurse is palpating a female patient’s adnexa. The findings include a firm, smooth uterine wall; the ovaries are palpable and feel smooth and firm. The fallopian tube is firm and pulsating. How should the nurse proceed? a. Tell the patient that her examination is normal. b. Give her an immediate referral to a gynecologist. c. Suggest that she return in a month for a recheck to verify the findings. d. Tell the patient that she may have an ovarian cyst that should be evaluated further. 34. A 65-year-old woman is in the office for routine gynecologic care. She had a complete hysterectomy 3 months ago after cervical cancer was detected. Which statement does the nurse know to be true regarding this visit? a. Her cervical mucosa will be red and dry looking. b. She will not need to have a Pap smear performed. c. The nurse can expect to find that her uterus will be somewhat enlarged and her ovaries small and hard. d. The nurse should plan to lubricate the instruments and the examining hand adequately to avoid a painful examination. 35. The nurse is preparing to examine the external genitalia of a school-age girl. Which position would be most appropriate in this situation? a. In the parent’s lap b. In a frog-leg position on the examining table c. In the lithotomy position with the feet in stirrups d. Lying flat on the examining table with legs extended 36. When assessing a newborn infant’s genitalia, the nurse notices that the genitalia are somewhat engorged. The labia majora are swollen, the clitoris looks large, and the hymen is thick. The vaginal opening is difficult to visualize. The infant’s mother states that she is worried about the labia being swollen. How should the nurse reply? a. “This is a normal finding in newborns and should resolve within a few weeks.” b. “This finding could indicate an abnormality and may need to be evaluated by a physician.” c. “We will need to have estrogen levels evaluated to ensure that they are within normal limits.” d. “We will need to keep close watch over the next few days to see if the genitalia decrease in size.” 37. During a vaginal examination of a 38-year-old woman, the nurse notices that the vulva and vagina are erythematous and edematous with thick, white, curdlike discharge adhering to the vaginal walls. The woman reports intense pruritus and thick white discharge from her vagina. The nurse knows that these history and physical examination findings are most consistent with which condition? a. Candidiasis b. Trichomoniasis c. Atrophic vaginitis d. Bacterial vaginosis 38. A 22-year-old woman is being seen at the clinic for problems with vulvar pain, dysuria, and fever. On physical examination, the nurse notices clusters of small, shallow vesicles with surrounding erythema on the labia. Inguinal lymphadenopathy is also present. What do these findings indicate? a. HPV b. Pediculosis pubis c. Contact dermatitis d. Herpes simplex virus type 2 39. When performing an external genitalia examination of a 10-year-old girl, the nurse notices that no pubic hair has grown in and the mons and the labia are covered with fine vellus hair. According to the Sexual Maturity Rating scale, what stage of sexual maturity do these findings indicate? a. 1 b. 2 c. 3 d. 4 40. A 46-year-old woman is in the clinic for her annual gynecologic examination. She voices concern about ovarian cancer because her mother and sister died of it. Which statement does the nurse know to be correct regarding ovarian cancer? a. Ovarian cancer rarely has any symptoms. b. The Pap smear detects the presence of ovarian cancer. c. Women over age 40 years should have a thorough pelvic examination every 3 years. d. Women at high risk for ovarian cancer should have annual transvaginal ultrasonography for screening. 41. During a bimanual examination, the nurse detects a solid tumor on the ovary that is heavy and fixed, with a poorly defined mass. What does this finding suggest? a. Ovarian cyst b. Endometriosis c. Ovarian cancer d. Ectopic pregnancy 42. A 25-year-old woman comes to the emergency department with a sudden fever of 38.3 C and abdominal pain. Upon examination, the nurse notices that she has rigid, boardlike lower abdominal musculature. When the nurse tries to perform a vaginal examination, the patient has severe pain when the uterus and cervix are moved. What do these findings suggest? a. Endometriosis b. Uterine fibroids c. Ectopic pregnancy d. Pelvic inflammatory disease 43. During an external genitalia examination of a woman, the nurse notices several lesions around the vulva. The lesions are pink, moist, soft, and pointed papules. The patient states that she is not aware of any problems in that area. What do these findings likely indicate? a. Syphilitic chancre b. HPV or genital warts c. Pediculosis pubis (crab lice) d. Herpes simplex virus type 2 (herpes genitalis) 44. During an examination, how would the nurse expect the cervical os of a woman who has never had children to appear? a. Everted b. Stellate c. Small and round d. As a horizontal irregular slit 45. A woman has just been diagnosed with HPV or genital warts. The nurse should counsel her to receive regular examinations because this virus makes her at a higher risk for what type of cancer? a. Uterine b. Cervical c. Ovarian d. Endometrial 46. During an internal examination, the nurse notices that the cervix bulges outside the introitus when the patient is asked to strain. How should the nurse document this finding? a. A normal finding b. Uterine prolapse, graded first degree c. Uterine prolapse, graded third degree d. Uterine prolapse, graded second degree 47. A 35-year-old woman is at the clinic for a gynecologic examination. During the examination, she asks the nurse, “How often do I need to have this Pap test done?” Which reply by the nurse is correct? a. “It depends. Do you smoke?” b. “A Pap test needs to be performed annually until you are 65 years of age.” c. “If you have two consecutive normal Pap tests, then you can wait 5 years between tests.” d. “After age 30 years, if you have three consecutive normal Pap tests, then you may be screened every 2 to 3 years.” MULTIPLE RESPONSE 1. The nurse is palpating an ovarian mass during an internal examination of a 63-year-old woman. Which findings of the mass’s characteristics would suggest the presence of an ovarian cyst? (Select all that apply.) a. Fixed b. Poorly defined c. Heavy and solid d. Mobile and solid e. Smooth and round f. Mobile and fluctuant Chapter 28: The Complete Health Assessment: Adult Jarvis: Physical Examination and Health Assessment, 8th Edition MULTIPLE CHOICE 1. A hospitalized patient does not require a full neurologic examination during every shift assessment. What is a method of assessing the neurologic status of a patient without performing a full neurological examination? a. Palpate the carotid pulse. b. Offer the patient a glass of water. c. Look at the significant other throughout the examination. d. Assign the nursing assistant to ask the patient questions and report the findings. 2. The examiner is assessing the extraocular muscles. Which of the following tests would be inappropriate? a. Confrontation test b. Corneal light reflex c. Six cardinal positions of gaze d. Cranial nerve III, IV, and VI testing 3. What should the examiner do during auscultation of breath sounds? a. Listen with the bell of the stethoscope. b. Compare sounds on the left and right sides. c. Listen only to the posterior chest for adventitious sounds. d. Instruct the patient to breathe in and out through the nose. 4. In which situation should the examiner auscultate for carotid bruits? a. Middle-aged or older patient b. Pregnant patient with gestational diabetes c. Patient that reports abdominal pain d. Patient with enlarged, tender cervical lymph nodes 5. When standing with their eyes closed, feet together, and arms at their sides, a patient sways and starts to fall. How should the nurse document this finding? a. Positive Romberg sign b. Positive Babinski sign c. Positive Ortolani sign d. Positive modified Allen test 6. The nurse is conducting a hearing screening. Which technique will the nurse use during the whisper test? a. The nurse pulls the pinna up and back. b. The nurse covers their lips to obscure them from view. c. The nurse asks the patient to repeat 3 letters or numbers. d. The nurse stands 4 feet away from the patient and whispers three different words. 7. When auscultating heart sounds, which technique should the nurse use? a. Listen with the bell. b. Listen with the diaphragm. c. Listen with both the diaphragm and bell working from apex to base in a Z pattern. d. Listen with both the bell and diaphragm comparing sides of the heart as progress from apex to base. 8. The nurse is assessing the cranial nerves. To assess cranial nerve XII, what should the nurse ask the patient to do? a. Say “ahh”. b. Stick out tongue. c. Smile and then frown. d. Follow the nurses fingers through the six cardinal positions of gaze. 9. When performing a health history, the nurse would note immunizations under which category? a. Family history b. Personal history c. Past medical history d. History of present illness 10. While conducting a musculoskeletal assessment the nurse stands behind the patient and has the patient bend over and touch his or her toes. What is the nurse assessing? a. Balance b. The spine c. Cervical range of motion d. External rotation of hips 11. The nurse is preparing to perform an examination of the eyes. Which test will the nurse conduct to assess the patient’s vision? a. Weber test b. Snellen test c. Confrontation test d. Corneal light reflex 12. Which statement is true regarding the recording of data from the history and physical examination? a. Use long, descriptive sentences to document findings. b. Record the data as soon as possible after the interview and physical examination. c. The examiner should avoid taking any notes during the history and examination because of the possibility of decreasing the rapport with the patient. d. If the information is not documented, then it can be assumed that it was done as a standard of care. MULTIPLE RESPONSE 1. When gathering information relative to a complete health assessment, the nurse should include which in the decision-making process? (Select all that apply.) a. Treat the health assessment as a legal document. b. Use line drawings to explain and record pertinent findings. c. Do not document findings on the computer while the patient is present. d. Gather needed equipment before the start of the health assessment. e. Write down “word for word” what the patient says as evidence of reliable documentation. Chapter 29: The Complete Physical Assessment: Infant, Young Child, and Adolescent Jarvis: Physical Examination and Health Assessment, 8th Edition MULTIPLE CHOICE 1. A 5-year-old child is in the clinic for a checkup. When performing the physical examination, what would the nurse expect? a. Needs to be held on his mother’s lap. b. Is able to sit on the examination table. c. Is able to stand on the floor for the examination. d. Is able to remain alone in the examination room. 2. When assessing the neonate, the nurse should test for hip stability with which method? a. Eliciting the Moro reflex b. Performing the Romberg test c. Assessing the stepping reflex d. Checking for the Ortolani sign 3. Which of these actions is most appropriate to perform on a 9-month-old infant at a well-child checkup? a. Testing for Ortolani sign b. Blood pressure measurement c. Assessment for stereognosis d. Assessment for the presence of the startle reflex 4. A fourteen-year-old male is being seen at the clinic for a well-person exam. How should the nurse proceed? a. Defer assessment of the inguinal area for hernias. b. Have the patient remain in his street clothes and work around them. c. Allow the patient’s mother to remain in the room during the examination. d. Allow the patient to remain in the sitting position for examination of the abdomen. 5. The nurse is conducting a complete physical assessment of a 6-month-old infant. Which assessment should the nurse perform last? a. Blood pressure b. Babinski reflex c. Palpate the testes d. Elicit the Moro reflex Chapter 30: Bedside Assessment and Electronic Documentation Jarvis: Physical Examination and Health Assessment, 8th Edition MULTIPLE CHOICE 1. When entering a patient’s room for the first time, what should the nurse do first? a. Offer the patient something to drink. b. Check the infusion pump settings for accuracy. c. Check the intravenous (IV) infusion site for swelling or redness. d. Make eye contact with the patient, and introduce him or herself as the patient’s nurse. 2. During an assessment, the nurse is unable to palpate pulses in the left lower leg. What should the nurse do next? a. Reassess the pulses in 1 hour. b. Document that the pulses are nonpalpable. c. Use a Doppler device to assess the pulses. d. Ask the patient turn to the side, and then palpate for the pulses again. 3. During a morning assessment, the nurse notices that a patient’s urine output is below the expected amount. What should the nurse do next? a. Perform a bladder scan test. b. Refer the patient to an urologist. c. Obtain an order for a Foley catheter. d. Obtain an order for a straight catheter. 4. What should the nurse assess before entering the patient’s room on morning rounds? a. Patient’s general appearance b. Presence of any visitors in the room c. Posted conditions, such as isolation precautions d. Patient’s input and output chart from the previous shift 5. The nurse has administered a pain medication to a patient by an IV infusion. When should the nurse reassess the patient’s response to the medication? a. Within 5 minutes b. Within 15 minutes c. Within 30 minutes d. Within 60 minutes 6. During an assessment of a hospitalized patient, the nurse pinches a fold of skin under the clavicle and on the forearm. What is the nurse assessing? a. Presence of edema b. Mobility and turgor c. Patient’s response to pain d. Percentage of the patient’s fat-to-muscle ratio 7. When assessing the neurologic system of a hospitalized patient during morning rounds, what should the nurse include during the assessment? a. Blood pressure b. Patient’s ability to communicate c. Patient’s personal hygiene level d. Patient’s rating of pain on a scale of 1 to 10 8. When assessing a patient in the hospital setting, the nurse knows which statement to be true? a. The patient will need a brief assessment at least every 4 hours. b. The patient will need a consistent, specialized examination every 8 hours that focuses on certain parameters. c. The patient will need a complete head-to-toe physical examination every 24 hours. d. Most patients require a minimal examination each shift unless they are in critical condition. 9. The nurse is giving report to the next shift and is using the situation, background, assessment, recommendation (SBAR) framework for communication. Which of these statements reflects the B portion of SBAR? a. “We need an order for oxygen.” b. “He is 4 days postoperative, and his incision is open to air.” c. “I’m worried that his gastrointestinal bleeding is getting worse.” d. “My name is Ms. Smith, and I’m giving the report on Mrs. X in room 1104.” 10. Consider the below scenario phone conversation when answering the following three questions: “Dr. Jones, this is Mary Smith, RN, on the postsurgical unit at City Hospital. I’m calling about Tom King, your 46-year-old patient who had an inguinal hernia repair this morning. He has not voided since surgery, 8 hours ago. He has received 1900 mL Lactated Ringers IV and 720 mL oral fluids. He can’t initiate a stream, but states that he “feels the need to urinate.” His bladder is distended by palpation and shows a volume of 800 mL when scanned with the bladder scanner. We’ve tried standing him to void, providing privacy, and running water, but he is still unable to go. He appears to have urinary retention and I’d like to try using a straight catheter to relieve his retention, what do you think?” In the above scenario, what part of the SBAR communication tool is the underlined information? a. S b. B c. A d. R MULTIPLE RESPONSE 1. The nurse is assessing the IV infusion at the beginning of the shift. Which factors should be included in the assessment of the infusion? (Select all that apply.) a. The IV site date is noted. b. Capillary refill in the fingers is checked and noted. c. Whether the patient is sufficiently voiding is noted. d. Proper IV solution is infusing, according to the physician’s orders. e. The IV solution is infusing at the proper rate, according to physician’s orders. f. The infusion is proper, according to the nurse’s assessment of the patient’s needs. 2. The nurse is completing an assessment on a patient who was just admitted from the emergency department. Which assessment findings would require immediate attention? (Select all that apply.) a. Sudden restlessness b. Temperature: 38.6C c. Oxygen saturation: 95% d. Heart rate: 130 beats per minute e. Systolic blood pressure: 150 mm Hg f. Respiratory rate: 22 breaths per minute Chapter 31: The Pregnant Woman Jarvis: Physical Examination and Health Assessment, 8th Edition MULTIPLE CHOICE 1. Which of these statements best describes the action of the hormone progesterone during pregnancy? a. Progesterone produces the hormone human chorionic gonadotropin. b. Duct formation in the breast is stimulated by progesterone. c. Progesterone promotes sloughing of the endometrial wall. d. Progesterone maintains the endometrium around the fetus. 2. A female patient has nausea, breast tenderness, fatigue, and amenorrhea. Her last menstrual period was 6 weeks ago. What signs of pregnancy is this patient experiencing? a. Positive b. Possible c. Probable d. Presumptive 3. A woman who is 8 weeks’ pregnant is visiting the clinic for a checkup. Her systolic blood pressure is 30 mm Hg higher than her prepregnancy systolic blood pressure. How should the nurse proceed? a. Consider this a normal finding. b. Expect the blood pressure to decrease as the estrogen levels increase throughout the pregnancy. c. Consider this an abnormal finding because blood pressure is typically lower at this point in the pregnancy. d. Recommend that the patient decreases her salt intake in an attempt to decrease her peripheral vascular resistance. 4. A patient is being seen at the clinic for her 10-week prenatal visit. She asks when she will be able to hear the baby’s heartbeat. How should the nurse reply? a. “The baby’s heartbeat is not usually heard until the second trimester.” b. “The baby’s heartbeat may be heard anywhere from the ninth to the twelfth week.” c. “It is often difficult to hear the heartbeat at this point, but we can try.” d. “It is normal to hear the heartbeat at 6 weeks. We may be able to hear it today.” 5. A patient who is in her first trimester of pregnancy tells the nurse that she is experiencing significant nausea and vomiting and asks when it will improve. What is an appropriate response by the nurse? a. “Did your mother have significant nausea and vomiting?” b. “Many women experience nausea and vomiting until the third trimester.” c. “Usually, by the beginning of the second trimester, the nausea and vomiting improve.” d. “At approximately the time you begin to feel the baby move, the nausea and vomiting will subside.” 6. During the examination of a woman in her second trimester of pregnancy, the nurse notices the presence of a small amount of yellow drainage from the nipples. How does the nurse interpret this finding? a. An indication that the woman’s milk is coming in. b. A sign of possible breast cancer in a pregnant woman. c. Most likely colostrum and considered a normal finding at this stage of the pregnancy. d. Too early in the pregnancy for lactation to begin and refers the woman to a specialist. 7. A woman in her second trimester of pregnancy complains of heartburn and indigestion. When discussing this with the woman, the nurse considers which explanation for these problems? a. Tone and motility of the gastrointestinal tract increase during the second trimester. b. Sluggish emptying of the gallbladder, from the effects of progesterone, often causes heartburn. c. Lower blood pressure at this time decreases blood flow to the stomach and gastrointestinal tract. d. Enlarging uterus and altered esophageal sphincter tone predispose the woman to have heartburn. 8. A patient who is 20 weeks’ pregnant tells the nurse that she feels more shortness of breath as her pregnancy progresses. The nurse recognizes which statement to be true? a. High levels of estrogen cause shortness of breath. b. Feelings of shortness of breath are abnormal during pregnancy. c. Hormones of pregnancy cause an increased respiratory effort. d. The patient should get more exercise in an attempt to increase her respiratory reserve. 9. The nurse auscultates a functional systolic murmur, grade ii/iv, on a woman in week 30 of her pregnancy. The remainder of her physical assessment is within normal limits. How should the nurse proceed? a. Consider this finding abnormal, and refer her for additional consultation. b. Ask the woman to run briefly in place and then assess for an increase in intensity of the murmur. c. Know that this finding is normal and is a result of the increase in blood volume during pregnancy. d. Ask the woman to restrict her activities and return to the clinic in 1 week for re-evaluation. 10. A woman who is 28 weeks’ pregnant has bilateral edema in her lower legs after working 8 hours a day as a cashier at a local grocery store. She is worried about her legs. What is the nurse’s best response? a. “You will be at risk for development of varicose veins when your legs are edematous.” b. “I would like to listen to your heart sounds. Edema can indicate a problem with your heart.” c. “Edema is usually the result of too much salt and fluids in your diet. You may need to cut down on salty foods.” d. “As your baby grows, it slows blood return from your legs, causing the swelling. This often occurs with prolonged standing.” 11. When assessing a woman who is in her third trimester of pregnancy, the nurse looks for the classic symptoms associated with preeclampsia. What symptoms is the nurse looking for that would indicate preeclampsia? a. Edema, headaches, and seizures b. Decreased blood pressure and edema c. Elevated blood pressure and proteinuria d. Elevated liver enzymes and high platelet counts 12. The nurse should know that what is the best time to assess a woman’s blood pressure during an initial prenatal examination? a. At the end of the examination when she will be the most relaxed b. At the beginning of the interview as a nonthreatening method of gaining rapport c. During the middle of the physical examination when she is the most comfortable d. Before beginning the pelvic examination because her blood pressure will be higher after the pelvic examination 13. When examining the face of a woman who is 28 weeks’ pregnant, the nurse notices the presence of a butterfly-shaped increase in pigmentation on the face. How should the nurse document this finding? a. Striae b. Chloasma c. Linea nigra d. Mask of pregnancy 14. Which finding is considered normal and expected when the nurse is performing a physical examination on a pregnant woman? a. Palpable, full thyroid b. Edema in one lower leg c. Pale mucous membranes of the mouth d. Significant diffuse enlargement of the thyroid 15. When auscultating the anterior thorax of a pregnant woman, the nurse notices the presence of a murmur over the second, third, and fourth intercostal spaces. The murmur is continuous but can be obliterated by pressure with the stethoscope or finger on the thorax just lateral to the murmur. How does the nurse interpret these findings? a. Murmur of aortic stenosis b. Most likely a mammary souffle c. Associated with aortic insufficiency d. Indication of a patent ductus arteriosus 16. When the nurse is assessing the deep tendon reflexes (DTRs) on a woman who is 32 weeks’ pregnant, which of these would be considered a normal finding? a. Absent DTRs b. 2+ c. 4+ d. Brisk reflexes with clonus 17. When performing an examination of a woman who is 34 weeks’ pregnant, the nurse notices a midline linear protrusion in the abdomen over the area of the rectus abdominis muscles as the woman raises her head and shoulders off of the bed. Which response by the nurse is correct? a. The presence of diastasis recti should be documented. b. This condition should be discussed with the physician because it will most likely need to be surgically repaired. c. The possibility that the woman has a hernia attributable to the increased pressure within the abdomen from the pregnancy should be suspected. d. The woman should be told that she may have a difficult time with delivery because of the weakness in her abdominal muscles. 18. The nurse is palpating the fundus of a pregnant woman. Which statement about palpation of the fundus is true? a. The fundus should be hard and slightly tender to palpation during the first trimester. b. Fetal movement may not be felt by the examiner until the end of the second trimester. c. After 20 weeks’ gestation, the number of centimeters should approximate the number of weeks’ gestation. d. Fundal height is usually less than the number of weeks’ gestation, unless an abnormal condition such as excessive amniotic fluid is present. 19. The nurse is palpating the abdomen of a woman who is 35 weeks’ pregnant and notices that the fetal head is facing downward toward the pelvis. What does this describe? a. Fetal lie b. Fetal variety c. Fetal attitude d. Fetal presentation 20. The nurse is palpating the uterus of a woman who is 8 weeks’ pregnant. Which finding would be most consistent with a normal finding at this stage of pregnancy? a. The uterus seems slightly enlarged and softened. b. It reaches the pelvic brim and is approximately the size of a grapefruit. c. The uterus rises above the pelvic brim and is approximately the size of a cantaloupe. d. The uterus is about the size of an avocado, approximately 8 cm across the fundus. 21. Which best describes the average length of pregnancy? a. 38 weeks b. 9 lunar months c. 280 days from the last day of the last menstrual period d. 280 days from the first day of the last menstrual period 22. A patient’s pregnancy test is positive, and she wants to know when the baby is due. The first day of her last menstrual period was June 14, and that period ended June 20. Using the Nägele rule, what is her expected date of delivery? a. March 7 b. March 14 c. March 21 d. March 27 23. During the assessment of a woman in her 22nd week of pregnancy, the nurse is unable to hear fetal heart tones with the fetoscope. How should the nurse proceed? a. Wait 10 minutes, and try again. b. Use ultrasound to verify cardiac activity. c. Ask the woman if she has felt the baby move today. d. Immediately notify the physician, then wait 10 minutes and try again. 24. A patient who is 24 weeks’ pregnant asks about wearing a seatbelt while driving. Which response by the nurse is correct? a. “Seatbelts should not be worn during pregnancy.” b. “Place the lap belt below the uterus and use the shoulder strap at the same time.” c. “Place the lap belt below the uterus but omit the shoulder strap during pregnancy.” d. “Place the lap belt at your waist above the uterus and use the shoulder strap at the same time.” MSC: Client Needs: Safe and Effective Care Environment: Safety and Infection Control 25. During a health history interview, a 38-year-old woman shares that she is thinking about having another baby. The nurse knows which statement to be true regarding pregnancy after 35 years of age? a. Fertility does not start to decline until age 40 years. b. Occurrence of Down syndrome is significantly more frequent after the age of 35 years. c. Genetic counseling and prenatal screening are not routine until after age 40 years. d. Women older than 35 years who are pregnant have the same rate of pregnancy-related complications as those who are younger than 35 years. 26. A 25-year-old woman is in the clinic for her first prenatal visit. The nurse will prepare to obtain which laboratory screening test at this time? a. Urine toxicology b. Alpha-fetoprotein c. Complete blood cell count d. Carrier screening for cystic fibrosis 27. A woman at 25 weeks’ gestation comes to the clinic for her prenatal visit. The nurse notices that her face and lower extremities are swollen, and her blood pressure is 154/94 mm Hg. The woman states that she has had headaches and blurry vision but thinks it is just because she is tired. What should the nurse suspect? a. Eclampsia b. Preeclampsia c. Preterm labor d. Diabetes type 1 28. During auscultation of fetal heart tones (FHTs), the nurse determines that the heart rate is 136 beats per minute. How should the nurse proceed? a. Document the results, which are within normal range. b. Have the patient change positions and count the FHTs again. c. Immediately notify the physician for possible fetal distress. d. Take the maternal pulse to verify these findings as the uterine souffle. 29. During a woman’s 34th week of pregnancy, she is told that she has preeclampsia. The nurse knows which statement concerning preeclampsia is true? a. Preeclampsia has little effect on the fetus. b. Edema is one of the main indications of preeclampsia. c. Eclampsia only occurs before delivery of the baby. d. Untreated preeclampsia may contribute to restriction of fetal growth. MULTIPLE RESPONSE 1. During a group prenatal teaching session, the nurse teaches Kegel exercises. Which statements would be appropriate for this teaching session? (Select all that apply.) a. “Kegel exercises should be performed twice a day.” b. “Kegel exercises should be performed 50 to 100 times a day.” c. “Kegel exercises help keep your uterus strong during the pregnancy.” d. “To perform Kegel exercises, rapidly perform alternating squeeze-release exercises up to the count of eight.” e. “To perform Kegel exercises, slowly squeeze to a peak at the count of eight, and then slowly release to a count of eight.” Chapter 32: Functional Assessment of the Older Adult Jarvis: Physical Examination and Health Assessment, 8th Edition MULTIPLE CHOICE 1. The nurse is assessing an older adult’s functional ability. Which definition correctly describes one’s functional ability? a. It denotes an older person’s cognitive level. b. It is the measure of the expected changes of aging that one is experiencing. c. It describes the individual’s motivation to live independently. d. It refers to one’s ability to perform activities necessary to live in modern society. 2. The nurse is preparing to perform a functional assessment of an older patient. What is an appropriate approach for the nurse to take? a. Observe the patient’s ability to perform the tasks. b. Ask the patient’s wife how he does when performing tasks. c. Review the medical record for information on the patient’s abilities. d. Ask the patient’s physician for information on the patient’s abilities. 3. The nurse needs to determine an older adult’s competence and maintenance of life skills in order to determine the most suitable living situation for them. What tool should the nurse use for this assessment? a. Katz Index of ADL b. Lawton IADL scale c. Montreal Cognitive Assessment (MoCA) d. Mini-Mental State Examination (MMSE) 4. The nurse is preparing to use the Lawton IADL instrument as part of an assessment. Which statement about the Lawton IADL instrument is true? a. The nurse uses direct observation to implement this tool. b. The Lawton IADL instrument is designed as a self-report measure of performance rather than ability. c. This instrument is not useful in the acute hospital setting. d. This tool is best used for those residing in an institutional setting. 5. The nurse is assessing an older adult’s advanced activities of daily living (AADLs), which would include a. Recreational activities b. Meal preparation c. Balancing the checkbook d. Self-grooming activities 6. When using the various instruments to assess an older person’s ADLs, what should the nurse keep in mind as a disadvantage of these instruments? a. Reliability of the tools b. Lack of confidentiality during the assessment c. Self or proxy reporting of functional activities d. Insufficient details concerning the deficiencies identified 7. An 85-year-old man has been hospitalized after a fall at home, and his 86-year-old wife is at his bedside. She tells the nurse that she is his primary caregiver. What should the nurse assess the patient’s wife for as a sign of possible caregiver burnout? a. Depression b. Weight gain c. Hypertension d. Social phobias 8. During a functional assessment of an older person’s home environment, which statement or question by the nurse is most appropriate regarding common environmental hazards? a. “These low toilet seats are safe because they are nearer to the ground in case of falls.” b. “Do you have a relative or friend who can help to install grab bars in your shower?” c. “These small rugs are ideal for preventing you from slipping on the hard floor.” d. “It would be safer to keep the lighting low in this room to avoid glare in your eyes.” 9. When beginning to assess a person’s spirituality, which question by the nurse would be most appropriate? a. “Do you believe in God?” b. “What religious faith do you follow?” c. “Do you believe in the power of prayer?” d. “How does your spirituality relate to your health care decisions?” 10. The nurse is preparing to assess an older adult and discovers that the older adult is in severe pain. Which statement about pain and the older adult is true? a. Pain is inevitable with aging. b. Older adults with cognitive impairments feel less pain. c. Alleviating pain should be a priority over other aspects of the assessment. d. Completion of the assessment should take priority so that care decisions can be made. 11. A patient will be ready to be discharged from the hospital soon, and the patient’s family members are concerned about whether the patient is able to walk safely outside alone. Which tool or test would be best to assess this? a. Lawton IADL scale b. Timed Up and Go Test c. Katz Index of ADL scale d. Geriatric Depression Test 12. During a routine well-person checkup, the daughter of an older patient mentions to the nurse notices she has noticed her mother is less attentive and sometimes unable to recall events from a previous day. The daughter said this does not happen all the time, but that she has noticed it twice in the last month. Which test would be best to assess this patient’s mental status? a. Timed Up and Go Test (TUG) b. Montreal Cognitive Assessment (MoCA) c. Mini-Mental State Examination (MMSE) d. Geriatric Depression Scale, short form MULTIPLE RESPONSE 1. The nurse is assessing the abilities of an older adult. Which activities are considered IADLs? (Select all that apply.) a. Walking b. Toileting c. Feeding oneself d. Preparing a meal e. Grocery shopping f. Balancing a checkbook [Show More]

Last updated: 1 year ago

Preview 1 out of 390 pages

Reviews( 0 )

$22.00

Add to cart

Instant download

Can't find what you want? Try our AI powered Search

OR

GET ASSIGNMENT HELP
151
0

Document information


Connected school, study & course


About the document


Uploaded On

Jul 01, 2020

Number of pages

390

Written in

Seller


seller-icon
Expert1

Member since 4 years

882 Documents Sold


Additional information

This document has been written for:

Uploaded

Jul 01, 2020

Downloads

 0

Views

 151

Document Keyword Tags

Recommended For You


$22.00
What is Browsegrades

In Browsegrades, a student can earn by offering help to other student. Students can help other students with materials by upploading their notes and earn money.

We are here to help

We're available through e-mail, Twitter, Facebook, and live chat.
 FAQ
 Questions? Leave a message!

Follow us on
 Twitter

Copyright © Browsegrades · High quality services·